Polity MCQ 300 by (Saurabh Kumar)

You might also like

Download as pdf or txt
Download as pdf or txt
You are on page 1of 124

‘POLITY – 300’

“This booklet contains 300 questions with detailed explanation


and notes on Polity and governance covering the basic and static
concepts and other current updates from various sources”.

Specially designed for UPSC Prelims 2024

By

Saurabh Kumar
For all current updates on polity and governance, please follow Saurabh Sir's Telegram Channel
"Polity by Saurabh Kumar”

1. Which of the following is the most important 2. The President appoints the council of
feature of Indian Republic? ministers on the advice of the Prime
minister.
(a) Head of the state is chosen by the universal
3. The President can veto any law made by
adult franchise.
the legislatures.
(b) Office of head of the state is open for all the
citizens of the country. How many of the statements given above
(c) Head of the state is directly responsible to is/are correct?
the people of the country. (a) Only one
(d) Head of the Government can be changed (b) Only two
directly or indirectly by the people of the (c) All three
country. (d) None

2. Which of the following is the most appropriate


5. With reference to the parliamentary form of
definition of a limited government?
government in India, consider the following
(a) A govt. that operates with limited powers statements:
when the elections are due and model code
1. The President and the Prime minister both
of conduct is in operation.
are elected indirectly by the people.
(b) A subsidiary govt. that owes its sovereignty
2. The Prime minister can be a member from
to any other government.
any of the houses of Parliament.
(c) A govt. that rules within the limits set by
3. The Council of ministers is responsible to
constitutional law.
the Lok Sabha only.
(d) A govt. that has lost the majority in the
House of people before the completion of its How many of the statements given above
tenure. is/are correct?
(a) Only one
3. Which of the following arguments against (b) Only two
democracy is/are correct? (c) All three
(d) None
1. Democracy leads to instability and delay in
decision making. 6. With reference to the uniqueness of Indian
2. Direct democracy is suitable only for the secularism, consider the following statement:
countries that have a sufficiently smaller
population. 1. It allows everyone to freely practice and
propagate a particular religion without any
Which of the statements given above is/are
limitation.
correct?
2. Every religious group is free to manage its
(a) 1 only
religious affairs.
(b) 2 only
3. It follows a principled and equal distance
(c) Both 1 and 2
from all religions.
(d) Neither 1 nor 2

4. With reference to the Presidential form of How many of the statements given above
government, consider the following is/are correct?
statements: (a) Only one
(b) Only two
1. The President is both the head of the state (c) All three
and the head of the government. (d) None

1
For all current updates on polity and governance, please follow Saurabh Sir's Telegram Channel
"Polity by Saurabh Kumar”

7. Which of the following is regarded as the heart 2. A Party must win at least four percent
and soul of the Indian constitution? seats in the Assembly elections to more
than two states.
(a) The preamble
(b) Directive principles of state policy
Select the correct code:
(c) The right to constitutional remedies
(a) 1 only
(d) Both b and c
(b) 2 only
8. With respect to the ‘Federal form of (c) Both 1 and 2
government’, consider the following statement: (d) Neither 1 nor 2

1. It helps in accommodating regional 11. In order to contest elections in India, a


diversity. candidate must declare the details of which of
2. It guarantees cooperation between federal the following:
and provincial governments as well as
among the provincial governments. 1. All the civil and criminal cases pending
3. It promotes unity in the country. against the candidate.
2. Details of the assets and liabilities of the
How many of the statements given above candidate and his/her family.
is/are correct? 3. Education qualification of the candidate.
(a) Only one
(b) Only two How many of the statements given above
(c) All three is/are correct?
(d) None (a) Only one
(b) Only two
9. With reference to the political parties, consider (c) All three
the following statements: (d) None

1. The Constitution has laid down detailed 12. Consider the following pairs:
criteria for a political party to be a
recognised party. # List 1 List 2
2. The Election Commission of India has Power shared among Community
recognised only six political parties as 1. different organs of the government
National parties. government.

Power shared among Separation of


Which of the statements given above is/are
2. governments at powers
correct?
different levels.
(a) 1 only
(b) 2 only Power shared by Coalition
(c) Both 1 and 2 3. different social groups. government
(d) Neither 1 nor 2 Power shared by two or Federal
4. more political parties. government
10. Which of the followings is/are the conditions
for a political party to become a National How many of the pairs given above are correct?
party? (a) Only two
(b) Only three
1. A Party must secure at least six percent of
(c) All four
the total votes in an election to the Lok
(d) None
Sabha.

2
For all current updates on polity and governance, please follow Saurabh Sir's Telegram Channel
"Polity by Saurabh Kumar”

13. Consider the following subjects: 16. With reference to the scheduled languages in
India, consider the following statements:
1. Education
2. Forest 1. There are 22 scheduled languages under
3. Administration of judgement the constitution of India.
4. Cryptocurrency 2. States may have their own list of scheduled
5. Cyber Security languages.
3. A candidate in an examination conducted
Which of the subjects given above belong to the for a central government position may opt
concurrent list of Indian Constitution? to take examination in any of these
(a) 1, 2 and 5 only languages.
(b) 1, 2 and 4 only
Which of the statements given above is/are
(c) 2, 3 and 4 only
correct?
(d) 1, 2 and 3 only
(a) 1 and 2 only
(b) 2 and 3 only
14. The Constitution of India is a blend of rigidity
(c) 1 and 3 only
and flexibility. Many of its provisions can be
(d) 1, 2 and 3
amended solely by the Parliament whereas
others require a complex and lengthy process.
17. How many of the following is considered as the
With reference to this, a bill must satisfy which
violation of fundamental rights:
of the following conditions in order to amend a
provision of the constitution that relates to the 1. Not paying minimum wages.
sharing of power between the union and the 2. Not allowing workers to go on strike.
states: 3. Banning a movie.
1. The bill must be passed in both the houses
of parliament separately, with the majority Select the correct answer using the codes given
of at least two-third of the members below:
present and voting. (a) 1 and 2 only
2. The bill must be ratified by the legislatures (b) 2 only
of at least half of the total states, with the (c) 3 only
majority of at least one-half of the (d) 1, 2 and 3
members present and voting.
18. With reference to the First Past the Post
System, consider the following statements:
Which of the statements given above is/are
correct? 1. The entire country is divided into smaller
(a) 1 only constituencies.
(b) 2 only 2. Every constituency elects only one
(c) Both 1 and 2 representative.
(d) Neither 1 nor 2 3. Every political party gets seats in
proportion to the percentage of votes polled
15. Which of the following is not an essential for it.
feature of ‘federalism’?
How many of the statements given above
is/are correct?
(a) Written constitution.
(a) Only one
(b) Independent judiciary.
(b) Only two
(c) Division of power among different
(c) All three
constituent units of government equally.
(d) None
(d) None of the above

3
For all current updates on polity and governance, please follow Saurabh Sir's Telegram Channel
"Polity by Saurabh Kumar”

19. With reference to Indian federalism, consider 2. Power is being shared among different
the following statements: Organs of the Government.
3. Power is being Shared among different
1. Only the central government can make
Social Groups such as Religious and
laws only on the subjects mentioned in the
linguistic groups.
Union list.
2. Only state governments can make laws on Choose the correct code:
the subjects mentioned in the state list. (a) 2 Only
3. Central and state governments both can
(b) 1 and 3 Only
make laws on ‘residuary’ subjects.
(c) 1 and 2 Only
How many of the statements given above (d) All of these
is/are correct?
(a) Only one 23. Which of the following are the functions of the
(b) Only two “Constitution”?
(c) All three
(d) None 1. To provide a set of rules that allow for
coordination between members of the
20. Which of the following is Considered as an
society.
important feature of “Parliamentary System of
2. To specify the power who will take
Government”?
decisions in the society.
1. The Ministers are the members of both the 3. To put a limit on the Government Power to
Legislature and Executive. impose restriction on its Citizens.
2. Ministers take oath of Secrecy in the Office
before the Pro-term Speaker. How many of the statements given above
3. It reflects the rule of majority. is/are correct?
(a) Only one
Choose the correct code:
(b) Only two
(a) 1 and 2 Only (c) All three
(b) 2 and 3 Only (d) None
(c) 1 and 3 Only
(d) All of these 24. Consider the following statements:

21. “Supremacy of the Constitution” means: 1. The Constitution is a written document


(a) The Laws enacted by the Centre and States describing the formation and powers of the
must Conform to the Provisions of the Government.
Constitution. 2. The Constitution exists and is required
(b) Parliament draws its authority from the only in democratic Countries.
Constitution. 3. The Constitution is a legal document that
(c) The Constitution draws its authority from deals with ideas and values.
the People. 4. A Constitution gives its Citizens a new
(d) The Constitution can be amended only by identity.
a Special Majority.
How many of the statements given above
22. Which of the following characterises “Power is/are correct?
Sharing” in Modern Democracies? (a) Only one
(b) Only two
1. Power is being Shared between different (c) Only three
levels of the Government. (d) All four

4
For all current updates on polity and governance, please follow Saurabh Sir's Telegram Channel
"Polity by Saurabh Kumar”

25. Which of the following jeopardise the 3. Power to abridge Fundamental Rights.
“Philosophy of the Constitution”? 4. Power to make law on all the matters listed
in the Union list.
1. It appears to have glossed over important
issues like Gender Justice. How many of the statements given above
2. Socio-economic rights are relegated to is/are correct?
Directives Principles, which are Non- (a) Only one
Justifiable. (b) Only two
Which of the statements given above is/are (c) Only three
correct? (d) All four
(a) 1 only
(b) 2 only 29. Consider the following statements which
(c) Both 1 and 2 reflects the basic Principle of “Federalism”:
(d) Neither 1 nor 2 1. Legislative authority is partitioned between
26. Which of the following is the central pillar of Centre and States by the Law.
“Liberalism”: 2. Executive authority is partitioned between
Centre and State by the Constitution.
1. Tolerant to one’s views. 3. Executive authority is partitioned between
2. Right to express in case of disagreement. Centre and State by the Law.
3. Priority of Individual over society. 4. Legislative authority is partitioned between
How many of the statements given above Centre and State by the Constitution
is/are correct?
How many of the statements given above
(a) Only one
is/are correct?
(b) Only two
(a) Only one
(c) All three
(b) Only two
(d) None
(c) Only three
27. Which of the following is/are not “federal (d) All four
features” of the Constitution? 30. Consider the following statements with respect
1. The Constitution is written and not easily to ‘Federalism’:
amenable. 1. Courts have a power to interpret the
2. Equal representation of all states in Rajya Constitution and Powers of the different
Sabha. levels of the Government.
3. State governments derive authority from 2. Sources of Revenue for each level of the
the Centre. Government are clearly specified to ensure
Choose the correct code: its financial autonomy.
3. Different tiers of the Government govern
(a) 2 Only
the same citizens, but each tier has its own
(b) 1 and 3 Only
Jurisdiction.
(c) 1 and 2 Only
(d) All of these How many of the statements given above
is/are correct?
28. Consider the following statements with respect (a) Only one
to the “Constituent Power of the Parliament”: (b) Only two
1. Power to amend Fundamental Rights. (c) All three
2. Power to make law for the enforcement of (d) None
DPSP.

5
For all current updates on polity and governance, please follow Saurabh Sir's Telegram Channel
"Polity by Saurabh Kumar”

31. Which of the following best describes the idea 3. Freedom is about expanding People ’s
of “Justice”? ability to develop themselves.

(a) Equal consideration of all Individuals. How many of the statements given above
(b) Fairness in Giving Opportunity to all is/are correct?
Individuals. (a) Only one
(c) Principle of Proportionate Justice (b) Only two
(d) Recognition of Special needs
(c) All three
(d) None
32. Which of the following is included in “Freedom
of Religion”?
35. Which of the following is the correct
justification of “Freedom of Expression”?
1. It includes persuading people to convert
from one religion to another.
1. No one can always be false.
2. It also provides for not to follow any
2. It is required because conflicting ideas
religion.
result into Right ideas.
3. It is required for creation of Knowledge as
Which of the statements given above is/are
an asset.
correct?
(a) 1 only
Choose the correct code:
(b) 2 only
(c) Both 1 and 2 (a) Only one
(d) Neither 1 nor 2 (b) Only two
(c) All three
33. Which of the following are most appropriate (d) None
explanations for people claiming “Rights”?
36. Consider the following statements with respect
1. Rights should be recognised by the to the claim of “Rights”:
Society.
2. Rights must be sanctioned by the Law. 1. All the Rights are derived from basic Right
3. Rights come with the obligation to respect of Life and Liberty.
other Rights. 2. Rights are legitimate claims that must be
recognised by the Society.
How many of the statements given above 3. Natural Rights keeps a check on
is/are correct? arbitrariness of Government.
(a) Only one
(b) Only two How many of the statements given above
(c) All three is/are correct?
(d) None (a) Only one
(b) Only two
34. Consider the following statements with respect (c) All three
to Idea of “Freedom” in Polity: (d) None

1. It is said to exist when there is an absence


of Constraints.
2. Constraints on Freedom can result from
Social Inequality.

6
For all current updates on polity and governance, please follow Saurabh Sir's Telegram Channel
"Polity by Saurabh Kumar”

37. Which of the following arguments are valid to 40. Which of the following can be attributed to the
Justify State action to provide basic minimum idea of “Collective responsibility”?
Conditions of life to all the Citizens?
1. All the members of a government are
unanimous in support of policy.
(a) Providing free services to the poor and
2. The ministers are responsible for the
needy can be justified as an act of charity.
success and failures of the policies.
(b) Providing all citizens with a basic
3. The Council of ministers is collectively
minimum standard is one way of ensuring
responsible to the Lok Sabha, hence as
equality of opportunity.
soon as the ministry loses its confidence of
(c) Some people in the state are naturally lazy the House, it must resign.
and the State must think of their welfare. 4. The principle of collective responsibility
(d) Ensuring basic facilities and a minimum applies to only ministers of cabinet.
standard of living to all is recognition of our
shared humanity and also stands to be a Choose the correct code:
Human Right. (a) 1 and 3 only
(b) 1, 2 and 3 only
38. Which of the following statements best (c) 3 only
describes "transformative constitutionalism"? (d) All the four

(a) A mechanism that provides legitimacy to a 41. In the context of India, which of the following
democratic government best resembles to the idea of “Socialism”?
(b) Entails instilling principles such as
equality, liberty, fraternity, and dignity
(a) Rule of few for betterment of all.
into society.
(b) Rule of law for the betterment of all.
(c) Adherence to the fundamental rights of the
constitution. (c) Rule of all for the betterment of all.
(d) A government limited by the terms of the (d) Rule of law for the welfare of the
Constitution. marginalized.

39. Consider the following statements with respect 42. Consider the following statements:
to Constitutional Morality:
1. The Constituent Assembly acted as
1. The term has been originated by George Provisional Parliament till 1952.
Grote in the context of Greece and Athens. 2. The Law passed by the Provisional
2. The term was used to highlight the values Parliament enjoys the status of
of Popular sovereignty. Constitutional Provisions.
3. The idea of constitutional morality has 3. The Constituent Assembly ratified the
been mentioned for the first time by India’s membership of the Commonwealth.
judiciary in Kesavananda Bharati case. 4. The Constituent Assembly adopted the
national flag of India.
How many of the statements given above
is/are correct? How many of the statements given above
(a) Only one is/are correct?
(a) Only one
(b) Only two
(b) Only two
(c) All three
(c) Only three
(d) None (d) All four

7
For all current updates on polity and governance, please follow Saurabh Sir's Telegram Channel
"Polity by Saurabh Kumar”

43. Consider the following statements with 46. With reference to committees of the
respect to the “Constituent Assembly”: Constituent Assembly, consider the following
pairs:
1. It was constituted roughly on lines of the
# Person Chairman
Cabinet Mission Proposals.
1. Dr. B.R. Union
2. The authority of the Constituent Assembly
Ambedkar Constitution
comes from the procedures adopted to
Committee
frame the Constitution.
3. Only one provision in the Constituent 2. B. Pattabhi House Committee
Assembly passed without any debate. Sitaramayya
3. Usha Nath Sen Ad-hoc Committee
How many of the statements given above on Citizenship
is/are correct? 4. Sardar Patel State Committee
(a) Only one
(b) Only two How many pairs given above is/are correctly
(c) All three matched?
(d) None (a) Only one pair
(b) Only two pairs
44. Which of the following is the main
(c) Only three pairs
consideration of “Objectives Resolution”?
(d) All four pairs
1. All People shall be guaranteed with Social,
47. Consider the following statements with regard
Political and Economic Justice.
to the “Preamble”:
2. India should promote World Peace and
welfare of Mankind. 1. Preamble may be invoked to determine the
ambit of Fundamental rights and DPSP.
Select the correct code: 2. Preamble was put to vote in Constituent
(a) 1 Only assembly by a motion which says Preamble
(b) 2 Only is a part of the constitution.
(c) Both 1 and 2 Which of the statements given above is/are
(d) Neither 1 nor 2 correct?
(a) 1 Only
45. With reference to the Constituent Assembly, (b) 2 Only
consider the following statements: (c) Both 1 and 2
(d) Neither 1 nor 2
1. The members of the Assembly were elected
on the basis of universal franchise. 48. Which of the following comes under the ambit
2. The British government nominated the of word “Socialism” added in Preamble?
representatives of the princely states in the
Constituent Assembly. 1. Equal pay for Equal work.
3. The Assembly comprised representatives of 2. State Ownership of any Industry.
all sections of the Indian society. 3. Decent standard of living.
4. To reduce Inequalities.
How many of the statements given above
Choose the correct code:
is/are correct?
(a) 1, 3 and 4 Only
(a) Only one
(b) 1 and 4 Only
(b) Only two
(c) 1 Only
(c) All three
(d) All of these
(d) None

8
For all current updates on polity and governance, please follow Saurabh Sir's Telegram Channel
"Polity by Saurabh Kumar”

49. With respect to the Preamble to the 52. Which of the following statements is incorrect?
Constitution of India, consider the following
1. The Constitution of India does not
statements:
guarantee the territorial integrity of any
1. It provides a standard to examine and state of the Union.
evaluate any law and action of government. 2. Under Article 3, Parliament can even cut
2. The Preamble has been amended only once the entire area of a state to form a new
so far. state.
3. The Preamble was enacted by the
Choose the code:
Constituent Assembly.
(a) 1 Only
How many of the statements given above (b) 2 Only
is/are correct? (c) Both 1 and 2
(a) Only one (d) Neither 1 nor 2
(b) Only two
(c) All three 53. With reference to the formation of new States,
(d) None Consider the following arguments:

50. With respect to the Preamble to the 1. Any State can be converted into Union
Constitution of India, consider the following Territory without the consent of the State
statements: legislature.
2. Any State can be converted into Union
1. The Preamble is a source of power to the Territory without the recommendation of
legislature. the President.
2. It was adopted on January 26, 1950.
3. It is a part of the Constitution. Which of the statements given above is/are
4. It plays an important role in the correct:
interpretation of statutes and provisions of (a) 1 Only
the Constitution. (b) 2 Only
(c) Both 1 and 2
How many of the statements given above
(d) Neither 1 nor 2
is/are incorrect?
(a) Only one 54. Which of the following statement is /are
(b) Only two correct?
(c) Only three 1. The JVP Committee was set up to study the
(d) All four recommendations of the Dhar Committee.
2. The JVP Committee did not advocate for
51. Consider the following:
large-scale linguistic reorganisation, it did
1. Ideal of Welfare State make special mention of Andhra Pradesh.
2. Liberty of worship 3. The Dhar Commission recommended the
3. Environment Justice reorganisation of States on the basis of the
4. Economic liberty Geographical contiguity and
5. Secular state administrative viability.
How many of the above are mentioned in the Choose the correct one:
Preamble to the Constitution of India? (a) 1 Only
(a) Only two
(b) 2 and 3 Only
(b) Only three
(c) 1 and 2 Only
(c) Only four
(d) All five (d) All of these

9
For all current updates on polity and governance, please follow Saurabh Sir's Telegram Channel
"Polity by Saurabh Kumar”

55. With reference to the formation of the States, 2. Citizenship Act 2003 provides for dual
consider the following statements: citizenship to Persons of Indian Origin.
3. National identity card is issued by the
1. Daman and Diu and Dadra and Nagar
Central Government to every citizen of
Haveli have been merged by simple
India as per Citizenship Act 2003.
majority.
2. Sikkim has been added to Indian territory Select the correct code:
by Special majority. (a) Only one
3. The acquisition of foreign state does not (b) Only two
require the recommendation of the (c) All three
President. (d) None
Which of the following statements given above 59. Consider the following statements with respect
is /are correct: to Citizenship:
(a) 1 and 3 Only
(b) 3 Only 1. It grants individuals a membership in the
political community.
(c) 2 and 3 Only
2. The idea of Citizenship came into existence
(d) All of these
on 26 January 1950.
56. Which of the following may be impacted 3. All citizens are Nationals of the country,
because of formation of the new States: and all the Nationals are not the citizens of
the country.
1. Schedule One
2. Schedule Four
How many of the statements given above
3. Representative of People act 1951.
is/are correct?
4. Article 240
(a) Only one
5. Article 3
(b) Only two
Choose the correct code: (c) All three
(a) 1, 2, 3 and 5 Only (d) None
(b) 2, 3, and 5 Only
60. Consider the following statements with respect
(c) 1, 2 and 5 Only
to Overseas citizen of India:
(d) 1, 2 and 3 Only
57. Which of the following came into force on “26th 1. Any person who is a citizen of another
Nov 1949”? country but eligible to become citizen of
1. Citizenship by Migration India at the time of commencement of the
2. Citizenship by Registration constitution.
3. Idea of having Election Commission 2. Any person who is a citizen of any country,
but belonged to the territory that became
Choose the correct code: part of India after 15th August 1947.
(a) 1 and 2 Only 3. He is not eligible to enjoy equality in the
(b) 2 Only matter of public employment.
(c) 1 and 3 Only
(d) All of these Choose the correct code: -
58. Which of the following statements with respect (a) 1 and 2 Only
to “Citizenship” is/are incorrect? (b) 1 and 3 Only
(c) 1 and 2 Only
1. One can be a Citizen of India even without (d) All of these
having a domicile of India.

10
For all current updates on polity and governance, please follow Saurabh Sir's Telegram Channel
"Polity by Saurabh Kumar”

61. Consider the following statements with respect (c) All three
to the National Population Register: (d) None
1. A resident identity card (RIC) will be issued 64. Consider the following situations:
only to individuals over the age of 18.
2. It is mandatory to enrol in NPR, and false 1. If an Indian citizen makes a declaration
information amounts to an offence. renouncing his/her Indian citizenship.
3. A respondent will not be required to 2. If an Indian citizen voluntarily acquires the
produce any document for NPR. citizenship of another country.
3. If the citizen has shown disloyalty to the
How many of the statements given above Constitution of India.
is/are correct? 4. If the citizen has traded or communicated
(a) Only one with the enemy during a war.
(b) Only two
(c) All three How many of the above are the ways
(d) None prescribed by the Citizenship Act 1955 for
62. With reference to the Citizenship, consider the losing Indian citizenship?
following statements: (a) Only one
(b) Only two
1. In India both a citizen by birth as well as (c) Only three
a naturalised citizen are eligible for the (d) All four
office of President.
2. The Constitution of India empowers the 65. Which of the following is the most appropriate
Parliament to enact a law with respect to definition of the Right to Equality?
the matters related to citizenship.

Which of the statements given above is/are (a) Giving everyone the same treatment
correct? irrespective of their status in the society.
(a) 1 Only (b) To do away with all sorts of distinctions in
the society.
(b) 2 Only
(c) Both 1 and 2
(c) Providing an atmosphere where everyone
has the same rights and duties.
(d) Neither 1 and 2
(d) Enabling everyone to have an equal
63. Consider the following statements: opportunity to achieve whatever one is
capable of.
1. The constitution of India defines the term
citizen. 66. Which of the statements given below are true
2. There is only one citizenship and one for the Right against Exploitation?
domicile.
3. The constitution of India provides for the (a) No person shall be detained without
citizenship of those individuals who informing the reasons for the detention.
migrated from Pakistan at the time of (b) Right against the practice of all the forms
partition. of untouchability.
(c) Right against discrimination to access
How many of the statements given above public places.
is/are correct?
(d) Prohibition of traffic in human beings.
(a) Only one
(b) Only two

11
For all current updates on polity and governance, please follow Saurabh Sir's Telegram Channel
"Polity by Saurabh Kumar”

67. Which of the following is an instance of an 2. The only limitation on the government is to
exercise of a Fundamental Right? compensate adequately.

1. An Indian from one state may travel freely


Choose the correct code: -
throughout the territory of India and may
(a) 1 only
settle in part until not limited.
2. Any Indian citizen may choose to own land (b) 2 only
in any part of the country to start a (c) Both 1 and 2
business until not limited. (d) Neither 1 nor 2
3. A citizen of India who is accused of any
offence, may choose not to provide his/her 70. Which of the following is part of “Right against
biometric details until not limited. Exploitation”?

Select the correct answer using the codes given


1. Prohibition of Traffic.
below:
2. Prohibition of Employment of Children in
(a) 1 and 3 only hazardous Jobs.
(b) 2 and 3 only 3. Abolition of Untouchability.
(c) 1 and 2 only 4. Equality of Opportunity in Public
(d) 1, 2 and 3 Employment.

68. Which among the following is matched Choose the correct code:
correctly? (a) 1 and 3 only
(b) 1, 2 and 3 only
Writs Situation
(c) 1 and 2 only
1. Mandamus When a particular office
holder is not doing legal (d) All of these
duty
71. The Constitution has provided different
2. Certiorari When a lower court has
“Rights to ensure Fair trial”. Which of the
considered a case going
following are Such Rights?
beyond its jurisdiction
3. Prohibition When a court has to order
1. No person would be punished for the Same
a lower court to transfer a
Offence more than once.
matter pending before it
2. No Law shall declare any action as illegal
Select the correct answer using the codes given retrospectively.
below: 3. No Person shall be asked to give evidence
(a) 1 only against himself.
(b) 2 and 3 only
(c) 1 and 3 only Choose the correct code:
(d) 1, 2 and 3 (a) 1 and 3 only
(b) 2 and 3 only
69. Consider the following Statements with (c) All of these
respect to “Minority Educational Institutions”:
(d) None of these

1. The property can be acquired by the


Government only through enactment of
Law.

12
For all current updates on polity and governance, please follow Saurabh Sir's Telegram Channel
"Polity by Saurabh Kumar”

72. Which of the following Statements is correct 75. Which of the following Fundamental rights are
with regard to “Reservation”? available against “Private Individuals only”?

1. Reservation is a fundamental Right of an 1. Article 15(2)


individual. 2. Article 17
2. Reservation on the basis of “Residence” 3. Article 23
can be made by State Legislature 4. Article 16

Choose the correct code: Choose the correct code: -


(a) 1 only (a) 1, 2 and 3 only
(b) 2 only (b) 2, 3 and 4 only
(c) Both 1 and 2 (c) None of these
(d) Neither 1 nor 2 (d) All of these

73. Which of the following statements validate the 76. Which of the following constitutes the part of
objective of “Fundamental Rights”: “Freedom of Speech and Expression”?

1. These Rights are the consequence of the 1. Right to know Information


Declaration contained in the Preamble. 2. Right to maintain silence.
2. These rights are guarantees of the way of 3. Right to choose the medium of Instruction.
life. 4. No pre censorship of the press.
3. An act of State, whether legislative or
executive, if inconsistent with fundamental Choose the correct code:
rights will be considered null and void. (a) 1 and 2 only
(b) 1, 2 and 3 only
Choose the correct code:
(c) 1, 3 and 4 only
(a) 1 and 2 only
(d) All of these
(b) 2 only
(c) 2 and 3 only 77. Consider the following Statements with
(d) All of these respect to “Free and Fair Trial”:

74. Which of the following Fundamental Rights are 1. It is a sine qua non of Article 20.
available against “Private entity”? 2. It protects against the risk that evidence
will be lost by the passage of time, thus
1. Prohibition of Discrimination on Certain impairing the ability of the accused to
Grounds. defend himself.
2. Abolition of Untouchability
3. Right to Education Choose the correct code:
4. Prohibition of Trafficking (a) 1 only
5. Right to Life (b) 2 only
(c) Both 1 and 2
Choose the correct code:
(d) Neither 1 nor 2
(a) 1, 2 and 3 only
(b) 1 ,3 and 5 only
(c) 1, 2, 3 and 4 only
(d) 1, 2, 3, 4 and 5 only

13
For all current updates on polity and governance, please follow Saurabh Sir's Telegram Channel
"Polity by Saurabh Kumar”

78. Which of the following is correct with respect (c) 1 and 3 only
to Freedom of Conscience”: (d) All of these
1. It has no necessary connection with any
particular religion or any faith in God. 81. Which of the following is the part of the “Right
2. It implies the right not to be converted into to establish and administer an institution”
another religion and to belong to any under Article 30?
religion at all. 1. Right to take disciplinary action.
Choose the correct code: - 2. Right to impart Religious institutions.
3. Right to select students and principal.
(a) 1 only
(b) 2 only Choose the correct code:
(c) Both 1 and 2 (a) 1 and 2 only
(d) Neither 1 nor 2 (b) 2 only
(c) 2 and 3 only
79. Consider the following statements with respect
(d) All of these
to “Preventive Detention”:

1. It is the right of detenu to be informed of 82. Consider the following statements with regard
the ground of detention. to “Fundamental Rights”:
2. The first law relating to Preventive
detention was enforced in 1950. 1. Not all the Fundamental rights are directly
3. One can’t be detained beyond three enforceable.
months without the approval of the 2. Law for the Preventive detention can be
Advisory Committee. enacted by the State Legislature also but
only in case of national security.
Choose the correct code: -
(a) 1 and 2 only Choose the correct code:
(b) 2 and 3 only (a) 1 only
(c) 3 only (b) 2 only
(d) All of these (c) Both 1 and 2
80. Which of the following correctly differentiates (d) Neither 1 nor 2
between “Article 26 and Article 30”?
83. The right to move to the Court under “Article
1. Article 26 provides the right to maintain 32” can be suspended in the case of:
the institution while Article 30 provides for
the right to administer the institution. 1. Suspension of Fundamental Rights of
2. Article 26 is subjected to public order, Members of Armed forces.
while article 30 is free from such 2. Proclamation of emergency under Article
limitations. 359.
3. Article 26 gives right to establish 3. Restriction on Fundamental Rights under
institution to the majority as well as Martial Law.
minority while article gives right to
establish institution to the minorities only. Choose the correct code:
(a) 1 and 3 only
Choose the correct code: (b) 1 Only
(a) 1 and 2 only (c) 1 and 2 only
(b) 2 and 3 only (d) None of these

14
For all current updates on polity and governance, please follow Saurabh Sir's Telegram Channel
"Polity by Saurabh Kumar”

84. Consider the following statements with regard How many of the above-mentioned
to “Habeas Corpus”: fundamental rights are available to both
1. Only the aggrieved party can ask for the citizens of India and foreign nationals?
enforcement of the writ. (a) Only two
2. This is the oldest writ of common law of (b) Only three
England. (c) Only four
3. It cannot be issued to enforce (d) All five
departmental instruction that does not
possess statutory force. 87. With reference to the Fundamental Rights,
consider the following statements:
How many of the statements given above
is/are correct? 1. Reasonable restrictions can be imposed
(a) Only one on them only by the State.
(b) Only two 2. They are sacrosanct and cannot be
(c) All three repealed by the Parliament.
(d) None 3. All of them are available only against the
arbitrary action of the state.
85. Consider the following statements with respect
to "Fundamental Rights": How many of the statements given above
is/are correct?
1. These rights in India has its origins in (a) Only one
England bill of Rights and US Bill of rights. (b) Only two
2. Fundamental rights can’t be amended by (c) All three
the Parliament after Keshavananda Bharti (d) None
case.
3. They allow concentration of power in the 88. Consider the following:
hands of parliament and executive when
necessary. 1. Right against tapping of telephonic
conversation.
How many of the above statements is/ are 2. Right to know about government
correct? activities.
(a) Only one 3. Right to answer the criticism.
(b) Only two 4. Right against handcuffing.
(c) Only three 5. Right to demonstration or picketing and
(d) All four strike.

86. Consider the following: How many of the above comes under the ambit
of Freedom of Speech and Expression granted
1. Equality before law and equal protection by Article 19 of the Indian Constitution?
of laws. (a) Only two
2. Right to elementary education. (b) Only three
3. Prohibition of discrimination on grounds (c) Only four
of religion, race, caste, sex, or place of (d) All five
birth.
4. Freedom to manage religious affairs.
5. Protection in respect of conviction for
offences.

15
For all current updates on polity and governance, please follow Saurabh Sir's Telegram Channel
"Polity by Saurabh Kumar”

89. With reference to the Protection in respect of 3. Jains have been recognised as Minority in
conviction for offences, consider the following 2015.
statements:
Choose the correct code:
1. No penalties can be imposed for both (a) Only one
criminal and civil offence retrospectively. (b) Only two
2. The protection under Article 20 extends to
(c) All three
the preventive detention cases.
(d) None of these
3. The protection against double jeopardy is
available only in proceedings before a
92. Consider the following statements:
court of law.
4. No person shall be compelled to give
1. The origin of Fundamental Rights can be
his/her to give thumb impression and
traced to Magna Carta of England.
blood specimens.
2. The Declaration of French Revolution 1789
may be regarded as a concrete political
How many of the statements given above
statement on Human Rights.
is/are correct?
3. Bill of Rights have been incorporated in the
(a) Only one
US Constitution after the adoption of the
(b) Only two
constitution.
(c) Only three
4. The Indian Constitution guarantees
(d) All four
Human Rights in the form of Fundamental
Rights and Directives Principles of State
90. Consider the following with respect to the
Policy.
Fundamental Rights granted by the
Constitution of India:
How many of the statements given above
is/are correct?
1. Prohibition of Traffic in Human Beings
(a) Only one
and Forced Labour protects the individual
(b) Only two
only against the State.
(c) Only three
2. No person shall be compelled to pay any
(d) All four
taxes or fee for the promotion or
maintenance of any particular religion or
93. Which of the following comes under the
religious denomination.
meaning of State for the purpose of Part III of
the Constitution?
Which of the statements given above is/are
correct?
1. National Council of Educational Research
(a) 1 only
and Training.
(b) 2 only
2. Central Board of Secondary Education
(c) Both 1 and 2
3. Ramkrishna Mission
(d) Neither 1 nor 2
4. Cooperatives

91. Which of the following statement is/are


How many of the statements given above
correct?
is/are correct?
1. The Constitution recognizes only religious
(a) Only one
and linguistic minorities.
(b) Only two
2. The National Commission for Minorities
(c) Only three
Act, 1992 defines minorities as notified by
(d) All four
the Central government.

16
For all current updates on polity and governance, please follow Saurabh Sir's Telegram Channel
"Polity by Saurabh Kumar”

94. Which of the following comes under the How many of the statements given above
meaning of Law according to Article 13 in Part is/are correct?
III of the Constitution? (a) Only one
(b) Only two
1. Bye Laws of Statutory body
(c) All three
2. Regulations made by LIC.
(d) None
3. Personal Laws
4. Administrative instruction
97. Consider the following statements:
How many of the statements given above
is/are correct? 1. Article 23 permits the State to impose
(a) Only one compulsory services for national interest.
(b) Only two 2. The provision of Article 24 can be
(c) Only three enforceable only by the law.
(d) All four 3. The Constitution of India does not define
the meaning of Religious denomination
95. Consider the following Statements regarding under Article 26 of the constitution.
Right to Equality: 4. Article 27 is violated if the tax’s proceeds
would go towards the promotion or
1. Rule of Law is the part of Equality before maintenance of a particular religion or
law. religious denomination.
2. Equal protection of the law is incurred
from the 14th Amendment Act of the US How many of the statements given above
constitution. is/are correct?
3. The laws enacted by the Parliament to (a) Only one
enforce any of the directive Principles (b) Only two
cannot be challenged on the ground that it (c) Only three
violates Article 14. (d) All four

How many of the statements given above 98. Consider the following statements regarding
is/are correct? Right to Freedom:
(a) Only one
(b) Only two 1. Freedom of Assembly can be limited on the
(c) All three ground of sovereignty and integrity of India
(d) None and Public Morality.
2. Freedom of Trade and Business can be
96. Which of the following clearly demarcates limited on the ground of Public order.
Martial law and National Emergency? 3. Freedom of Movement can be restricted on
the ground of public interest.
1. Martial law can be imposed in the whole of
India while National emergency can be Choose the correct code:
imposed in part or whole of India. (a) 1 and 3 only
2. Martial law suspends court only while
(b) 2 Only
National emergency suspends both the
(c) 2 and 3 only
government and courts.
3. Martial law affects Fundamental Rights (d) 3 only
while National emergency affects
Fundamental rights and also centre State
relations.

17
For all current updates on polity and governance, please follow Saurabh Sir's Telegram Channel
"Polity by Saurabh Kumar”

99. Consider the following statements with respect 102. Which of the following are “Non- Justifiable
to Freedom of Speech and Expression: Rights”?

1. It is available only to the citizens of India 1. Right against Economic Exploitation.


and not to foreign nationals. 2. Right to Adequate Livelihood.
2. Freedom of speech and expression can be 3. Right to Work.
limited only by the law, not by the 4. Right to Drinking water.
executive action.
3. Right to Freedom of Speech and Choose the correct code:
Expression is exercisable only in India.
(a) 1 and 3 only
Choose the correct code: (b) 2 and 3 only
(a) 1 and 3 only (c) 1 only
(b) 1 only (d) 1, 3 and 4 only
(c) 2 and 3 only
103. Consider the following statements with respect
(d) 1 and 2 only
to “DPSP”:
100. Which of the following statements regarding
the Writs is correct? 1. These directives are forerunners of the UN
Convention on Right to Development.
1. The Writ of Mandamus cannot be issued in 2. These directives do not confer upon or take
the case of violation of contract. away any legislative power from the
2. The Writ of certiorari can be issued against appropriate legislature.
administrative authorities.
3. The writ of Habeas corpus can be issued Choose the correct code:
against Kidnapping of the Person. (a) 1 only
Choose the correct code: (b) 2 only
(a) 1 and 3 only (c) Both 1 and 2
(b) 1 and 2 only (d) Neither 1 nor 2
(c) 2 and 3 only
104. Consider the following statements with respect
(d) None of these
to Directive Principles of State Policy:
101. Which of the following is incorrect with respect
to Directive Principles of State Policy? 1. There is no legal force behind the
Directives.
1. Directive Principles proceeded on the basis 2. Directives are forerunner of the UN
of Human Rights. Convention on Right to Development.
2. The Concept of Public Interest receives 3. The court cannot declare any law as void
orientation from the Directive Principles. on the ground that it contravenes any of
3. Directives do not confer upon or take away the Directive Principles.
any legislation from the appropriate
legislature. How many of the statements given above
is/are correct?
Choose the correct code:
(a) Only one
(a) 2 only
(b) Only two
(b) 3 only (c) All three
(c) 1 and 3 Only (d) None
(d) None of these.

18
For all current updates on polity and governance, please follow Saurabh Sir's Telegram Channel
"Polity by Saurabh Kumar”

105. With respect to the Directive Principles of State (b) Only three
Policy (DPSP) of the Indian Constitution, (c) Only four
consider the following statements: (d) All five

1. Wage disparity between men and women 108. With respect to the Directive Principles of State
violates the DPSP. Policy (DPSP) of the Indian Constitution,
2. It assists the courts in examining and consider the following statements:
determining the constitutional validity of a
law. 1. They aim at establishing political
3. DPSP constitutes limitations upon democracy in the country.
legislative and executive functions. 2. These principles are positive in nature as
4. Separation of judiciary from the executive they require the State to do certain things.
in the public services of the State is
Which of the statements given above is/ are
enjoined by the DPSP.
correct?
How many of the above statements is/ are (a) 1 only
correct? (b) 2 only
(a) Only one (c) Both 1 and 2
(b) Only two (d) Neither 1 nor 2
(c) Only three
(d) All four 109. Consider the statements “Fundamental
Duties”:
106. Which principle among the following was
added to the Directive Principles of State Policy 1. The duties may be referred by the court
by the 86th Amendment to the Constitution? while harmonizing them with
Fundamental Rights.
(a) Equal pay for equal work for both men and 2. Legislation is necessary for their
women enforcement.
(b) Participation of workers in the
management of industries Choose the correct code:
(c) Right of Children to Free and Compulsory (a) 1 Only
Education (b) 2 Only
(d) Securing living wage and human (c) Both 1 and 2
conditions of work to workers (d) Neither 1 nor 2
107. Consider the following: 110. Which of the following Statement is/ are
1. Humane conditions of work and maternity correct regarding the “Fundamental Duties”?
relief
1. It can be made enforceable by Parliament.
2. Educational and economic interests of SCs
2. Any Law in relation to Fundamental Duties
and STs
will be considered unreasonable if it
3. Protect and improve the environment and
violates Right to Equality.
wildlife.
3. Parliament can impose Penalty for failure
4. Prevention of concentration of wealth and
to fulfil Fundamental Duties.
means of production.
5. Secure for all citizens a uniform civil code Choose the correct code:

How many of the above are the Socialist (a) 1 and 3 only
Principles of that are reflected in the Directive (b) 1 and 2 only
Principles of State Policy? (c) 2 and 3 only
(a) Only two (d) All of these

19
For all current updates on polity and governance, please follow Saurabh Sir's Telegram Channel
"Polity by Saurabh Kumar”

111. With reference to the Fundamental Duties of 114. Consider the following statements with respect
an Indian citizen, consider the following to “veto Power”:
statements:
1. The President enjoys a Pocket veto in case
1. The original constitution contained the of Ordinary Bill reserved by the Governor.
Fundamental Duties. 2. The President does not enjoy a Suspensive
2. The Fundamental Duties have been Veto in case of a Constitutional
amended only once so far. Amendment Bill.
3. It is applicable to both citizens and 3. Absolute Veto has been used by the
foreigners. President twice till today.
How many of the above statements is/ are
Choose the correct code:
correct?
(a) Only one (a) 1 and 3 only
(b) Only two (b) 2 and 3 only
(c) Only three (c) 1 and 2 only
(d) All four (d) All of these

112. With reference to the Fundamental Duties of 115. Which of the following statements with respect
an Indian citizen, consider the following to election of “President” is correct?
statements:
1. The drafting committee of the Constituent
1. The Santhanam Committee recommended
assembly explained the formula for the
the inclusion of a separate chapter on
calculation of Vote of MP and MLA.
fundamental duties in the Constitution.
2. An election to fill the vacancy caused by
2. The duty to pay taxes was one of the
the death must be held within six months.
recommendation to be included in
Fundamental Duties of citizens.
Choose the correct code:
3. These duties draw inspiration from the
(a) 1 only
French Constitution.
(b) 2 only
How many of the above statements is/ are (c) Both 1 and 2
correct? (d) Neither 1 nor 2
(a) Only one
(b) Only two 116. Consider the following statements regarding
(c) Only three “Oath of Vice President”:
(d) All four
1. To bear true faith to the constitution.
113. Which duty was added to the Part IVA by the 2. To uphold the constitution of India.
86th Amendment Act to the Constitution? 3. To uphold the law of the land.
4. To discharge duty faithfully.
(a) Free and compulsory education to all
children of age 6-14 years How many of the above statements is/ are
(b) Defend the country and render national correct?
service. (a) Only one
(c) Opportunities for education to his child or (b) Only two
ward between the 6- 14 years (c) Only three
(d) Harmony and the spirit of common (d) All four
brotherhood amongst all.

20
For all current updates on polity and governance, please follow Saurabh Sir's Telegram Channel
"Polity by Saurabh Kumar”

117. Consider the following statements regarding 120. Which of the following Statements is correct
participation of members in election of “Vice with respect to “Council of Minister”?
President”:
1. The number of ministers including Chief
1. Elected Member of Parliament only. Minister should not be less than 12% of the
2. Elected and nominated members of total strength of the State Legislative
Parliament only. assembly.
3. Elected member of State legislative 2. The provision of minimum and maximum
assemblies. strength of CoM was introduced through
the 91st Constitution (Amendment) Act,
How many of the statements given above 2003.
is/are correct?
(a) Only one Choose the correct code:
(b) Only two (a) 1 only
(c) All three (b) 2 only
(d) None
(c) Both 1 and 2
(d) Neither 1 nor 2
118. Which of the following statements is correct
with respect to the discretionary power of the
121. Which among the following belong to the
“Governor of the State”?
electoral college for the election to the
President of India?
1. He can dissolve the State Legislative
assembly in case of President’s rule.
1. All the elected members of both the houses
2. He can appoint any person as Chief
of Parliament.
Minister in case of Hung assembly.
2. All the elected members of State Legislative
Councils.
Choose the correct code:
3. All the elected members of the Legislative
(a) 1 only
Assemblies of Union Territories.
(b) 2 only
(c) Both 1 and 2 Select the correct answer using the codes given
(d) Neither 1 nor 2 below:
(a) 1 only
119. Which of the following are not the Conditions (b) 2 and 3 only
for the “Office of Governor”?
(c) 1 and 3 only
1. The Governor shall not hold any office of (d) 1, 2 and 3
Profit.
2. The Governor is entitled to allowance as
determined by the State Government.
3. The Governor must take the Oath of
upholding sovereignty and integrity.

Choose the correct code:


(a) 1 and 2 only
(b) 1 only
(c) 1 and 3 only
(d) 2 and 3 only

21
For all current updates on polity and governance, please follow Saurabh Sir's Telegram Channel
"Polity by Saurabh Kumar”

122. Consider the following statements, with 3. The President enjoys Pocket veto in the
reference to the office of President in India: case of a State legislature bill.

1. Any citizen of India can contest for the Choose the correct code:
election to the office of President. (a) 1 and 3 only
2. The office has a fixed tenure of five years (b) 2 and 3 only
for the maximum of two terms.
(c) 2 only
3. The office is an integral part of the
(d) All of these
Parliament.

126. Consider the following statements regarding


Which of the statements given above is/are
Vice President:
Incorrect?
(a) 1 and 2 only
1. The Indian Vice President is elected
(b) 2 only separately with the Presidential election
(c) 1 and 3 only while in the USA, the election of President
(d) 1, 2 and 3 and Vice President is held simultaneously.
2. The American Vice President is part of the
123. Which of the following is the best President’s cabinet.
Characteristic of “Parliamentary Executive”? 3. The Vice President of the United States
shall be President of the Senate just like
(a) They are routinely under the Control and the Indian Vice President is the chairman
Supervision of the Legislature. of Rajya Sabha.
(b) They are responsible for day-to-day Policy
making. Choose the correct code:
(c) The Prime Minister must have the support (a) 1 and 3 only
of majority in the Lok Sabha. (b) 1 and 2 only
(d) The Prime Minister decides who will be the (c) All of these
Ministers in the Council of Ministers. (d) None of these

124. The Pardoning Power of the President is: 127. With reference to the Minister in the
government, consider the following
(a) Judicial Power statements:
(b) Executive Power
1. The Salaries of the Prime Minister and
(c) Quasi-Judicial Power
other MP’s are the same.
(d) Concurrent Power
2. The salary of a Minister is equal to the
Member of the Parliament.
125. With reference to the power of President and
3. The Deputy Minister also gets a sumptuary
Governor, consider the following statements:
allowance like the Prime Minister.

1. Governor can’t issue an ordinance on the


How many of the statements given above
matter relating to restrictions on Freedom
is/are correct?
of Trade and commerce.
(a) Only one
2. The President doesn't have Constitutional
(b) Only two
discretion while the Governor has in the
(c) All three
matter of giving assent to the bill.
(d) None

22
For all current updates on polity and governance, please follow Saurabh Sir's Telegram Channel
"Polity by Saurabh Kumar”

128. With reference to the Cabinet Committees, (c) All three


consider the following statements: (d) None

1. The Cabinet Committee on Parliamentary 131. Which of the following is correct with respect
Affairs is headed by the Defence Minister to “Floor Test”?
in 2024.
2. The Cabinet committee on Economic 1. It is done by an effective Majority.
Affairs is headed by the Prime Minister in 2. Legislators may choose not to be present
2024. and vote during the Floor Test.
3. The Cabinet committee on Political Affairs 3. Composite Floor Test means when there is
is headed by the Minister of Parliamentary more than One Person making a claim to
Affairs. form the Government.

How many of the statements given above Choose the correct code:
is/are correct? (a) 1 and 2 only
(a) Only one (b) 2 and 3 only
(b) Only two (c) 3 only
(c) All three
(d) 2 only
(d) None

129. Why is the Prime Minister of India not directly 132. Which of the following Statements are Correct?
elected by the people? Choose the most
appropriate answer? 1. “Session” refers to the time period between
the first meeting of the house and ending
(a) Direct election for Prime Minister will with Prorogation.
involve a lot of expenditure on election. 2. The period between Adjournment of the
(b) Lok Sabha can remove the Prime Minister house and reassembly in a new session is
even before the expiry of his/her term. termed as “Recess”.
(c) Since the Prime Minister is appointed by
the President, there is no need for direct Choose the correct code: -
election. (a) 1 only
(d) In a Parliamentary democracy only the (b) 2 only
leader of the majority party in Lok Sabha (c) Both 1 and 2
can become Prime Minister.
(d) Neither 1 nor 2
130. Consider the following Statements with
respect to “Privilege Motion”: 133. Which of the following are the Instruments of
Parliamentary Control?
1. Rules of Both Lok Sabha and Rajya Sabha
Regulates Privilege Motion.
1. Deliberation and Discussion
2. The Speaker/Chairman can decide on the
2. Approval or Refusal of Laws
privilege motion himself or herself or refer
3. Financial Control
it to the privileges committee of Parliament.
4. No Confidence Motion
3. Privilege Committee can give the
punishment of Imprisonment for the
Choose the correct code:
breach of Privileges.
(a) 1 and 2 only
How many of the statements given above (b) 3 and 4 only
is/are correct?
(c) All of these
(a) Only one
(d) None of these
(b) Only two

23
For all current updates on polity and governance, please follow Saurabh Sir's Telegram Channel
"Polity by Saurabh Kumar”

134. Which of the following calls for “Joint Sitting"? 137. Consider the following Statements with
respect to “Zero Hour”:
1. Ordinary Bill
2. Finance Bill 1. Twenty matters per day as per their
3. Bill involving Expenditure from priority in the ballot are allowed to be
consolidated fund of India. raised during "Zero Hour".
2. The term `Zero Hour' is now formally
Choose the correct code: recognised in Rules of Procedure.
(a) 1 and 3 only 3. The name Zero Hour has been assigned as
(b) 1 and 2 only it starts at 12 Noon.
(c) 1 only
How many of the statements given above
(d) All of these
is/are correct?
(a) Only one
135. Consider the following Statements with
(b) Only two
respect to “Annual Financial Statement”:
(c) All three
(d) None
1. It is a constitutional obligation to mention
Revenue and Capital Expenditure in the
138. With respect to the “Presidential Address” of
Budget.
the House, consider the following statements:
2. The Appropriation Bill contains the
expenditure exceeding the amount shown
1. The Constitution makes it incumbent upon
in the Annual Financial Statement.
the President to address both Houses of
3. An amendment to reduce tax can be
Parliament assembled together at the
brought under Financial Bill only.
commencement of the first Session after
How many of the statements given above each General Election to the Lok Sabha
is/are correct? and at the commencement of the first
(a) Only one Session each year.
(b) Only two 2. No Member can raise questions on the
(c) All three Address by the President.
(d) None 3. Discussion on matters referred to in the
Address takes place on a Motion of Thanks
136. Which of the following is correct with respect moved by a Member and seconded by
to “Calling Attention Motion”? another Member.

1. Only those matters which are primarily the How many of the statements given above
concern of the Union Government can be is/are correct?
raised through a Calling Attention notice. (a) Only one
2. The Calling Attention matter is not subject (b) Only two
to the vote of the House. (c) All three
3. Prior Permission of the Speaker is required (d) None
to bring Calling Attention Motion.

Choose the correct code:


(a) 1 and 3 only
(b) 1 and 2 only
(c) 2 and 3 only
(d) All of these

24
For all current updates on polity and governance, please follow Saurabh Sir's Telegram Channel
"Polity by Saurabh Kumar”

139. Consider the following statements with respect 3. No money from the Consolidated Fund of
to “Motions”: India can be withdrawn without the
consent of the Parliament.
1. The mover of a motion frames it in a form
in which he/she wishes it to be ultimately Choose the correct code:
passed by the House. (a) 1 and 2 only
2. The House has a power to reject the (b) 2 only
motion, but no power to amend it.
(c) 2 and 3 only
3. All the motions are not required to put to
(d) 3 only
Vote.

142. Consider the following statements about “Joint


How many of the statements given above
sitting” of the parliament:
is/are correct?
(a) Only one
1. The speaker of the Lok Sabha summons
(b) Only two
and presides over the joint sitting of both
(c) All three
the houses.
(d) None
2. Joint sitting has been used for first time in
1961 and last time in 1989.
140. Consider the following statements regarding
the “Speaker of Lok Sabha”:
Choose the correct code: -

1. The Speaker of Lok Sabha leaves the office (a) 1 only


just after dissolution of the assembly. (b) 2 only
2. The Business Advisory Committee and (c) Both 1 and 2
Rules Committee work directly under the (d) Neither 1 nor 2
Chairmanship of the Speaker.
3. In the case Speaker decides some bill as a 143. Which of the following Statement is incorrect
money bill, this decision can be challenged with regard to “Deputy Chairman of Rajya
in parliament. Sabha”?

How many of the statements given above 1. A substantive motion is moved after the
is/are correct? date for his election is announced by the
(a) Only one Chairman.
(b) Only two 2. The Deputy Chairman presides over the
(c) All three Rajya Sabha in the absence of the
(d) None Chairman and performs the duties of the
office of the Chairman if the Vice- President
141. Consider the following with reference the is acting as President or if there is a
“Budget”. Which one of the following is vacancy in the office of the Vice-President.
Incorrect?
Choose the correct code:
1. Departmental Standing committees have (a) 1 only
the responsibility to draft and create the (b) 2 only
annual budget.
(c) Both 1 and 2
2. Estimates Committee can impose cuts on
(d) Neither 1 nor 2
the budget if the government cannot justify
expenditure on a particular head.

25
For all current updates on polity and governance, please follow Saurabh Sir's Telegram Channel
"Polity by Saurabh Kumar”

144. Consider the following statements regarding 147. Who among the following is eligible to form the
presentation of “Budget in the Parliament”: Government in the case none of the parties
obtained majority in the “Legislative Assembly
1. The Finance Bill is introduced on the very
of the State”?
first day when the Finance Minister
presents the Budget in the Parliament. 1. The leader of an alliance or coalition
2. Appropriation Bill is introduced after the formed before the election.
voting on demand for grants is over. 2. The leader of an alliance or coalition
formed after the election.
Choose the correct code: 3. The leader of the single largest party in the
(a) 1 only Lower House.
(b) 2 only 4. The leader of the single largest party
(c) Both 1 and 2 ignoring the claim of alliance having a
(d) Neither 1 nor 2 majority is invited by the Governor.
Choose the correct code:
145. Which of the following is correct with respect
(a) 1 and 3 Only
to “Cabinet Committees”?
(b) 1, 3 and 4 Only
1. These posts have been introduced by the (c) 1 and 4 Only
42nd Constitutional Amendment.
(d) All of these
2. These executives in India work as per GOI
Transaction of Business Rules 1961. 148. Consider the following statements:
3. Rules of Business Provides for their 1. Adjournment does not have any effect on
establishment. Pending bills, but Prorogation has.
4. These include cabinet Ministers only. 2. The Bill passed by the State Legislative
assembly but pending in Legislative
Choose the correct code:
Council does not lapse.
(a) 1, 2 and 3 only
(b) 2 and 3 only Choose the correct code:
(c) 1,3 and 4 only (a) 1 only
(d) 2,3, and 4 only (b) 2 only
(c) Both 1 and 2
146. Consider the following Statements with
(d) Neither 1 nor 2
Respect to “Parliamentary Committees”:
149. Which of the following is incorrect with respect
1. Parliamentary Committees draw their to “Legislative Council”?
authority from the Constitution.
1. The members can be reappointed for a
2. The Constitution mentions their
maximum of 2 terms only.
Composition, Tenure, and Functions.
2. The nomination of the members is done at
3. These Committees look after the Cases of
the start of 3rd year by the Governor.
Corruption or advise the Government in
3. Representation of People’s Act 1951
respect of Policy Decisions.
provides for a 6-year term of members of
Legislative Council.
How many of the statements given above
is/are correct? Choose the correct code:
(a) Only one (a) 2 only
(b) Only two (b) 1 and 3 only
(c) All three (c) 1 only
(d) None (d) 2 and 3 only

26
For all current updates on polity and governance, please follow Saurabh Sir's Telegram Channel
"Polity by Saurabh Kumar”

150. Consider the following statements with respect Choose the correct code:
to “Office of Profit”: (a) 1 and 3 only
(b) 1 and 2 only
1. The President decides in case a MP is
(c) 2 and 3 only
holding Office of Profit or not.
2. As per RPA 1951, President accepts the (d) 3 only
Opinion of Election Commission in case of
Office of Profit. 153. Which of the following pairs of statement is
correct?
Choose the correct code:
1. The constitution has a provision for special
(a) 1 only
sitting of the house in the context of
(b) 2 only
national emergency.
(c) Both 1 and 2 2. The term Special session is mentioned in
(d) Neither 1 nor 2 the constitution.
3. Special session of the house was called for
151. Which of the Following is incorrect with the first time during the 1962 war.
respect to “Office of Profit”? 4. The Leader of the House made a pitch to
the Speaker for holding a Special session
1. Holding Office of Profit under Local of the Parliament.
Authority is not a disqualification for being
elected to Lok Sabha or State Legislature. Choose the correct one:
2. Holding Office of Profit under Local (a) Only one
authority means disqualified for being
(b) Only Two
elected as President or Vice President.
(c) Only three
3. Judges of SC and HC cannot contest
Elections because they were working in (d) All four
affairs of the Union and thus holding Office
of Profit. 154. With reference to the working of Parliament,
Consider the following statements:
Choose the correct code:
1. The Secretaries general issues summons to
(a) 1 and 3 only
individual members regarding the
(b) 1 and 2 only
President's call for summoning the house.
(c) All of these 2. The Speaker Pro tem enables the new
(d) None of these members to take oath and also oversee
election of Speaker.
152. Which of the following Statements are 3. A Motion of Thanks is passed in the central
incorrect? hall of Parliament where the President
addresses the house together.
1. Delimitation is a mandate of the
Constitution while readjustment after each How many of the statements given above
Census is not the mandate of the is/are correct?
Constitution. (a) Only one
2. Readjustment can be carried out during (b) Only two
the duration of life of the Lok Sabha. (c) All three
3. Power for enforcement of Delimitation of (d) None
Territorial Constituencies rests with
Parliament.

27
For all current updates on polity and governance, please follow Saurabh Sir's Telegram Channel
"Polity by Saurabh Kumar”

155. Which of the following are not the effects of 3. The Public Account committee is described
Dissolution of the Lok Sabha on the Bill as the twin sister of the Committee on
presented in the houses of Parliament? Public Undertakings.

1. A bill originated in Lok Sabha and is How many of the statements given above
Pending there lapses. is/are correct?
2. A bill originated in Rajya Sabha and (a) Only one
Pending there lapses. (b) Only two
3. A bill passed by both the house and (c) All three
Pending before the President does not (d) None
lapse.
4. A bill originated in Rajya Sabha and 158. With respect to the process of Budget,
passed by it but is Pending in Lok Sabha Consider the following statements:
lapses.
1. The budgets of Union territories and States
Choose the correct one: where President rule is in operation are
(a) 1, 2 and 3 only presented in Lok Sabha.
(b) 2, 3 and 4 only 2. The demand for excess grants are
presented to the house before the end of
(c) 2 only
the financial year.
(d) 1, 3 and 4 only
3. Vote on account is passed by Lok Sabha
156. Consider the following statements with respect without any discussion.
to Parliamentary Privileges:
How many of the statements given above
1. The origin of Parliamentary privileges in is/are correct?
India can be traced to Charter act 1833. (a) Only one
2. The 42nd Constitutional amendment (b) Only two
allowed the media to publish the true (c) All three
reports of the parliamentary proceedings (d) None
except the same related to the house’s
159. With reference to the type of majority required
secret sitting.
in the matter relating to the Parliament,
3. Parliamentarians can use these privileges
consider the following statements:
against the judges too.
1. Absolute majority is required only at the
How many of the statements given above
time of formation of the government.
is/are correct?
2. A majority of the total membership of each
(a) Only one
house and two third majority of members
(b) Only two
present, and voting is required for
(c) All three
proclamation of national emergency.
(d) None
3. A Simple majority is required in the case of
157. Consider the following statements regarding election of the Presiding officer of the
“Parliamentary Committees”: Parliament.

1. The findings of the Privileges Committee How many of the statements given above
are subject to the ultimate decision of the is/are correct?
House. (a) Only one
2. The Deputy Speaker is the chairman of the (b) Only two
Business advisory committee when the (c) All three
office of Speaker is vacant. (d) None

28
For all current updates on polity and governance, please follow Saurabh Sir's Telegram Channel
"Polity by Saurabh Kumar”

160. With respect to “Anti Defection”, consider the 163. Consider the following statements regarding
following statements: Intergovernmental Delegation of power”:

1. Judicial review is not available at a stage 1. It can be done through agreement and
prior to the making of a decision by the legislation both.
Speaker/Chairman. 2. The State can delegate its administrative
2. The Court can review the Speaker's power on the Centre only under the
decision based on violation of agreement with the Centre.
constitutional mandate only. Choose the correct code:
Choose the correct code: (a) 1 only
(a) 1 only (b) 2 only
(b) 2 only (c) Both 1 and 2
(c) Both 1 and 2 (d) Neither 1 nor 2
(d) Neither 1 nor 2 164. Consider the following statements regarding
“Fiscal Federalism”:
161. With respect to “Anti Defection”, consider the
following statements: 1. Centre Imposes taxes on stamp duties but
are collected by the States.
1. The resignation from the party simply 2. The State can’t impose Cess but can collect
means voluntarily giving up membership, if imposed by the Centre.
this calls for disqualification. 3. The Central Government can give grants in
2. If the member has taken prior permission aid to the states who are in the need of
or is condoned by the party within 15 days financial assistance.
from such voting or abstention, the
member shall not be disqualified. How many of the statements given above
is/are correct?
Choose the correct code: (a) Only one
(a) 1 only (b) Only two
(b) 2 only (c) All three
(c) Both 1 and 2 (d) None
(d) Neither 1 nor 2
165. Consider the following Statements with
respect to “Inter State water dispute”: -
162. Consider the following statements with respect
to “Centre- State Relation”: 1. The first water dispute Tribunal is for
Krishna and Mahanadi respectively.
1. If State requests a parliament to make a 2. A River board can be established by the
law on the State List, State ceases to enact central Government even without the
a law on that matter. consent of the State Government
2. The law once enacted by the Parliament 3. SC can issue Mandamus to the Central
will survive for the maximum period of one Government to carry out obligations under
year at a time. Article 262.

How many of the statements given above


Choose the correct code:
is/are correct?
(a) 1 only (a) Only one
(b) 2 only (b) Only two
(c) Both 1 and 2 (c) All three
(d) Neither 1 nor 2 (d) None

29
For all current updates on polity and governance, please follow Saurabh Sir's Telegram Channel
"Polity by Saurabh Kumar”

166. Which of the following truly represents (b) 2 only


‘Administrative relations’ between Centre and (c) Both 1 and 2
State?
(d) Neither 1 nor 2
1. The executive power with respect to
Concurrent List rests upon the Centre only 169. Consider the following statements regarding
if the Constitution provides for the same. “Inter State Council”:
2. States are bound to follow the Centre's
Directives, otherwise they are liable for the 1. All questions at a meeting of the council
President's Rule. are decided by the consensus.
2. Though it is constituted by the President,
Choose the correct code: the procedure to be followed in the meeting
(a) 1 only is decided by the Chairman of the
(b) 2 only Interstate council.
3. The president issued the Interstate council
(c) Both 1 and 2
order 1990 for the setting of Interstate
(d) Neither 1 nor 2
council for the first time.
167. Consider the following with respect to the
Power of “Taxation of State”: How many of the statements given above
is/are correct?
1. A State Legislature can impose tax only on (a) Only one
the supply of goods inside the State. (b) Only two
2. A State Legislature can’t impose tax on the (c) All three
Inter State River Valley Project. (d) None
3. A State Legislature can impose tax on
Goods and services in Course of Inter State 170. With respect to the 15th Finance Commission,
Trade and Commerce. consider the following statements:

How many of the statements given above 1. It is for the first time that Grants in aid has
is/are correct? been provided for the Local Bodies.
(a) Only one 2. It has recommended for Sector specific
(b) Only two Grants which was stopped by 14th
(c) All three Finance Commission.
(d) None 3. The Commission has also recommended
Special Grants to three States.
168. Consider the Following Statements with
respect to “Inter- Governmental Tax How many of the statements given above
Immunities”: is/are correct?
(a) Only one
1. Property and income of Local Authorities (b) Only two
within the State are exempted from Central (c) All three
Taxation. (d) None
2. The Share & Debt of the Central
Government can be taxed by the State
Government.

Choose the correct code:


(a) 1 only

30
For all current updates on polity and governance, please follow Saurabh Sir's Telegram Channel
"Polity by Saurabh Kumar”

171. Consider the following statements with respect 174. With reference to the Delegation of executive
to “National Emergency”: power, consider the following statements:

1. The 44th Amendment Act provides that 1. The Centre can delegate executive power to
emergency can be declared only on the the state with and without conditions.
recommendation of the cabinet. 2. The State can delegate executive function
2. If Lok Sabha is not in session, notice for to the centre only by enacting law for the
revocation of emergency is served to the purpose.
President. 3. The State government can’t delegate
3. The Parliament can modify the transfer of executive function to the centre directly.
Finance from Centre to the State.
How many of the statements given above
How many of the statements given above
is/are correct?
is/are correct?
(a) Only one
(a) Only one
(b) Only two
(b) Only two
(c) All three
(c) All three
(d) None
(d) None

172. Which of the following is invalid during 175. Which of the following institutions classifies to
“President Rule”? be both, a Non constitutional and advisory
bodies?
1.The President can dissolve the State
Legislative Assembly on the aid and advice of 1. Zonal council
Council of Ministers. 2. University Grants commission
2.The President can provide power to the Chief 3. National Integration council
Secretary for State Administration. 4. Niti Aayog

Choose the correct code: Choose the correct answer:


(a) 1 only (a) 1 and 4 only
(b) 2 only (b) 2 and 4 only
(c) Both 1 and 2 (c) 2, 3 and 4 only
(d) Neither 1 nor 2 (d) All of these
173. With reference to the Parliamentary legislation
in the State field, consider the following 176. Consider the following statements regarding
statements? Freedom of Trade, commerce and intercourse:

1. The law enacted by Parliament when Rajya 1. The State legislature can impose
Sabha passes a resolution for the same has reasonable restrictions on such freedom
a maximum lifetime of one year and six but only after the Governor’s consent.
months. 2. The President can appoint an appropriate
2. When the State makes a request to the authority to supervise the provisions of
Parliament to enact a law, then the State such freedom.
can revoke the law after six months.

Choose the correct code: Choose the correct code:


(a) 1 only (a) 1 only
(b) 2 only (b) 2 only
(c) Both 1 and 2 (c) Both 1 and 2
(d) Neither 1 nor 2 (d) Neither 1 nor 2

31
For all current updates on polity and governance, please follow Saurabh Sir's Telegram Channel
"Polity by Saurabh Kumar”

177. With respect to restrictions on Fundamental 2. The money for Statutory Grants are made
Rights during National emergency, consider from Consolidated Fund of India.
the following statement: 3. Union Territories are not funded under the
Statutory Grant.
1. The Right guaranteed under Right to life
can’t be suspended. How many of the statements given above
2. The fundamental rights under Article 19 is/are correct?
can be suspended in whole or part of India. (a) Only one
3. Only the executive action, not the laws (b) Only two
enacted during an emergency cannot be (c) All three
questioned in the court of law. (d) None
How many of the statements given above
180. Which of the following bills can be introduced
is/are correct?
in the Parliament only on the recommendation
(a) Only one
of the President?
(b) Only two
(c) All three
1. A Bill which imposes or varies any tax in
(d) None
which States are interested.
178. Which of the following statement regarding 2. A Bill which imposes any surcharge on tax.
Distribution of Tax Revenue between Centre 3. A Bill to change the name of the State.
and State is correct? 4. A Bill regarding the declaration of
expenditure charged on Consolidated fund
1. The Centre imposes tax on stamp duties of India.
but collected and appropriated by the
State. Choose the correct answer:
2. Every cess is collected after Parliament has (a) 1, 3 and 4 only
authorised its creation through an
(b) 3 and 4 only
enabling legislation that specifies the
(c) 3 only
purpose for which the funds are being
raised. (d) All of these
3. The revenue through direct taxes can be
spent by the government for any public 181. Consider the following statements regarding
purpose in any manner it deems the impact of Financial Emergency:
appropriate for the nation’s good.
1. The President may issue directions for
Choose the correct code: reduction of Salaries of Supreme court
(a) 1 and 3 only Judges.
(b) 2 and 3 only 2. The Parliament may issue directions for
(c) 1 and 2 only reduction of Salaries of any class of
persons serving in the Union.
(d) None of these

Choose the correct answer:


179. Consider the following statements regarding
(a) 1 only
Statutory Grants:
(b) 2 only
1. Grant-in-aid under Article 275(1) of (c) Both 1 and 2
Constitution of India provided by (d) Neither 1 nor 2
Government of India for development of
infrastructure in the tribal areas.

32
For all current updates on polity and governance, please follow Saurabh Sir's Telegram Channel
"Polity by Saurabh Kumar”

182. Which of the following Statements is/are Choose the correct code:
correct with respect to administration of Union (a) 1 and 2 only
Territories? (b) 2 and 3 only
(c) All of the above
1. Union Territory of Jammu and Kashmir (d) None of these
has the same power of legislation on State
List and Concurrent List as Delhi has. 185. With reference to the Union Territories,
2. Lt. Governor can issue Ordinance only consider the following statements:
when assembly is suspended or dissolved. 1. Puducherry and Dadra and Nagar Haveli
3. The number of seats in Puducherry have been created due to cultural
Assembly has been fixed at 30. distinctiveness.
2. An administrator of a union territory
Choose the correct code: serves as the head of state.
(a) 2 and 3 only 3. The Parliament can enact laws on any
(b) 3 only subject from the State List for Jammu and
(c) 1 and 2 only Kashmir.
(d) None of these
How many of the statements given above
183. Consider the following statements with respect is/are correct?
to administration of “Regional council in (a) Only one
Nagaland”: (b) Only two
(c) All three
1. The composition of the Regional council is (d) None
entirely decided by the Governor.
2. The members of Nagaland Legislative 186. With reference to the Union Territories,
Assembly from this area are elected by consider the following statements:
Regional council only.
1. The President of India places union
Which of the statements given above is/are territories under the jurisdiction of the
correct? high court of the adjacent state.
(a) 1 only 2. The Constitution contains separate
(b) 2 only provisions for the administration of
(c) Both 1 and 2 acquired territories.
(d) Neither 1 nor 2 3. The Ministry of Home Affairs is the nodal
ministry for all matters of Union Territories
184. Which one of the following is/are correct with relating to legislation, finance, and budget.
respect to administration of “Tribal areas
under 6th Schedule”? How many of the statements given above
is/are correct?
1. The district Council can have their own (a) Only one
court to hear the appeals from the tribes. (b) Only two
2. Governor can dissolve a district and (c) All three
regional Council on the basis of a report of (d) None
the Commission Constituted to examine
the administration of these areas.
3. The entire State of Meghalaya comes under
Autonomous District Council.

33
For all current updates on polity and governance, please follow Saurabh Sir's Telegram Channel
"Polity by Saurabh Kumar”

187. With reference to scheduled areas and tribal Which of the statements given above is/are
areas, consider the following statements: correct?
(a) 1 only
1. The governor has the power to alter the (b) 2 only
boundary lines of the scheduled areas and
(c) Both 1 and 2
tribal areas.
(d) Neither 1 nor 2
2. The Governor can establish a tribal
advisory council in a state having
190. Consider the following statements, with
scheduled tribes but not scheduled areas.
reference to the writs issued by courts:
Which of the statements given above is/are
1. It can be issued for the enforcement of
correct?
fundamental as well as legal rights.
(a) 1 only
2. It can be issued by the Supreme court as
(b) 2 only
well as the High courts.
(c) Both 1 and 2
3. It can be issued against the actions of the
(d) Neither 1 nor 2
executive as well as the judiciary.

188. With reference to the sixth schedule areas,


How many of the statements given above
consider the following statements:
is/are correct?
(a) Only one
1. The governor is empowered to increase or
(b) Only two
decrease the sixth schedule areas.
(c) All three
2. The governor has the power to constitute
(d) None
courts for the trial of suits and cases
between the tribes.
191. Which of the following statements is incorrect
3. The regional councils make laws on certain
with reference to the independence of the
specified matters that require the assent of
judiciary in India:
the governor.

1. Judges of the Supreme Court can be


How many of the statements given above
removed only on the ground of proved
is/are correct?
misbehaviour or incapacity.
(a) Only one
2. Discussion on the conduct of the judges of
(b) Only two
the Supreme Court is barred in the
(c) All three
Parliament.
(d) None

Which of the statements given above is/are


189. Consider the following statements, with
correct?
reference to the judiciary of India:
(a) 1 only
1. The Supreme court and The High Courts (b) 2 only
both have the power to interpret the (c) Both 1 and 2
constitution of India. (d) Neither 1 nor 2
2. The Supreme court and The High Courts
both can declare any law of the legislature
or the action of the executives as invalid
whether at the union level or at the state
level.

34
For all current updates on polity and governance, please follow Saurabh Sir's Telegram Channel
"Polity by Saurabh Kumar”

192. Consider the following statements with 2. The order of the President to remove the
reference to the powers of Supreme Court: Judge can only be passed if the
impeachment process gets completed
1. Its decisions are binding on all courts
within the same session from both the
within the territory of India.
houses.
2. It can transfer any case from one High
Court to another.
Which of the statements given above is/are
3. It can remove judges of High Courts.
correct?
How many of the statements given above (a) 1 only
is/are correct? (b) 2 only
(a) Only one (c) Both 1 and 2
(b) Only two (d) Neither 1 nor 2
(c) All three
(d) None 196. Consider the following statements with regard
193. Which of the following statements is correct to “Judiciary”:
with reference to an advisory jurisdiction of the
Supreme Court? 1. Parliament decides the salaries of both SC
and HC Judges.
(a) It allows a person to seek advice from the 2. The salary of CJI and Judges of SC is
Supreme Court against the judgments of different.
High Courts. 3. Pension of the HC Judges is charged on
(b) It allows a state to seek advice from the Consolidated Fund of India.
Supreme Court in a matter of conflict with 4. Salaries of HC Judges are charged on the
the centre or any other state. Consolidated Fund of State.
(c) It allows the President to seek advice from
the Supreme Court on a matter of public Choose the correct code:
importance. (a) 1 and 2 only
(d) None of the above
(b) 1, 2 and 3 only
194. Which of the following statements are correct (c) 1, 3 and 4 only
in regard to “Appeal by Special Leave”? (d) All of these

1. The proposed appeal must be against


197. Which of the following is not the part of
Judicial and Quasi-Judicial order only.
“Original Jurisdiction” of Supreme Court?
2. It is available when the order must have
been passed by Court or any Tribunal.
1. Dispute arising out of non-
Which of the statements given above is/are implementation of river dispute Tribunal
correct? award.
(a) 1 only 2. Dispute between Citizens and the GOI.
3. Matters with regard to the Finance
(b) 2 only
Commission.
(c) Both 1 and 2
(d) Neither 1 nor 2 Choose the correct code:
195. Consider the following statements regarding (a) 2 only
“Judiciary”: (b) 1 and 3 only
(c) 3 only
1. The dispute over the age of SC Judge is
decided by Parliament. (d) All of these

35
For all current updates on polity and governance, please follow Saurabh Sir's Telegram Channel
"Polity by Saurabh Kumar”

198. Consider the following statements with respect 201. Which of the following statement is correct?
to “Rule making Power of Judiciary” under
Article 145: 1. The constitution says that HC will consist
of Chief justice and such other judges as
1. The Constitution confers SC the power to determined by the President.
make rules and regulations relating to its 2. The constitution says that the minimum
procedure. number of the judges to decide the case
2. This rule making power is subjected to the relating to interpretation of the
provisions of any law made by Parliament. Constitution should be Five.

Choose the correct code: Choose the correct code:


(a) 1 only (a) 1 only
(b) 2 only (b) 2 only
(c) Both 1 and 2 (c) Both 1 and 2
(d) Neither 1 nor 2 (d) Neither 1 nor 2

199. Consider the following statements with regard 202. Which of the following statements with respect
to “Judiciary”: to ‘Writ Jurisdiction of Supreme Court’ is
incorrect?
1. The Constitution debars SC Judges to
appear before any court and tribunal after 1. SC Can’t refuse to exercise its writ
retirement. Jurisdiction.
2. The judges of HC however can practice in 2. It is because of Writ Jurisdiction that SC is
the Supreme Court or in any High court. called as Protector and Guarantor of the
Constitution.
Which of the statements given above is/are 3. Proceedings under Article 32 can be
correct? transferred by the Supreme Court to the
(a) 1 only High Court and NHRC also.
(b) 2 only
Choose the correct code:
(c) Both 1 and 2
(a) 1 and 3 only
(d) Neither 1 nor 2
(b) 1 only
200. Consider the following Statements with regard (c) 1 and 2 only
to “Judiciary”: (d) All of these

1. SC has a power to make law under Article 203. The final decision in matters of appointment of
141 of Indian Constitution. Judges of Supreme Court rests with: -
2. The judges of the High Court can be
transferred on the charges of corruption. (a) Chief Justice of India
(b) Prime Minister
Which of the statements given above is/are
(c) Council of Ministers headed by Prime
correct?
minister.
(a) 1 only
(d) President
(b) 2 only
(c) Both 1 and 2
(d) Neither 1 nor 2

36
For all current updates on polity and governance, please follow Saurabh Sir's Telegram Channel
"Polity by Saurabh Kumar”

204. Which of the following is incorrect with respect 207. Which of the following Legislation will come
to “Review Petition”? under “Judicial Review”?

1. Only the parties to a case can seek a review


1. Subordinate Legislation
of the judgement.
2. Constitutional Amendment laws.
2. It must be filed within 45 days of the date
3. Administrative action of States.
of judgement.
4. Ordinary Legislation.
3. Only SC can review its judgement as per
Constitution of India.
Choose the correct code:
Choose the correct code: (a) 1, 2, 4 only
(a) 1 and 3 only (b) 2 and 4 only
(b) 2 and 3 only (c) 2 only
(c) 3 only (d) All of these
(d) All of these
208. Consider the following Statements with
205. Consider the following Statements with respect to “Power of Contempt”:
respect to “Subordinate Judiciary”:
1. The Constitution of India empowers the
1. Only the initial Promotion and posting of Supreme Court and High Court
District judge is done by the Governor. respectively to punish people for their
2. Session judges have both Original and respective contempt.
appellate Jurisdiction. 2. The Contempt of Courts Act of 1971
defines the power of the High Court to
Which of the statements given above is/are punish contempt of its subordinate courts.
correct?
(a) 1 only Which of the statements given above is/are
(b) 2 only correct?
(c) Both 1 and 2 (a) 1 only
(d) Neither 1 nor 2 (b) 2 only
(c) Both 1 and 2
206. With reference to the to the Human Rights (d) Neither 1 nor 2
Court, consider the following statements:
209. Which of the following is necessary for
1. These Courts are constituted at State Level admission of “Contempt of Court petition”?
under the supervision of the High Court.
2. These are constituted by the State in 1. Suo-Moto by Supreme Court
Concurrence with the Chief Justice of the 2. On the motion of the Attorney- General or
High Court. the Solicitor-General
3. The State Government specifies Public 3. On motion by any other person with the
Prosecutors for these Courts. consent of the Attorney-General or the
Solicitor-General.
How many of the statements given above
Choose the correct code:
is/are correct?
(a) 1 and 2 only
(a) Only one
(b) Only two (b) 1 only
(c) All three (c) 1 and 3 only
(d) None (d) All of these

37
For all current updates on polity and governance, please follow Saurabh Sir's Telegram Channel
"Polity by Saurabh Kumar”

210. Consider the following statements with respect (b) Only two
to use of “Language in Judiciary”: (c) All three
(d) None
1. The Constitution is silent on the use of
regional language in the High Court. 213. Consider the following statements with respect
2. The Parliament can make the provision for to Family Court:
the use of language other than English in
the Supreme Court. 1. The judges are appointed by the Governor
in consultation with the High court of the
Which of the statements given above is/are state.
correct? 2. It is mandatory for the State Government
(a) 1 only to constitute Family court if the population
(b) 2 only of the town exceeds 1 million.
(c) Both 1 and 2
Which of the statements given above is/are
(d) Neither 1 nor 2
correct?
(a) 1 only
211. Consider the following statements with respect
to “Tribunals”: (b) 2 only
(c) Both 1 and 2
1. Appointment of CAT Members are made (d) Neither 1 nor 2
after Concurrence with Appointment
committee on cabinet. 214. Consider the following statements with respect
2. The minimum age criteria to be appointed to “Permanent Lok Adalat”:
as member of tribunal is 50.
3. Members of Tribunal are not eligible for 1. Permanent Lok Adalat have been set up as
reappointment. permanent bodies with a Chairman and
two members for providing compulsory
How many of the statements given above pre-litigative mechanism for conciliation.
is/are correct? 2. The jurisdiction of the Permanent Lok
(a) Only one Adalat is up to Rs. One Crore.
(b) Only two 3. Every award made by the Permanent Lok
(c) All three Adalat shall be final and binding on all the
(d) None parties.

212. Consider the following statements with respect How many of the statements given above
to “UAPA Tribunal”: is/are correct?
(a) Only one
1. It must be headed by a High court judge. (b) Only two
2. The expense of the tribunal is charged on (c) All three
Consolidated Fund of India. (d) None
3. The Government sent any matter to
tribunal regarding declaration of individual
as terrorist.

How many of the statements given above


is/are correct?
(a) Only one

38
For all current updates on polity and governance, please follow Saurabh Sir's Telegram Channel
"Polity by Saurabh Kumar”

215. Consider the following statements with respect 3. The Supreme Court can issue writs only in
to the “NGT Power of using different principles” the case of appeal, whereas high courts
while deciding environmental cases: can issue writs only when the party directly
approaches it.
1. For the first time, Polluter pays principle
4. High courts can issue writs not only for the
was introduced in Solid Waste
purpose of enforcement of Fundamental
Management Rules 2016.
Rights but also for any other purpose,
2. Strict liability allows exceptions to the
whereas the Supreme Court can issue
industry, if the liability has been accrued
writs only for the purpose of enforcement
by an Act of God.
of Fundamental Rights.
3. Absolute liability offers no exception to
industries involved in hazardous activities. Choose the correct answer:
(a) 1 and 2
How many of the statements given above (b) 2 and 3
is/are correct? (c) 1, 2 and 3
(a) Only one (d) 4 only
(b) Only two
218. Which of the following statements are
(c) All three
incorrect, with reference to Panchayati raj
(d) None
elections in India?
216. Which of the following process is used in 1. The elections are conducted by an
designating Senior Advocates: independent election commission
appointed by the president of India.
(a) It is based on an interview by the
2. One third of the seats are reserved for
committee composed of Judges of SC and
scheduled castes and scheduled tribes.
HC and bar council.
3. The age limit to contest in the elections is
(b) It is done on the basis of the list
25 years.
forwarded by the Bar council.
(c) It is done on the basis of appointment by Select the correct answer using the codes given
the SC and HC judges. below:
(d) It is based on an interview by the (a) 1 and 2 only
committee composed of SC Judge and (b) 3 only
Attorney General of India and nominated (c) 1 and 3 only
members of Bar.
(d) 1, 2 and 3
217. Which of the following statements are 219. Consider the following statements with
incorrect about the difference between the respect to Panchayati Raj Institution:
writ jurisdiction of the Supreme Court and
1. Reduction of corruption in the
high courts in India?
government.
1. The Supreme Court can issue writs not 2. Strengthening of democracy at the grass
only for the purpose of enforcement of root level.
Fundamental Rights but also for any other 3. Reduces the efficiency of policy
purpose, whereas high courts can issue implementation in the government.
writs only for the purpose of enforcement How many of the statements given above
of Fundamental Rights. is/are correct?
2. High courts can issue the writ of In- (a) Only one
junction, whereas the Supreme Court (b) Only two
cannot issue the writ of Injunction. (c) All three
(d) None

39
For all current updates on polity and governance, please follow Saurabh Sir's Telegram Channel
"Polity by Saurabh Kumar”

220. With reference to the Gram Panchayats under 223. Which of the following is the correct
the Panchayati raj system of India, consider explanation of the meaning of “Local
the following statements: authority”?

1. All the voters in the village are its 1. The authority must have a separate legal
members. existence.
2. It is the decision-making body for the 2. It must function in a defined area.
entire village. 3. It must be entrusted by the Statute as are
3. It is a legal body. usually entrusted to Municipal Bodies.

How many of the statements given above Choose the correct code:
is/are correct? (a) 1 and 2 only
(a) Only one (b) 1 and 3 only
(b) Only two
(c) 2 and 3 only
(c) All three
(d) All of these
(d) None
224. Consider the following statement with respect
221. Which of the Following is Correct with respect
to “Urban Governance”:
to “Reservation in Panchayats”?
1. The 74th Constitutional amendment does
1. Reservation for Women does not mean
apply to Union Territory only when the
merely for General Category but also for
President directs so.
Women from SC/ST.
2. District planning committee shall consult
2. Reservation is restricted not only to
such institutions as the Governor specify
ordinary members in the Panchayats but
for drafting developmental plans.
also extends to the Position of Chairperson
at all the three levels.
Which of the statements given above is/are
Select the correct code: correct?
(a) 1 only (a) 1 only
(b) 2 only (b) 2 only
(c) Both 1 and 2 (c) Both 1 and 2
(d) Neither 1 nor 2 (d) Neither 1 nor 2

222. Consider the following statements with respect 225. Which of the following statements are correct
to “Municipal Authorities”: with respect to “Office of CAG”?

1. The term of Mayor is not fixed in India. 1. He compiles the account of the state
2. Mayor is directly elected in some of the government and does auditing of both
states in India. Central and State government.
3. A Mayor is the legislative and executive 2. He ascertains net proceeds of tax.
head of Municipal Corporation. 3. CAG cannot ask for details of a particular
expenditure.
How many of the statements given above
is/are correct? Choose the correct code:
(a) Only one (a) 1 and 2 only
(b) Only two (b) 2 and 3 only
(c) All three (c) 1 and 3 only
(d) None (d) All of these

40
For all current updates on polity and governance, please follow Saurabh Sir's Telegram Channel
"Polity by Saurabh Kumar”

226. Which of the following is incorrect with respect 229. Consider the following statements with respect
to the office of CAG? to “Comptroller and Auditor General of India”:

1. CAG ascertains and certifies the net


1. Under the Government of India Act 1919,
proceeds of any tax or duty and his
the Auditor General became independent
certificate is final on the matter.
of the government as statutory backing
2. The salaries and service conditions of CAG
was given for the position.
is decided by Parliament.
2. In India, CAG is not a member of the
3. He is not eligible for further office, either
parliament while in Britain; CAG is a
under the Government of India or of any
member of the House of the Commons.
state, after he ceases to hold his office.

Choose the correct code: - How many of the statements given above
(a) 1 only is/are correct?
(b) 2 only (a) Only one
(c) Both 1 and 2 (b) Only two
(d) Neither 1 nor 2 (c) All three
(d) None
227. Consider the following statements with respect
to “Audit and accounts on India”: 230. Which of the following is correct with respect
to “Attorney General of India”?
1. Accountant General in state is equivalent 1. The Attorney General of India is
in status to CAG of India. responsible for the amendment and
2. The Muddiman committee has enforcement of laws.
recommended separation of account and 2. This office can be traced to the system of
audit as a necessary financial reform for the USA.
the first time. 3. The Remuneration is the same as the judge
of the Supreme Court.
Which of the statements given above is/are
correct? How many of the statements given above
is/are correct?
(a) 1 only
(a) Only one
(b) 2 only
(b) Only two
(c) Both 1 and 2 (c) All three
(d) Neither 1 nor 2 (d) None

228. Which of the following can’t be audited by the 231. Consider the following Statements with regard
“CAG” directly? to “Additional Advocate General”:

1. The State Government may appoint him in


1. Panchayati Raj Institution discharge of constitutional Power.
2. RBI 2. He will perform such functions as assigned
3. Issues related to Biodiversity. by the Governor from time to time.
4. LIC
Which of the statements given above is/are
Choose the correct code: correct?
(a) 1 and 4 only (a) 1 only
(b) 1, 2 and 4 only (b) 2 only
(c) 3 and 4 only (c) Both 1 and 2
(d) All of these (d) Neither 1 nor 2

41
For all current updates on polity and governance, please follow Saurabh Sir's Telegram Channel
"Polity by Saurabh Kumar”

232. Consider the following statements with respect (c) Both 1 and 2
to “Election machineries”: (d) Neither 1 nor 2
1. Chief Electoral officer is the head of 235. Consider the following Statements with
election machinery at the State level for respect to “Finance Commission”:
State Assembly elections only.
2. Candidates take oath after filing 1. The State Finance Commission has been
nomination before Assistant Returning granted Constitutional status as per the
officer if Returning officer is absent. 74th Amendment for the first Time.
3. No polling station in general should have 2. The members of the Finance Commission
to deal with more than 1500-2000 voters. are eligible for reappointment.
3. Union Finance Commission can be
How many of the statements given above constituted before the term of 5 years.
is/are correct?
(a) Only one How many of the statements given above
(b) Only two is/are correct?
(c) All three (a) Only one
(d) None (b) Only two
(c) All three
233. Consider the following Statements with (d) None
respect to “Electronic Voting Machine”:
236. Consider the following Statement with respect
1. It was first introduced in Kerala in 1978. to the “National Commission for Scheduled
2. It can field a maximum of 64 Candidates. Caste”:
3. It has been synchronised with VVPAT in
2013 in Nagaland Assembly Elections. 1. The President by Notification declares any
Community as Scheduled Caste.
How many of the statements given above 2. His decision can be modified by the Central
is/are correct? Government.
(a) Only one 3. The Commission can submit a report
(b) Only two anytime within a Year.
(c) All three 4. The Commission presents its reports to the
(d) None President or Governor of State Concerned.

234. Consider the following statements with respect How many of the statements given above
to “expenditure in Election”: is/are correct?
(a) Only one
1. In case of a state Legislative assembly (b) Only two
election, entire expenditure is incurred by (c) Only three
the State Government. (d) All four
2. The expenditure on capital equipment is
shared equally between Centre and States.

Which of the statements given above is/are


correct?
(a) 1 only
(b) 2 only

42
For all current updates on polity and governance, please follow Saurabh Sir's Telegram Channel
"Polity by Saurabh Kumar”

237. Which of the following is incorrect with respect 2. Prohibition of Simultaneous Membership
to “National Commission for Minorities”? Rules prescribe a period of 14 days for
1. It consists of a Chairperson, Vice making a choice of the House if one is
Chairperson and 5 members. elected to both State Legislature and
2. All the members shall be from Minority Parliament.
Community. Choose the correct code:
3. It recommends for grant of Minority Status
(a) 1 only
to Educational Institutions.
(b) 2 only
Choose the correct code:
(c) Both 1 and 2
(a) 1 and 2 only
(d) Neither 1 nor 2
(b) 2 and 3 only
(c) 1 and 3 only 241. With reference to the Goods and Services Tax
(d) 3 only Council, consider the following statements:
238. Consider the following Statement with respect
1. It was inserted by the 101st Constitutional
to "Public Services". Which one of the following
Amendment Act of 2016.
is correct:
2. The vision of GST council is to establish
1. Rules and regulations regarding Service the highest standards of co-operative
Conditions of Public Servants can be federation in the functioning of the
notified by the President and Governor. Council.
2. "Reasonable opportunity" of being heard is 3. The Union Finance Minister acts as the ex-
not available to Public Servants in case of officio Secretary to the Council.
criminal charges.
How many of the above statements is/are
Which of the statements given above is/are
correct?
correct?
(a) Only one
(a) 1 only (b) Only two
(b) 2 only (c) All three
(c) Both 1 and 2 (d) None
(d) Neither 1 nor 2
242. With reference to the Goods and Services Tax
239. Which of the following best describes the role Council, consider the following statements:
of Returning officer?
1. Three-fourths of the total number of
(a) He is the person responsible for
members of the Council is the quorum for
preparation of electoral roll in the
conducting a meeting.
Constituency.
2. The Council decisions require a One-half
(b) He is the person who assists Chief electoral
majority of the weighted votes of the
officer while determining star campaigner.
members present and voting at the
(c) He is the person responsible for timely meeting.
start and end of voting in a polling booth.
(d) He administers the oath of candidate after Which of the statements given above is/are
filling of nomination. correct?
240. Which of the following Statements is incorrect (a) 1 only
with respect to "RPA 1951": (b) 2 only
(c) Both 1 and 2
1. The Representation of People's Act 1951 (d) Neither 1 nor 2
provides for vacancy of seat in Parliament
if one is elected to State Legislature also.

43
For all current updates on polity and governance, please follow Saurabh Sir's Telegram Channel
"Polity by Saurabh Kumar”

243. With reference to the Special Officer for 3. The SPSC is consulted for reservations
Linguistic Minorities, consider the following favouring appointments or posts in favour
statements: of any backward class.

1. The Constitution of India provides the How many of the above statements is/are
procedure for removal of the Special correct?
Officer for Linguistic Minorities. (a) Only one
2. At the central level, the Commissioner for (b) Only two
Linguistic Minorities falls under the (c) All three
Ministry of Education. (d) None
3. The Commissioner has his headquarters
at Delhi. 246. With reference to the Finance Commission,
consider the following statements:
How many of the above statements is/are
correct?
1. It is a constitutional and quasi-judicial
(a) Only one body formed under Article 280 of the
(b) Only two Indian Constitution.
(c) All three 2. Constitution of India authorizes President
(d) None to determine the qualifications of members
of the commission.
244. With reference to the Union Public Service 3. K.C. Neogy was the chairman of the 1st
Commission (UPSC), consider the following Finance Commission of India.
statements: 4. The members of the commission are not
1. The Constitution provides provisions eligible for reappointment.
regarding the appointment and removal of
members of the UPSC. How many of the statements given above
2. The Constitution specifies the strength of is/are correct?
the UPSC. (a) Only one
3. The entire expenses of the chairman and (b) Only two
members of the UPSC are charged on the (c) Only three
Consolidated Fund of India. (d) All four

How many of the above statements is/are 247. Consider the following Statement with respect
correct? to "National Human Right Commission:
(a) Only one 1. The Chief Commissioner of persons with
(b) Only two Disabilities is the ex officio Member of
(c) All three NHRC.
(d) None 2. The salaries and service Conditions of the
members are determined by Parliament.
245. With reference to the State Public Service 3. The serving judicial members of NHRC are
Commission (SPSC), consider the following appointed only after the consultation with
statements: CJI.

1. The chairman of SPSC is removed by the Choose the correct code:


President of India. (a) 1 and 3 Only
2. The chairman of SPSC hold office for a (b) 2 and 3 Only
term of six years or until they attain the (c) Only
age of 65 years. (d) All of these

44
For all current updates on polity and governance, please follow Saurabh Sir's Telegram Channel
"Polity by Saurabh Kumar”

248. Which of the following Statements is correct 251. Which of the following statement is correct
with respect to Lokpal: regarding the appointment of DGP:

1. Lokpal cannot initiate Suo-moto 1. Only police officers with at least six months
proceedings against any public servant. of service left before retirement will be
2. It will also provide Legal assistance to the considered for appointment as the Director
Public Servant against whom Complaint is General of Police of a State.
filed. 2. UPSC will not assess those IPS Officer who
are on central deputation for the
Choose the correct code: appointment of DGP.
(a) 1 only 3. The number of shortlisted officers for the
(b) 2 only appointment of DGP cannot exceed three
(c) Both 1 and 2 but may consist of less than three officers.
(d) Neither 1 nor 2
Choose the correct code:
249. Consider the Following Statements with (a) 1 and 3 Only
respect to “National Investigation Agency”: (b) 2 and 3 Only
(c) 1 and 2 Only
1. It is headed by DG Rank Officer. (d) 1 Only
2. It can investigate cases related to UAPA
and Atomic Energy act only. 252. Who is the Chairperson of Delhi Disaster
3. Session Court will act as Special NIA Court Management Authority:
for trial.
(a) Lt Governor
How many pairs of the statement is/are (b) Chief Minister
incorrect: (c) Home Minister
(a) 1 and 2 only (d) Secretary of Delhi
(b) 2 and 3 only
(c) All of these 253. Consider the following statements with respect
(d) None of these to “Central Consumer Protection Authority”:

250. Which of the following statement is correct 1. It will have a Chief Commissioner as head,
regarding Representation of People act 1951: and only two other commissioners as
members.
1. A person contesting from three seats at the 2. It has the power to provide punishment for
same time was abolished in 1996. subsequent offences committed by the
2. Returning Officer could not reject endorser of the product.
nomination papers on the ground that
candidate information was false. Choose the correct code:
3. Election commission can only file FIR (a) 1 only
against a person, failing to lodge an (b) 2 only
account of election expenses. (c) Both 1 and 2
(d) Neither 1 nor 2
Choose the correct code:
(a) 1 and 3
(b) 1 and 2
(c) 1 Only
(d) Except 1 and 3

45
For all current updates on polity and governance, please follow Saurabh Sir's Telegram Channel
"Polity by Saurabh Kumar”

254. Which of the following statements is correct Which of the statements given above is/are
with respect to “Right to Information Act”? correct?
(a) 1 only
1. The authorities under RTI are Quasi-
(b) 2 only
Judicial Authorities.
2. The Second appellate authority in case of (c) Both 1 and 2
appeal is Chief PIO. (d) Neither 1 nor 2
3. NGOs whose 50% financial needs are met
by the Government will be considered as 257. With reference to the Competition Commission
Public Authority. of India (CCI), consider the following
4. The information sought under PMNRF statements:
can’t be rejected while in case of
PMCARES, it can be rejected by PIO. 1. It is a quasi-judicial body established
under Competition Act, 2002.
Choose the correct code: 2. The Union Minister of Corporate Affairs
(a) 1, 2 and 4 only serves as the ex-officio chairman of the
(b) 1 only CCI.
(c) 2, 3 and 4 only 3. It aims to foster a strong competitive
(d) 1 and 4 only environment through proactive
engagement with consumers, industry,
255. Consider the following statements with respect government, and international
to “Right to Information Act”: jurisdictions.

1. The Act recognises Suo-moto Declaration How many of the above statements is/are
of the information by the Public Authority. correct?
2. Every Public authority is under Obligation (a) Only one
to provide Information on written Request (b) Only two
or Request by Electronic means only. (c) All three
3. The definition of Public Authority would (d) None
include any NGO Substantially financed by
Government (Directly or indirectly or both). 258. With reference to the Delimitation
Commission, consider the following
Choose the correct code: statements:
(a) 1 and 2 only
(b) 2 and 3 only 1. It is appointed by the Parliament.
(c) 1 and 3 only 2. The Commission’s orders have the force of
(d) All of these law and cannot be called in question before
any court.
256. Consider the following Statements with 3. The present delimitation of constituencies
respect to Power of “National Commission for has been done on the basis of 2011
protection of child Rights” under Right to census.
education Act:
How many of the above statements is/are
1. It can summon an individual and demand correct?
evidence in case of Violation of law. (a) Only one
2. It can file a writ petition in the High Court (b) Only two
or Supreme Court both in case of any (c) All three
matter related to RTE Act. (d) None

46
For all current updates on polity and governance, please follow Saurabh Sir's Telegram Channel
"Polity by Saurabh Kumar”

259. Consider the following: 262. With reference to the National Investigation
Agency (NIA), consider the following
1. Power to seize assets. statements:
2. Power to summon.
3. Power to arrest 1. It was established against the backdrop of
4. Recovery of Fines, Penalties and Arrears of the 2001 attack on Parliament.
Penalties 2. The Director-General of NIA is appointed
by the central government.
How many of the above-mentioned is/are 3. The NIA aims at creating deterrence for
powers of the Directorate of Enforcement? existing and potential terrorist groups.
(a) Only one
(b) Only two How many of the above statements is/are
(c) Only three correct?
(d) All four (a) Only one
(b) Only two
260. With reference to the Central Bureau of (c) All three
Investigation (CBI), consider the following (d) None
statements:
263. Which of the following laws have been
1. It is a statutory body. amended when The Narcotic drugs and
2. The CBI conducts investigations into Psychotropic substance act was brought in:
organized crime and terrorism.
3. The High Courts can order CBI to 1. Opium act 1857
investigate a crime anywhere in the 2. Opium act 1878
country. 3. Dangerous drugs act 1930.

How many of the above statements is/are Choose the correct code:
correct? (a) 1 and 3 Only
(a) Only one (b) 1 Only
(b) Only two (c) All of these
(c) All three (d) None of these
(d) None
264. Supreme Court has recently recommended for
261. With reference to the Lokpal, consider the forming “Truth and Reconciliation
following statements: Commission” in the context of which of the
1. It is a statutory body. following:
2. The Lokpal has the authority to initiate
proceedings suo motu against any public (a) Electoral Bond
servant. (b) Human Right violation in Jammu and
3. The Prime Minister has been brought Kashmir
under the purview of the Lokpal. (c) Manipur Violence
(d) Extra Judicial Killings
How many of the above statements is/are
correct?
(a) Only one
(b) Only two
(c) All three
(d) None

47
For all current updates on polity and governance, please follow Saurabh Sir's Telegram Channel
"Polity by Saurabh Kumar”

265. Which of the following statements is correct 3. It increases UN entities to use education to
with respect to World Economic Forum (WEF): promote peace, human rights, and
democracy.
(a) WEF launched a program "Giving to
amplify Earth action" where member How many of the above statements is/are
countries will contribute to assist correct?
developing countries in order to achieve (a) Only one
Net zero. (b) Only two
(b) WEF launched a program "Giving to (c) All three
amplify Earth action" where fund is to be (d) None
accumulated by public private partnership
in order to achieve Net zero. 268. Consider the following statements:
(c) The World Economic forum released
"Global risk report' highlighting some of 1. India plans to eradicate Measles and
the risks associated with warming the rubella by 2020 but has not met the
planet. Target.
(d) The World Economic forum along with 2. The vaccine for Measles and rubella both
Forest research institute (India) launched are covered under Mission Indradhanush.
the Fire Aid initiative to predict wildfires. 3. Though Rubella and Measles are not the
same disease but are caused by same
266. Which of the following is the correct difference virus.
between First information report (FIR) and
Chargesheet: How many of the above statements is/are
correct?
1. FIR once filed can't be withdrawn and (a) Only one
charge sheet can be withdrawn but only on (b) Only two
the assent of judicial magistrate and (c) All three
above. (d) None
2. FIR can be filed by victim and Police officer,
but charge sheet can be filed only by Police 269. With respect to UN Habitat: Leaves no one
officer. Behind project, consider the following
3. FIR can be filed anytime but charge sheet statements:
must be filed in maximum of 90 days
without any delay. 1. It draws on the collective expertise of UN
Habitat, UNESCAP and UN-OICT to draw
How many of the above statements is/are urban policy and planning framework.
correct? 2. The project aims to achieve the targets set
(a) Only one under SDG 6 and SDG 11.
(b) Only two 3. The project aims to include disabled and
(c) All three other marginalised groups in the course of
(d) None urban development.
267. Consider the following statements with respect
to "Education for Democracy": How many of the above statements is/are
correct?
1. It is sponsored by India in partnership with (a) Only one
UNICEF. (b) Only two
2. It has been adopted for the second time, (c) All three
first in 2015 and now in 2023. (d) None

48
For all current updates on polity and governance, please follow Saurabh Sir's Telegram Channel
"Polity by Saurabh Kumar”

270. Which of the following is correct with regard to 273. Consider the following statements with respect
World Social Report: to Global action Plan on Child wasting:

1. The framework has been designed by WHO


1. It is published biannually by the United
in collaboration with UNICEF, FAO and 5
Nation Economic and social council.
more UN designated agencies.
2. Its 2023 report focuses on ageing,
2. The action plan aims to address the
exploring the economic and social
problem of malnutrition in India and the
implications of an ageing human
South-Southeast Asian region.
population.
3. This report has been published for the first Choose the correct code:
time in 2021. (a) 1 only
(b) 2 only
Select the correct code: (c) Both 1 and 2
(a) 1 and 3 (d) Neither 1 nor 2
(b) 2 and 3
(c) Only 2 274. In the light of demand of same sex marriage,
(d) Except 2 and 3 consider the following statements with respect
to Special Marriage act:
271. Which of the following correctly defines” Period
1. Marriage solemnised under the act does
Poverty”?
not cover personal laws.
2. It applies to Indians and Foreigners too.
(a) It is the estimation of wage loss when
3. Both parties along with three witnesses are
women take leave during her
required to be present on the date of
menstruation.
registration.
(b) It is the amount of expenditure on health
because of poor hygiene conditions. Choose the correct code:
(c) It refers to lack of access to menstrual (a) 1 and 2
hygiene products and education. (b) 1 only
(d) None of the above (c) 2 and 3 only
(d) All of these
272. Which of the following is correct with respect
to “Living will concept”? 275. Which of the following statement is correct
with respect to National commission for
1. It is the person’s right to make a directive Protection of child rights?
about the course of treatment and also 1. The commission works under the Ministry
removal of life supporting devices. of Social justice and empowerment.
2. A living will was required to be signed by 2. The commission defines the Child as a
an individual seeking euthanasia in the person who has completed 14 years of age.
presence of two witnesses. 3. The commission carries out the
3. UK and India allows for passive responsibilities mandated under POSCO
euthanasia, thus having the provision for And Juvenile justice care and protection
Living will. act.

Select the correct code: Choose the correct answer:


(a) 1 and 3 (a) 1 and 3
(b) 2 and 3 (b) 1 and 2
(c) 1 and 2 (c) 3 only
(d) All of the above (d) None of these

49
For all current updates on polity and governance, please follow Saurabh Sir's Telegram Channel
"Polity by Saurabh Kumar”

276. Which of the following is correct with respect (c) 2 only


to Pattern of migration according to Census (d) 3 only
2011?
279. Global report on neglected tropical diseases is
1. Movement for work was higher among being released by:
inter-state migrants.
(a) WHO
2. Marriage is commonly quoted as a key
(b) WHO and WORLD BANK
reason for female migration.
(c) WHO and FAO
3. Bihar and Jharkhand are the major states
(d) WHO, FAO and International classification
for migration to the metro cities.
of diseases
Choose the correct code:
(a) 1 and 2 280. Which of the following is the correct
(b) 2 and 3 comparison of Old pension and new Pension
(c) 1 Only system?
(d) None of these
1. In Old pension system, the contribution by
277. Consider the following statements with respect employee was fixed at 10% of basic pay
to National Green Hydrogen Mission: while in NPS, contribution of employee
depends on him but not more than 30% of
1. Strategic Interventions for Green Hydrogen basic pay.
Transition Programme has been launched 2. Amount of pension as per the old system
under this mission to target domestic was linked to the last salary drawn by the
manufacturing of electrolysers. employee, while this aspect is not
2. Strategic hydrogen Innovation partnership considered in the new pension system.
has been launched to facilitate PPP model
research and development. Choose the correct code:
(a) 1 only
Choose the correct code:
(b) 2 only
(a) 1 only
(c) Both 1 and 2
(b) 2 only
(d) Neither 1 nor 2
(c) Both 1 and 2
(d) Neither 1 nor 2
281. Consider the following statements with respect
278. Education Quality Upgradation and Inclusion to Special Category States:
Program (EQUIP) has been recently launched
for higher education. Which of the following 1. Currently 11 states are enjoying this
statements is correct in this regard: status as they share borders with other
countries.
1. It aims to expand the reach of the higher
2. Under this status, the government pays 90
education institutions to increase Gross
percent of the fund to the central schemes.
enrolment ratio to 52% by 2030.
2. The focus is to support 20 prominent
3. These states also enjoy a significant
concession on excise, customs duties,
higher education institutions in India to
income tax, and corporate tax.
get global top 200 rankings.
3. To fund higher education, focus is on
Choose the correct code:
Corporate social responsibility.
(a) 1 and 2 only
Choose the correct code: (b) 2 only
(a) 1 and 3 (c) 2 and 3 only
(b) 2 and 3 (d) All the three

50
For all current updates on polity and governance, please follow Saurabh Sir's Telegram Channel
"Polity by Saurabh Kumar”

282. The idea of reformative criminal justice system 3. Only central universities are eligible to be
is inspired from: part of the scheme.
4. Already the Government has chosen 15
(a) Patanjali Sutra public and private institutes under the
(b) Upanishad scheme.
(c) Smarta tradition
(d) Alamkara shastra How many of the above statements is/are
correct?
283. Which of the following statements with respect
(a)Only one
to MPLADS are correct?
(b)Only two
1. It is a non- lapsable fund. (c)All three
2. Interest on unused funds must be remitted (d)None
to the Consolidated Fund of India.
3. 15% of the fund must be used for the areas 286. The year 2023 has been declared as the
inhabited by Scheduled caste population. International year of Millets by the UN. Which
of the following statements is correct in this
Choose the correct code: regard?
(a) Only One
(b) Only two 1. The proposal for declaration has been
(c) All the three made by India.
(d) None of these 2. The maximum production of Millets is in
Africa followed by Asia.
284. Consider the following statements with respect
3. Recently the Global Millets (Shree Anna)
to Blood Transfusion council:
Conference has been called by India in the
1. It is a policy formulating body under the UNGA Meeting.
Ministry of Health and Family welfare.
2. NBTC is supported by a National Choose the correct code:
Transfusion Services Core Coordination (a) 1 and 3 only
Committee under the chairmanship of (b) 1 and 2 only
Director General of Health Services. (c) 2 and 3 only
(d) 2 only
Choose the correct code:
(a) 1 only 287. Consider the following Statements with
(b) 2 only respect to “Committee against Torture” (CAT)
(c) Both 1 and 2
(d) Neither 1 nor 2 1. It is a body of human rights experts that
monitors implementation of the
285. With respect to the Institute of Eminence
Convention by State parties.
scheme in 2016-17, consider the following
2. The Committee is one of the UN linked
statements:
human rights treaty bodies.
1. Only those institutions will be selected
under the scheme who are in the top 100 Choose the correct code:
in the National Institute of ranking (a) 1 only
framework. (b) 2 only
2. The selected public and private (c) Both 1 and 2
institutions under the scheme will get Rs (d) Neither 1 nor 2
1,000 crore each from the Ministry of
Education.

51
For all current updates on polity and governance, please follow Saurabh Sir's Telegram Channel
"Polity by Saurabh Kumar”

288. Consider the following statements regarding (c) Both 1 and 2


“Data Security Council of India”: (d) Neither 1 nor 2
1. It has been established by CERT in 291. Consider the following statements regarding
collaboration with business Advertising Standards Council of India (ASCI):
conglomerates.
2. It endeavours to increase India’s share in 1. ASCI, established in 1985, a statutory
the global security product and services body under the Ministry of Information
market through global trade development and Broadcasting.
initiatives. 2. ASCI is committed to the cause of self-
regulation in advertising ensuring the
Choose the correct code: protection of the interest of consumers.
(a) 1 only
(b) 2 only Choose the correct code:
(c) Both 1 and 2 (a) 1 only
(d) Neither 1 nor 2 (b) 2 only
(c) Both 1 and 2
289. Consider the following statements with respect (d) Neither 1 nor 2
to “Export Credit Guarantee Corporation of
292. Consider the following statements with respect
India”:
to “National Health System Resource Centre”:
1. It is a non-profit company registered under
1. It is designated as a WHO collaborating
companies Act 2013.
centre for priority medical devices and
2. It functions under the administration of
health Technology policy.
the Ministry of Finance.
2. The Governing body is chaired by the
3. The corporation provides credit insurance
secretary Ministry of Health and family
to banks on account of export credit to
welfare. It was set up under the National
exporters.
Health Mission.
How many of the above statements is/are
Choose the correct code:
correct?
(a) 1 only
a) Only one
(b) 2 only
b) Only two
(c) Both 1 and 2
c) All three
(d) Neither 1 nor 2
d) None
293. Which of the following statements is correct
290. Consider the following statement regarding with respect to Startup?
Higher Education Financing Agency” (HEFA):
1. Startups are defined in the terms of tenure,
1. HEFA is a non-profit, Non-Banking objectives and turnover only.
Financing Company for mobilising extra- 2. An entity is considered as a Startup for the
budgetary resources for building crucial period of 10 years if it is registered as
infrastructure in the higher educational partnership firm and for the period of 5
institutions. years if it is registered as the private
2. The HEFA is jointly promoted by the limited company.
identified Promoter and the Ministry of
Choose the correct code:
Human Resource Development (MHRD).
(a) 1 only
Choose the correct code: (b) 2 only
(a) 1 only (c) Both 1 and 2
(b) 2 only (d) Neither 1 nor 2

52
For all current updates on polity and governance, please follow Saurabh Sir's Telegram Channel
"Polity by Saurabh Kumar”

294. Consider the following statements with respect 297. Consider the following statements with respect
to National institute of agricultural extension to Deen Dayal Antodaya Yojna:
management (MANAGE): -
1. It has been launched as a restructured
1. It is an autonomous organisation of the version of Swarn Jayanti Swarojgar Yojna.
Ministry of agriculture and farmers 2. The scheme has support from the World
welfare. Bank.
2. Its mission is to provide service to 3. It has been launched under the Ministry of
fishermen for sustainable agriculture. Rural Development.
3. Feed the Future India triangular training 4. It focuses on Employment through Skill
program was started by MANAGE in 2010. Training and Placement.

Choose the correct code: Choose the Correct code:


(a) 1 and 2 only (a) 1, 2 and 3 only
(b) 2 and 3 only (b) 2, 3 and 4 only
(c) 1 and 3 only (c) 1, 3 and 4 only
(d) 1 only (d) All of these

295. Which of the following is correctly matched as 298. Twenty Point Program launched by the
per the One district one product scheme? Government in 1975 with the following
objective:
1. Coconut based products: - Kerala
2. Spices: - Andhra Pradesh (a) To improve the living condition of Tribals.
3. Sea buckthorn: - Ladakh (b) To improve living condition of women
4. Bakery: - Chandigarh (c) To end poverty in rural areas.
(d) To achieve robust infrastructure in border
Select the correct pair(s): areas.
(a) Only one pair
(b) Only two pair 299. Which of the following statements with respect
(c) Only three pair to Internet and mobile association of India is
(d) All the pairs correct: -

296. Consider the following statements with respect 1. It is a company registered under
to Mahila Samriddhi yojna: companies act 2013.
2. It is a joint collaboration of the government
1. It was started in 1993, to offer loans to the and industry association.
women SHG group. 3. It is the only professional body
2. The target group under the yojna is OBC, representing the online and offline mobile
SC, Minorities and people with disabilities. Value-Added Service industry in India.
3. The applicant's annual family income
should not be more than 3 lakhs. Choose the Correct Code:
(a) 1 and 3 only
Choose the correct code: (b) 2 and 3 only
(a) 1 and 3 only (c) 3 only
(b) 2 and 3 only (d) 2 only
(c) 1 and 2 only
(d) All the three

53
For all current updates on polity and governance, please follow Saurabh Sir's Telegram Channel
"Polity by Saurabh Kumar”

300. Which of the following is Correct with respect


to AFSPA?

1. The Governor of the State Can declare any


area as Disturbed Area under AFSPA.
2. It Gives Power to the Army, State Police
Force and Central Police Forces to shoot to
kill.
3. AFSPA has been declared in the entire
State of Nagaland.

Choose the correct code:


(a) 1 and 3 only
(b) 2 and 3 only
(c) 1 and 2 only
(d) All of these

54
For all current updates on polity and governance, please follow Saurabh Sir's Telegram Channel
"Polity by Saurabh Kumar”

EXPLANATION
1. (b)
Explanation
• A democratic polity can be classified into two categories—monarchy and republic. In a monarchy,
the head of the state (usually king or queen) enjoys a hereditary position, that is, he comes into
office through succession, e.g., Britain. In a republic, on the other hand, the head of the state is
always elected directly or indirectly for a fixed period, e.g., USA. Therefore, the term 'republic' in
our Preamble indicates that India has an elected head called the president. He is elected indirectly
for a fixed period of five years.
• The word Republic means two more things: All public Offices is open for everyone without any
discrimination and sovereignty rests with the people.

2. (c)
Explanation
• The limited government is the one which enjoys only a limited power. It does not have unfettered
powers like an authoritarian government. It cannot choose to act according to its whims and
fancies. Its power is bounded by the checks and balances accorded in the constitution of that
country. Thus, option c is the correct answer.

3. (c)
Explanation
• Running government in a democracy is a game of numbers. Whichever party or coalition garners
the maximum number of seats in the parliament, forms the government. However, the member of
parliaments or any party in the coalition may switch their support to any other coalition. This may
lead to the fall of the government without being able to complete its term. This brings instability
in governance.
• It is difficult to draw a mechanism to take the opinion of all the citizens, where the population is
sufficiently big. It may become an expensive affair to take public opinion on every (big or small)
issue. Therefore, it is suitable only for smaller countries and for bigger decision.

4. (b)
Explanation
• Statement 1 and 3 are correct for the Presidential form of government. However, statement 2 is
correct for parliamentary form of government.

5. (b)
Explanation
• In India, the members of Lok Sabha are elected directly, these members then elect a member from
among themselves as a Prime minister. Similarly, the President is also elected by members of
Parliament and State Assemblies.
• The Prime minister can be a member of any of the two houses of the Parliament.
• The Council of ministers is responsible to the Lok Sabha in particular and Parliament in general.
Thus statement 3 is Incorrect.

1
For all current updates on polity and governance, please follow Saurabh Sir's Telegram Channel
"Polity by Saurabh Kumar”

6. (b)
Explanation
• Indian secularism allows everyone to follow any religion of his/her choice. It does not put any bar
to follow any particular religion. One can even have no religion. This is given under art. 25.
• Art. 26, allows every religious group to freely manage its religious affairs.
• Indian secularism follows the principle of equidistance with all the religion. It has no religion of
its own nor does it favour any particular religion.

7. (c)
Explanation
• According to Dr. B. R. Ambedkar, the right to constitutional remedies under article 32 is the heart
and soul of the Indian constitution.

8. (b)
Explanation
• Federal form of government has the following advantages:
1. Helps accommodate diversity.
2. Promotes unity in the country.
3. Representation to different sections, etc.
• However, it does not guarantee cooperation between federal and its constituent governments.
There may or may not exist cooperation.

9. (b)
Explanation
• The Election Commission of India has laid down the detailed criteria for a political party to be a
recognised party.
• At present 6 political parties - BJP, Congress, CPI(M), BSP, National People’s Party (NPP) and the
AAP - are the national parties recognised by the Election Commission of India. Recently the
Trinamool Congress (TMC), Nationalist Congress Party (NCP), and Communist Party of India (CPI)
have lost their national party status.

10. (a)
Explanation
• A party that secures at least six percent of the total votes in the Lok Sabha elections or Assembly
elections in four states and wins at least four seats in the Lok Sabha is recognised as a national
party.

11. (b)
Explanation
• Only the details of all the criminal cases pending against the candidate, is required to be furnished
before the election commission.
• Other details: Assets and liabilities of the candidate and his/her family. Education qualification
of the candidate.

12. (d)
Explanation
• Community government: Form of govt. in which power is shared by different social groups.

2
For all current updates on polity and governance, please follow Saurabh Sir's Telegram Channel
"Polity by Saurabh Kumar”

• Separation of powers: Administrative arrangement where power in the govt. is shared among
different organs of the government.
• Coalition government: Form of govt. in which power is shared by two or more political parties.
• Federal government: Form of govt. in which power is shared among governments at different levels.

13. (d)
Explanation
• Education, Forest and Administration of judgement. These subjects belong to the concurrent list.
• Cryptocurrency and Cyber Security are residuary subjects. i.e. they do not belong to any of the
three lists.

14. (c)
Explanation
• A bill that seeks to amend a provision of the constitution of India, which relates to the sharing of
power between the states and the union must be passed in both the houses of parliament
separately, with the majority of at least half of the total membership of the house or the majority
of at least two-third of the members present and voting.
• Further, the bill must be ratified by the legislatures of at least half of the total states, with the
majority of at least one-half of the members present and voting.

15. (c)
Explanation
• Essential features of federalism: -
➢ Written constitution. Independent judiciary
➢ Division of power among the constituent units of the government (but not necessarily equally).
➢ Rigid constitution. etc.

16. (c)
Explanation
• Scheduled Languages of India are the list of 22 languages written in the eight schedules of the
constitution of India. Thus, statement 1 is correct.
• States may have their own official languages but there is no provision for them to have their own
list of schedule languages. Thus, statement 2 is incorrect.
• A candidate in an examination conducted for a central government position may opt to take
examination in any of these languages. Thus, statement 3 is correct.

17. (c)
Explanation
• Payment of minimum wages is a part of DPSP, not fundamental rights. Workers going on strike is
not a fundamental right.
• Banning of a movie is a violation of a right to freedom of speech and expression under art. 19
(1)(a). It is a part of fundamental rights.

3
For all current updates on polity and governance, please follow Saurabh Sir's Telegram Channel
"Polity by Saurabh Kumar”

18. (b)
Explanation
• In First Past the Post System: -
• The entire country is divided into small geographical units called constituencies. Every
constituency elects only one representative. A party may get more seats than votes polled for it.
Candidate who wins may not get majority votes.

19. (a)
Explanation
• Union List: Only the central government can make laws.
• State List: State govts. can make laws but as per Article 249, the Parliament can make laws on
items in the State List if the Rajya Sabha passes a resolution by ⅔ majority of its members present
and voting.
• Concurrent List: Both the central as well as the state govts. can make laws.
• Residuary List: Only the central govt. can make laws.

20. (c)
Explanation
• Ministers make a draft for law and a part of Parliament and are also responsible for the
implementation. Thus Statement 1 is Correct.
• Ministers take Oath of Secrecy before the President and Members take Oath before Pro Term
Speaker. Thus Statement 2 is Incorrect.
• Third Statement is Self-Explanatory. This is Correct.

21. (a)
Explanation
• Supremacy means the Laws made by Parliament must conform to the Principles of Constitution.
Thus Statement 1 is Correct.
• Statement B and C is Correct individually but not Correct as per the demand of the question.

22. (d)
Explanation
• Statement 1: -Power is shared as per Doctrine of Separation of power, to keep a check on different
organs of Government.
• Statement 2: - Power is shared between Centre and State as per List in 7th Schedule of the
Constitution.
• Statement 3: - Power is shared between different linguistic groups like in Belgium and in between
Different Religious Groups like in Sri Lanka.

23. (c)
Explanation
Self-Explanatory

24. (c)
Explanation
• Constitutions exist in all types of countries, whether democratic or authoritarian.

4
For all current updates on polity and governance, please follow Saurabh Sir's Telegram Channel
"Polity by Saurabh Kumar”

25. (d)
Explanation
• Both the Statements seem to be true, but the Logic is Philosophy of the Constitution is so intact
and strong that both the statements cannot jeopardize it.

26. (a)
Explanation
• Liberal ideas are based on individualism, Communitarians places society above the individual.

27. (a)
Explanation
• The constitution can be amended, with the safeguards like Special majority, effective majority
depending on the case.
• Equal representation of all the state is undoubtedly a feature of Federal polity, bit not a necessary
condition to be satisfied.
• There is a clear-cut separation of power between centre and state, barring some exception.

28. (b)
Explanation
• Constituent power means amending power of the Parliament, not the law-making power. Thus
Statements 1 and 3 are Correct.

29. (b)
Explanation
• Legislative and executive relation has been divided between centre and State as per 7th Schedule
in Article 245 to Article 263. Thus statement 2 and 4 is Correct.

30. (c)
Explanation
Self-explanatory

31. (b)
Explanation
• All of the Statement Comes under the definition of Justice, but with some limitations. Statement
2 is without any limitation and thus the Correct Definition of Justice.

32. (c)
Explanation
• Statement1 is Correct as persuading does not mean Forcing to Covert someone. Statement 2 is
Correct as has been explained by SC.

33. (b)
Explanation
• If Rights will not be recognized by Society, it will Create Chaos. The word should have been used
in the Statement 1. This is Correct.
• Statement 2 is Self-explanatory. This is Correct.
• Statement 3 is just the analysis of Rights, thus not appropriate as per the demand of the Question.

5
For all current updates on polity and governance, please follow Saurabh Sir's Telegram Channel
"Polity by Saurabh Kumar”

34. (a)
Explanation
• The word Freedom is always understood as absence of constraints, but it is a unidimensional way
of understanding Freedom as it can create anarchy in the State or society. Thus Statement 1 is
Incorrect
• Society places restriction on Freedom and also put Sanctions on it. Thus Statement 2 is Correct
in itself, but not according to the question asked.
• Freedom is always about expanding the person’s scope so that he/ she can develop himself or
herself completely.

35. (c)
Explanation
Self-Explanatory

36. (a)
Explanation
• Undoubtedly Right to life and liberty has widened the scope of Rights, but it is not the right from
where all the other rights are derived.
• In second statement the word “Must” makes the statement wrong.

37. (b)
Explanation
• Equality of opportunity is a three-way relationship between a person, some obstacles, and a
desired goal. However, a person only has an opportunity if she has a chance of achieving that goal.
One cannot have an opportunity if one faces insurmountable obstacles that make it impossible to
secure the goal.

38. (b)
Explanation
• Transformative constitutionalism entails instilling principles such as equality, liberty, fraternity,
and dignity into society. It entails achieving the Constitution's primary goal of transforming society
for the better. It tries to give paramount significance to Constitutional morality rather than what
constitutes morality in society.

39. (c)
Explanation
• The doctrine of Constitutional morality traces its origin back to the work of English Historian
George Grote. He described the phrase in the context of the history of Greece, Athenian democracy
and the perils surrounding it.
• He used ‘constitutional morality’ to describe popular sovereignty, governed based on ‘freedom’ and
self-restraint. Constitutional morality, for Grote also meant citizens’ right to criticise public
officials. Therefore, highlighting the limitation to the power of public officials and their duty to
respect the Constitution.
• Today, the meaning and connotation of the phrase have acquired newer interpretations. But
essentially, constitutional morality is a sentiment among the common masses necessary for
establishing a peaceful and stable government. It is supposed to be a perfect balance between
freedom and restrictions to those freedoms.

6
For all current updates on polity and governance, please follow Saurabh Sir's Telegram Channel
"Polity by Saurabh Kumar”

• In the Indian context, this word was first used by Dr Bhim Rao Ambedkar during the Constituent
Assembly Debate to justify the inclusion of administrative details in the Constitution itself.
• Two judges in Kesavananda Bharati v. State of Kerala judgement had invoked constitutional
morality but didn’t go through with it. In the SP Gupta Case (also known as the First Judge
Case), another judge described constitutional violation as ‘a serious breach of constitutional
morality’. It was later also used by Justice S.B Sinha in Islamic Academy of Education v. State of
Karnataka.

40. (b)
Explanation
• The cabinet collective responsibility has two main elements:
1. Cabinet Confidentiality: It means that all the decisions taken, or the deliberations discussed
in the cabinet meeting must not be revealed in the public and should remain with the council
of ministers.
2. Cabinet Solidarity: It means that the ministers in the council should show unanimity in their
decision even if they might have expressed a different view in the meeting of the cabinet.
• In India, the principle of collective responsibility applies to all the members of the government
from the ministers of cabinet to the minister of states.
• In Common Cause v Union of India Supreme Court held and observed that the principle of
collective responsibility has two meanings. Firstly, that all the members of a government are
unanimous in support of policy. Secondly, the ministers are responsible for the success and
failures of the policies.

41. (b)

42. (c)
Explanation
• Statement 1 is Correct.
• The Law passed by Provisional Parliament has the Legal force, not the Constitutional Provision.
Thus Statement 2 is Incorrect.
• The constituent assembly ratified India’s membership of the commonwealth of nation in May
1949.
• It also adopted National Flag on 22 July 1947.

43. (c)
Explanation
• The provision in the constituent assembly passed without any debate is universal adult Suffrage.

44. (c)
Explanation
Main Features of Objective Resolution: -
• India is an independent, sovereign, republic.
• India shall be a Union of erstwhile British Indian territories, Indian States, and other parts outside
British India and Indian States as are willing to be a part of the Union Territories forming the
Union shall be autonomous units and exercise all powers and functions of the Government and
administration.

7
For all current updates on polity and governance, please follow Saurabh Sir's Telegram Channel
"Polity by Saurabh Kumar”

• All powers and authority of sovereign and independent India and its constitution shall flow from
the people.
• All people of India shall be guaranteed and secured social, economic and political justice, equality
of status and opportunities and equality before law.
• The minorities, backward and tribal areas, depressed and other backward classes shall be
provided adequate safeguards.
• The territorial integrity of the Republic and its sovereign rights on land, sea and air shall be
maintained according to justice and law of civilized.

45. (a)
Explanation
• The members of the Constituent Assembly were not elected on the basis of universal franchise. In
the winter of 1945-46 provincial elections were held in India. The Provincial Legislatures then
chose the representatives to the Constituent Assembly.
• The representatives of the princely states were to be nominated by the heads of the princely states.
• The Assembly comprised representatives of all sections of the Indian society– Hindus, Muslims,
Sikhs, Parsis, Anglo-Indians, Indian Christians, SCs, STs including women of all these sections.

46. (a)
Explanation
• The Constituent Assembly appointed a number of committees to deal with different tasks of
constitution-making. Out of these, eight were major committees and the others were minor
committees.

• Major Committees
1. Union Powers Committee - Jawaharlal Nehru
2. Union Constitution Committee -Jawaharlal Nehru
3. Provincial Constitution Committee -Sardar Patel
4. Drafting Committee - Dr. B.R. Ambedkar
5. Advisory Committee on Fundamental Rights, Minorities and Tribal and Excluded Areas -
Sardar Patel.
6. Rules of Procedure Committee - Dr. Rajendra Prasad
7. States Committee (Committee for Negotiating with States) - Jawaharlal Nehru
8. Steering Committee - Dr. Rajendra Prasad

• Minor Committees
1. Finance and Staff Committee - Dr. Rajendra Prasad
2. House Committee - B. Pattabhi Sitaramayya
3. Order of Business Committee - Dr. K.M. Munshi
4. Ad-hoc Committee on Citizenship - S. Varadachari (Not an Assembly Member)

47. (c)
Explanation
• Statement 1 is Correct- Fundamental Corrects and DPSP are the reflection of ideals mentioned in
Preamble. SC uses these ideals while interpreting FR and DPSP.
• Statement 2 is Correct- The statement is correct, but SC ignores this fact in Berubari Union case,
but later recognized in Keshvananda Bharati case.

8
For all current updates on polity and governance, please follow Saurabh Sir's Telegram Channel
"Polity by Saurabh Kumar”

48. (a)
Explanation
• All statements are correct except statement 2.
• You may get confused with the second statement. The Word (Any Industry) is open to different
interpretations and thus depends on case-to-case basis by SC. But here any reflect both Public
and Private Industry thus Incorrect.

49. (c)
Explanation
• The Preamble of the Constitution contains the philosophy on which the entire Constitution has
been built. It provides a standard to examine and evaluate any law and action of the government,
to find out whether it is good or bad. It is the soul of the Indian Constitution.
• The Preamble has been amended only once so far, in 1976, by the 42nd Constitutional
Amendment Act, which has added three new words–Socialist, Secular and Integrity–to the
Preamble.
• Like any other part of the Constitution, the Preamble was also enacted by the Constituent
Assembly; but, after the rest of the Constitution was already enacted. The reason for inserting the
Preamble at the end was to ensure that it was in conformity with the Constitution as adopted by
the Constituent Assembly.

50. (b)
Explanation
• The Preamble is neither a source of power to legislature nor a prohibition upon the powers of
legislature. It is non-justiciable, that is, its provisions are not enforceable in courts of law.
• The date is mentioned in the preamble when it was adopted i.e. November 26, 1949.
• In the Berubari Case, the Court said that Preamble is the key to open the mind of the makers’,
but it cannot be considered as part of the Constitution. Therefore, it is not enforceable in a court
of law, but in Kesavananda Bharati Case, the court said the Preamble of the Constitution is part
of the Constitution. In this case SC said the Preamble is not the supreme power or source of any
restriction or prohibition but it plays an important role in the interpretation of statutes and
provisions of the Constitution.

51. (a)
Explanation
• Preamble of the Constitution
We, THE PEOPLE OF INDIA, having solemnly resolved to constitute India into a SOVEREIGN
SOCIALIST SECULAR DEMOCRATIC REPUBLIC and to secure to all its citizens:

JUSTICE, Social, Economic and Political.

LIBERTY of thought, expression, belief, faith and worship.

EQUALITY of status and of opportunity; and to promote among them all.

9
For all current updates on polity and governance, please follow Saurabh Sir's Telegram Channel
"Polity by Saurabh Kumar”

FRATERNITY assuring the dignity of the individual and the unity and integrity of the Nation.
IN OUR CONSTITUENT ASSEMBLY this twenty-sixth day of November 1949, do HEREBY ADOPT,
ENACT AND GIVE TO OURSELVES THIS CONSTITUTION”.
52. (d)
Explanation
• Statement 1 is Correct- Indian Federation is based on ideas of Union of States rather than
Federation of States. Thus, there is no agreement and secondly Article 3 provided power to
Parliament to reduce, include any area in the state.
• Statement 2 is Correct-Clause 3 of Article 3 gives power to diminish the area of any state. It can
be done by taking away any part or even cut away the entire area of the state.
53. (a)
Explanation
• A bill on a new state has to be recommended by the President. In India it is usually the Cabinet
which requests the President to do that. Article 3 makes it clear that the Parliament is the sole
authority on making a decision on a new state.
• The President refers the bill to the State Assembly for its views giving it a certain period of time.
Parliament is not obligated to follow the views of the State Assembly. If the State Assembly does
not express its opinion within the specified period of time, the bill could be introduced in the
Parliament after the expiry of the specified period.

54. (d)
Explanation
• The Indian National Congress established a three-member committee at the Jaipur session,
consisting of Nehru, Patel, and Pattabhi Sitaramayya. The committee was set up to examine the
Dhar Commission report and make recommendations to the government since it caused
widespread dissatisfaction, prompting Congress to appoint another Linguistic Provinces
Committee in December 1948.
• JVP Committee’s Report did not advocate for large-scale linguistic reorganisation, it did make
special mention of Andhra. It recommended its creation but on the condition that its protagonists
abandon their claims to the city of Madras. However, in October 1953, the Government of India
was forced to establish the first linguistic state, known as Andhra state, by separating Telugu-
speaking areas from Madras state.

55. (d)
Explanation
• The Dadra and Nagar Haveli and Daman and Diu (Merger of Union Territories) ACT, 2019 amends
the First Schedule to merge the territories of the two UTs: (a) Dadra and Nagar Haveli, and (b)
Daman and Diu. The merged territory will form the UT of Dadra and Nagar Haveli and Daman
and Diu
• 36th Amendment Act amended the First Schedule of the Constitution of India to include Sikkim
as a state of India. This act abolished the office of the Chogyal who was the hereditary ruler of
Sikkim and the Chogyal was replaced by a Governor appointed by the President of India. The act
provided for the establishment of a legislative assembly in Sikkim. The amendment proposes to
allot Sikkim one seat in the Council of States and one seat in the House of the People.
• Article 2 of the Indian Constitution provides the parliament with the power of enacting a law in
order to admit into the union or establish new States ‘on such terms and conditions as it thinks
fit. The phrase, ‘on such terms and conditions as it thinks fit,’ introduces the element of

10
For all current updates on polity and governance, please follow Saurabh Sir's Telegram Channel
"Polity by Saurabh Kumar”

parliamentary discretion into the provision. However, in the case of Mangal Singh v. Union of
India, the court adjudged that this power of parliament cannot override the constitutional scheme
and such terms and conditions should be in consonance with the basic features of the
constitution.
56. (d)
Explanation
• According to article 4, any law referred to in article 2 or article 3 shall contain such provisions for
the amendment of the First Schedule and the Fourth Schedule.
• The Representation of People act 1951 may change because the number of seats in Lok Sabha will
also change after the formation of a new state.
• Article 240 will see incidental change in any change with respect to Union Territory only, not in
case of State.

57. (d)
Explanation
• Article 394 says that Article 5,6,7,8,9,324,367,379,394 came into force on 26th November 1949
and rest on 26th January 1950.

58. (b)
Explanation
• One can be a citizen of India by the Process of Registration, Naturalization also. Thus Statement
1 is Correct.
• On the basis of L.M Singhvi Committee, Person of Indian origin has been provided with OCI Card
that was introduced by The Citizenship (Amendment) Act, 2005 in August 2005. Thus statement
2 is incorrect.
• There is not any National Identity card, National People register is issued to every citizen according
to Citizenship Act, 1955, and the Citizenship (Registration of Citizens and Issue of National
Identity Cards) Rules, 2003. Thus Statement 3 is Incorrect.

59. (b)
Explanation
• The idea of Citizenship came into existence on 26 Jan 1949. Thus Statement 2 is Incorrect.

11
For all current updates on polity and governance, please follow Saurabh Sir's Telegram Channel
"Polity by Saurabh Kumar”

60. (d)
Explanation

61. (c)
Explanation
• It is compulsory for every citizen of the country to register in a National Register of Indian Citizens
(NRIC) as per Section 14A of the Citizenship Act 1955 as amended in 2004.

12
For all current updates on polity and governance, please follow Saurabh Sir's Telegram Channel
"Polity by Saurabh Kumar”

• By definition, an NRI is not a usual resident of the country. Therefore, they would not be in the
NPR - 2010 till they are non-residents. When they come back to India and take up usual residence
within the country, they will be included in the NPR - 2010. Providing any false information would
attract penalties under Citizenship Rules 2003. A proposal to issue Resident Identity Cards to all
usual residents in the NPR - 2010 of 18 years of age and above is under consideration of the
Government. This Card would be a smart Card and would bear the Aadhaar number.
• NPR information will be self-attested, that is, whatever information is provided by the respondent
will be deemed correct and no documents or biometric would be required, thus no need to produce
any document.

62. (c)
Explanation
• In India both a citizen by birth as well as a naturalised citizen are eligible for the office of
President while in the USA, only a citizen by birth and not a naturalised citizen is eligible for the
office of President.
• The Constitution deals with the citizenship from Articles 5 to 11 under Part II. It only identifies
the persons who became citizens of India at its commencement (i.e., on January 26, 1950). It
does not deal with the problem of acquisition or loss of citizenship subsequent to its
commencement. It empowers the Parliament to enact a law to provide for such matters and
any other matter relating to citizenship. Accordingly, the Parliament has enacted the
Citizenship Act (1955), which has been amended from time to time.
63. (b)
Explanation
• The Constitution does not define the term ‘citizen’ but details of various categories of persons
who are entitled to citizenship are given in Part 2 (Articles 5 to 11).
• The constitution of India provides for the citizenship of those individuals who migrated from
Pakistan at the time of partition under Article 6.
• The Indian Constitution provides for only a single citizenship. There is no separate state
citizenship. There is only one domicile allowed in India. Domicile Certificate can be made only
in one State/UT.

64. (c)
Explanation
• The Citizenship Act (1955) prescribes three ways of losing citizenship whether acquired under the
Act or prior to it under the Constitution, viz, renunciation, termination and deprivation:
➢ By Renunciation: Any citizen of India of full age and capacity can make a declaration
renouncing his Indian citizenship. However, if such a declaration is made during a war in
which India is engaged, its registration shall be withheld by the Central Government. Further,
when a person renounces his Indian citizenship, every minor child of that person also loses
Indian citizenship. However, when such a child attains the age of eighteen, he may resume
Indian citizenship.
➢ By Termination: When an Indian citizen voluntarily (consciously, knowingly and without
duress, undue influence or compulsion) acquires the citizenship of another country, his Indian
citizenship automatically terminates. This provision, however, does not apply during a war in
which India is engaged.
➢ By Deprivation: It is a compulsory termination of Indian citizenship by the Central government,
if (a) the citizen has obtained the citizenship by fraud (b) the citizen has shown disloyalty to
the Constitution of India (c) the citizen has unlawfully traded or communicated with the enemy

13
For all current updates on polity and governance, please follow Saurabh Sir's Telegram Channel
"Polity by Saurabh Kumar”

during a war (d) the citizen has, within five years after registration or naturalisation, been
imprisoned in any country for two years and (e) the citizen has been ordinarily resident out of
India for seven years continuously.

65. (d)
Explanation
• Right to equality does not mean treating Unequal’s as equals. Right to equality is treating equals
as equal and Unequal’s as unequal. It does not intend to remove all sorts of distinctions in society;
however, it aims at empowering everyone to have equality of opportunity to achieve whatever one
desires for or capable of. Hence, d is the correct answer.

66. (d)
Explanation
• The Right against exploitation is given under art. 23 and 24 of the Indian constitution. Art. 23:
Prohibits forced labour and illegal trafficking. Art. 24: Prohibits child labour. Thus, d is the correct
answer.

67. (d)
Explanation
• Article 19.1(d) and Article 19.1(e) of constitution of India, is a part of Fundamental rights.
According to Article 19.1(d): All citizens have the right to move freely throughout the territory of
India. Article 19.1(e): All citizens the right to reside and settle in any part of the territory of India.
• Owning a land is not a fundamental right but a legal right.
• Art. (20.3) of the constitution of India, provides that no person accused of any offence shall be
compelled to be a witness against himself. However, the person may be asked to provide his/her
biometric details, which is an exception for Art. (20.3).

68. (a)
Explanation
• Mandamus: This writ is issued when the court finds that a particular office holder is not doing
legal duty and thereby infringing on the right of an individual.
• Certiorari: When a court has to order a lower court to transfer a matter pending before it.
• Prohibition: Issued by a higher court, when a lower court has considered a case going beyond its
jurisdiction.

69. (c)
Explanation
• Clause (1A) of the Article 30 - introduced by the Constitution 44th Amendment Act 1978 - relates
to the compulsory acquisition of any property of a minority educational institution.
• The compensation amount fixed by the State for the compulsory acquisition of any property of a
minority educational institution shall not restrict or abrogate the correct guaranteed to them.

70. (c)
Explanation

14
For all current updates on polity and governance, please follow Saurabh Sir's Telegram Channel
"Polity by Saurabh Kumar”

• Statement 3 and 4 is a part of Right to Equality, not the part of Right against Exploitation.

71. (c)
Explanation
• All the Statements are Self Explanatory.

72. (d)
Explanation
• The Court said that Reservation is not a Fundamental Right while refusing to act on a petition
filed by all political parties from Tamil Nadu who sought 50% OBC reservation in the all-India
NEET seats surrendered by states.
• All political parties from Tamil Nadu filed a writ petition under Article 32 of the Constitution.
• They accused the Centre of violating the “right of the people of Tamil Nadu to have a fair education”
by not implementing the 50% quota for Backward Classes and Most Backward Classes for the All-
India Quota seats in medical and dental science courses.
• Only Parliament under Article 16 (3) can provide for reservation on the basis of residence.

73. (b)
Explanation
• Preamble is the mirror of all the Principles contained in Fundamental Rights and DPSP. Thus
Statement 1 is Correct but Incorrect as per the demand of the Question. This Statement is not the
Objective of Fundamental Rights.
• Statement 2 is Correct and fulfil the objectives of Fundamental Rights is self – Explanatory.
• This Statement is the consequence of any action on Fundamental Rights. Thus Statement 3 is
also Incorrect.

74. (d)
Explanation
• Fundamental Corrects prevent the establishment of an authoritarian and despotic rule in the
country and protect the liberties and freedoms of the people against the invasion by the State. Some
FR are also available against private entities.
• Under Indian Constitution, All the Fundamental Rights are available against the ‘State' but only
5 fundamental Rights are available against both State as well as against Private Individual. These
Fundamental Rights are:
➢ Article 15(2) - Provides that No citizen shall be subject to any kind of discrimination on the
basis of his race, religion, place of birth or caste etc. It is available against every individual it
means, if anyone does any kind of discrimination on the basis on any of the above-mentioned
ground, then he shall be liable for punishment.
➢ Article 17 - Talks about abolition of Untouchability. It devises that anyone practicing
Untouchability shall be punished.
➢ Article 23 - Prohibits trafficking of humans and forced labour.
➢ Article 24 - Prohibits employment of children in factories and hazardous place
➢ Article 21 A
➢ Article 21 as said by 5 judge bench in January 2023.

15
For all current updates on polity and governance, please follow Saurabh Sir's Telegram Channel
"Polity by Saurabh Kumar”

75. (c)
Explanation
• It is because they are available against state and private individuals both. And Article 16 is
available against the State only.

76. (d)
Explanation
• All are the part of Freedom of speech and Expression. Other constituents of the Freedom under
Article 19 is defined as:
1. Correct to know criminal antecedents of candidates in election.
2. Correct to answer criticism of one’s views.
3. Correct against Sound Pollution (Moulana Mufti Sayed vs State of west Bengal 2014)

77. (b)
Explanation
• Statement 1 is Incorrect: It is a sine qua non of Article 21

78. (c)
Explanation
Self-Explanatory

79. (d)
Explanation
Article 22 (4) to 22(7) provides for:
• No detention beyond three months until such detention is approved by the Advisory committee.
• The detaining authority must communicate the detenu, grounds for detention, so that detenu can
make representation.
• No detention beyond the maximum period prescribed under the law made by the Parliament.

80. (d)
Explanation
• All the Statements are Correct.
• Other differences are: - Article 26 says correct of Religious denomination to maintain the
institution established by them, Article 30 (1) secures correct to minority only. Article 26 (a) is
subjected to Public order, Morality and health while Article 30 is not subjected to such restriction.

81. (d)
Explanation
• All the statements are correct. Correct to establish and administer was in the news recently
which is important to know. SC has said that the correct under Article 30 to choose a teacher is
not absolute. There is a difference between secular education and education regarding culture
and customs of community.

82. (a)
Explanation

16
For all current updates on polity and governance, please follow Saurabh Sir's Telegram Channel
"Polity by Saurabh Kumar”

• Both Parliament and State Legislature can make law on matters related to Security of the state,
supply of essential services and maintenance of Public order. However Only Parliament can enact
law in matters related to Defence, Foreign affairs and security of India.

83. (a)
Explanation
• Proclamation of emergency is done under Article 351 and the President suspends correct to move
to the court under Article 359.

84. (a)
Explanation
• In Kanu Sanyal vs DM Darjeeling 1973, it has been held by the court that a lawyer, Friend and
relative can file for writ petition and there is no need to produce the body of the detenu before the
court.
• Mandamus cannot be issued to enforce departmental instruction that does not possess statutory
force.
• The writ of habeas corpus, also known as the 'Great Writ of Liberty', has its roots in the Magna
Carta of 1215

85. (b)
Explanation
• Fundamental rights can be amended if it goes against the Basic structure of the constitution. They
allow concentration of power in the case of suspension of Fundamental rights during martial law.
86. (c)
Explanation
• Following are the Fundamental rights which are available to both citizens of India and foreign
nationals except enemy aliens:
➢ Equality before law and equal protection of laws (Article 14)
➢ Right to elementary education (Article 21A).
➢ Freedom to manage religious affairs (Article 26).
➢ Protection in respect of conviction for offences (Article 20).
➢ Protection of life and personal liberty (Article 21).
➢ Protection against arrest and detention in certain cases (Article 22).

• Rights available only to citizens and not to foreigners are as following:


➢ Prohibition of discrimination on grounds of religion, race, caste, sex or place of birth (Article
15).
➢ Equality of opportunity in matters of public employment (Article 16).
➢ Protection of six rights regarding freedom of: (i) speech and expression, (ii) assembly, (iii)
association, (iv) movement, (v) residence, and (vi) profession (Article 19).
➢ Right of minorities to establish and administer educational institutions (Article 30).

87. (a)
Explanation
• Some of them are available only to the citizens while others are available to all persons whether
citizens, foreigners or legal persons like corporations or companies.

17
For all current updates on polity and governance, please follow Saurabh Sir's Telegram Channel
"Polity by Saurabh Kumar”

• They are not absolute but qualified. The state can impose reasonable restrictions on them.
However, whether such restrictions are reasonable or not is to be decided by the courts. Thus,
they strike a balance between the rights of the individual and those of the society as a whole,
between individual liberty and social control.
• All of them are available against the arbitrary action of the state. However, some of them are also
available against the action of private individuals. Example Article 15 (2), 17 23 etc.
• They are defended and guaranteed by the Supreme Court. Hence, the aggrieved person can directly
go to the Supreme Court, not necessarily by way of appeal against the judgement of the high
courts.
• They are not sacrosanct or permanent. The Parliament can curtail or repeal them but only by a
constitutional amendment act and not by an ordinary act. Moreover, this can be done without
affecting the ‘basic structure’ of the Constitution.

88. (b)
Explanation
• Freedom of Speech and Expression It implies that every citizen has the right to express his views,
opinions, belief and convictions freely by word of mouth, writing, printing, picturing or in any
other manner. The Supreme Court held that the freedom of speech and expression includes the
following:
(a) Right to propagate one’s views as well as views of others.
(b) Freedom of the press.
(c) Freedom of commercial advertisements.
(d) Right against tapping of telephonic conversation.
(e) Right to telecast, that is, government has no monopoly on electronic media.
(f) Right against bundh called by a political party or organisation.
(g) Right to know about government activities/Right to Information
(h) Freedom of silence.
(i) Right to answer the criticism (Right to reply)
(j) Right against imposition of pre-censorship on a newspaper.
(k) Right to demonstration or picketing but not right to strike.

Note: Right against handcuffing comes under Article 21 of the Indian Constitution.

89. (a)
Explanation
• Article 20 grants protection against arbitrary and excessive punishment to an accused person,
whether citizen or foreigner or legal person like a company or a corporation. It contains three
provisions in that direction:
➢ No ex-post-facto law i.e., No person shall be (i) convicted of any offence except for violation of
a law in force at the time of the commission of the act, nor (ii) subjected to a penalty greater
than that prescribed by the law in force at the time of the commission of the act.
➢ An ex-post-facto law is one that imposes penalties retrospectively (retroactively), that is, upon
acts already done or which increases the penalties for such acts. However, this limitation is
imposed only on criminal laws and not on civil laws or tax laws. In other words, a civil liability
or a tax can be imposed retrospectively.
➢ Finally, the protection (immunity) under this provision cannot be claimed in case of preventive
detention or demanding security from a person.

18
For all current updates on polity and governance, please follow Saurabh Sir's Telegram Channel
"Polity by Saurabh Kumar”

➢ The protection against double jeopardy is available only in proceedings before a court of law
or a judicial tribunal. In other words, it is not available in proceedings before departmental or
administrative authorities as they are not of judicial nature.
➢ The protection against self-incrimination extends to both oral evidence and documentary
evidence. However, it does not extend to (i) compulsory production of material objects, (ii)
compulsion to give thumb impression, specimen signature, blood specimens, and (iii)
compulsory exhibition of the body. Further, it extends only to criminal proceedings and not to
civil proceedings or proceedings which are not of criminal nature.

90. (d)
Explanation
• Article 23 prohibits traffic in human beings, begar (forced labour) and other similar forms of forced
labour. Any contravention of this provision shall be an offence punishable in accordance with law.
This right is available to both citizens and non-citizens. It protects the individual not only against
the State but also against private persons.
• Article 27 lays down that no person shall be compelled to pay any taxes for the promotion or
maintenance of any particular religion or religious denomination. In other words, the State should
not spend the public money collected by way of tax for the promotion or maintenance of any
particular religion. This provision prohibits the State from favoring, patronizing and supporting
one religion over the other. This means that the taxes can be used for the promotion or
maintenance of all religions. This provision prohibits only levy of a tax and not a fee.

91. (b)
Explanation
• The term “minority” is not defined in the Indian Constitution. However, the Constitution recognizes
only religious and linguistic minorities. The National Commission for Minorities Act, 1992 defines
minorities as notified by the Central government.
• Article 29 preserves distinct language, script, and culture; Article 30 grants education institution
rights.
• Jains have been recognised as Minority in 2014.

92. (d)
Explanation
• Magna Carta was issued in June 1215 and was the first document to put into writing the principle
that the king and his government was not above the law. It sought to prevent the king from
exploiting his power, and placed limits of royal authority by establishing law as a power in itself.
The Magna Carta is a basic document that states liberties guaranteed to the English people. It
proclaims rights that have become a part of English law and are now the foundation of the
Constitution of every English-speaking country.
• The Declaration of the Rights of Man and of the Citizen (1791) is a fundamental document of the
French Revolution and in the history of human and civil rights. The inspiration and content of the
document emerged largely from the ideals of the American Revolution. The Universal Declaration
of the Rights of Man, signed in Paris on 10 December 1948, just like the European Convention on
Human Rights, signed in Rome on 4 November 1950, have the same origins.

19
For all current updates on polity and governance, please follow Saurabh Sir's Telegram Channel
"Polity by Saurabh Kumar”

• Bill of Rights were incorporated in 1791 in the US Constitution after the adoption of the
constitution in 1787.

93. (a)
Explanation
• NCERT is a society registered under the Societies registration act. It is largely an autonomous
body and government control is limited mostly to ensuring that its funds are properly utilised.
• Though the State had provided land to the Ramkrishna Mission, there is an absence of State
control in the management of hospitals created by the Ramkrishna Mission.
• A Cooperative society is not a State unless the tests indicated in Ajay Hasia case are satisfied. It
means case to case basis; one has to see whether cooperatives are under the meaning of State or
not.

94. (b)
Explanation
• Administrative order not the instruction comes under the meaning of Article 13.
• Personal laws as held in Narasu Appa Mali judgement does not come under the meaning of article
13. However, this has been criticised in Sabarimala judgement by CJI, but this judgement has not
overruled Narasu Appa Mali Judgement.
• Regulations made by a statutory body come under the meaning of State.

95. (a)
Explanation
• Equality before law is the part of the law not a vice versa.
• Article 31C was inserted by the 25th Amendment Act of 1971. It contained provisions related to
the saving of laws giving effect to certain directive principles. It contains the following provisions:
No law that seeks to implement Article 39 (b) and 39 (c) in directive principles specified in Part IV
shall be void on the ground of contravention of the fundamental rights conferred by Article 14
(equality before law and equal protection of laws) or Article 19 (protection of six rights in respect
of speech, assembly, movement, etc.)
• Specifying that no state will deny any person equal protection under the law. This amendment to
the U.S. constitution was put in place to prevent state and local jurisdictions from passing laws
that were discriminatory in nature, thus making it illegal for states to pass laws that benefit only
certain groups of people.

96. (a)
Explanation
• While Martial Law affects only fundamental rights, National Emergency has wider implications
upon fundamental rights, federal scheme, distribution of power etc.
• Martial law suspends the government as well as ordinary courts of law. In National Emergency,
ordinary courts of law keep working.
• Martial Law is imposed on account of breakdown or law and order. Emergency is imposed on
account of war, external aggression or armed rebellion.

20
For all current updates on polity and governance, please follow Saurabh Sir's Telegram Channel
"Polity by Saurabh Kumar”

• The constitution of India has no specific provisions on martial law i.e. in what conditions or
circumstances it will be imposed etc. On the other hand, Part XVIII has been dedicated to
emergency provisions.

97. (d)
Explanation
• Article 23 (2) says Nothing in this article shall prevent the State from imposing compulsory service
for public purpose, and in imposing such service the State shall not make any discrimination on
grounds only of religion, race, caste or class or any of them.
• Article 27 is violated when:
➢ There is a tax.
➢ Such tax’s proceeds would go towards the promotion or maintenance of a particular religion
or religious denomination.
➢ A person is being compelled to pay such a tax.
➢ The dominant purpose of the state behind levying such a tax is to intentionally and directly
promote or maintain any particular religion or religious denomination.
• In SP Mittal case, SC gave three test to determine whether the denomination comes under the
meaning of Article 26 or not and these tests are:
1. It should have common faith and organisation and be designated by a distinctive name.
2. A collection of individuals, classed together under the same name.
3. designated by a distinctive name.

98. (d)
Explanation
• The right to freedom of assembly could be restricted on the grounds of In the interests of the
sovereignty and integrity of India, and in the interests of public order.
• Freedom of Trade and Business can be limited on the ground of the professional or technical
qualifications necessary for practising any profession or carrying on any occupation, trade or
business, or the carrying on by the State, or by a corporation owned or controlled by the State, of
any trade, business, industry or service, whether to the exclusion, complete or partial, of citizens
or otherwise.
• Freedom of Movement can be restricted in the interest of the general public or for the protection
of the interest of any Scheduled Tribe.

99. (d)
Explanation
• Fundamental Rights Available Only to Citizens of India
1. Article 15 – Prohibition of discrimination on grounds of religion, race, caste, sex or place of
birth.
2. Article 16 – Equality of opportunity in matters of public employment.
3. Article 19 – Protection of six rights related to freedom – (a) of speech and expression; (b) to
assemble peaceably and without arms; (c) to form associations or unions; (d) to move freely
throughout the territory of India; (e) to reside and settle in any part of the territory of India;
and (f) to practice any profession, or to carry on any occupation, trade or business.

21
For all current updates on polity and governance, please follow Saurabh Sir's Telegram Channel
"Polity by Saurabh Kumar”

4. Article 29 – Protection of language, script and culture of minorities.


5. Article 30 – Right of minorities to establish and administer educational institutions.
• According to Justice PN Bhagwati, Freedom of Speech and expression is exercisable not only in
India but abroad also. State action in India may impair or restrict the exercise of this right
elsewhere.

100. (d)
Explanation
• The Writ of Mandamus cannot be issued against a private individual, to enforce contractual
obligation, against the President, Governor, Chief Justice of the High Court, to enforce
departmental instruction that does not possess statutory force.
• Previously, the writ of certiorari could only be issued against judicial and quasi-judicial
authorities, not administrative ones.
• However, the Supreme Court ruled in 1991 that certiorari can be issued even against
administrative authorities affecting individual rights.
• Certiorari, like prohibition, is not available against legislative bodies or private individuals or
bodies.

101. (d)
Explanation
• All the Statements are Correct. Both Fundamental Rights and DPSP Proceeded on the basis of
Human Rights.

102. (c)
Explanation
• Statement 2, 3 and 4 is a part of Right to life and Personal Liberty, thus Justifiable.
103. (c)
Explanation
• Both the Statements are Correct. These directives do not provide any power nor deter the power
of Parliament to make law.

104. (c)
Explanation
• A Three judge bench in Air India Statutory Corporation vs United Labour Union observed that
these directives are forerunner of the UN Convention on Right to Development.
• If the law is to enforce Article 39 (b) and 39 (c) and it violates Article 14, 19 and 31, then the law
may be declared null and void. So, it may or may not declare it void.

105. (c)
Explanation
• Under Article 39 (d) of the Constitution of India, the State shall direct its policy towards securing
equal pay for equal work for men and women.
• The Directive Principles, though non-justiciable in nature, help the courts in examining and
determining the constitutional validity of a law.
• Fundamental Rights constitutes limitations upon legislative and executive functions.
• Article 50: To separate the judiciary from the executive in the public services of the State.

22
For all current updates on polity and governance, please follow Saurabh Sir's Telegram Channel
"Polity by Saurabh Kumar”

106. (c)
Explanation
• The 86th Amendment Act of 2002 changed the subject-matter of Article 45 and made elementary
education a fundamental right under Article 21 A. The amended directive requires the State to
provide early childhood care and education for all children until they complete the age of six years.

107. (a)
Explanation
• Socialistic Principles reflect the ideology of socialism. They lay down the framework of a
democratic socialist state, aim at providing social and economic justice, and set the path towards
welfare state. Article 39 of the Constitution of India, the State shall direct its policy towards
securing that the operation of the economic system does not result in the concentration of
wealth and means of production to the common detriment. To make provision for just and
humane conditions of work and maternity relief (Article 42).
• To promote the educational and economic interests of SCs, STs, and other weaker sections of the
society and to protect them from social injustice and exploitation (Article 46) is the Gandhian
Principles.
• To secure for all citizens a uniform civil code throughout the country and to protect and improve
the environment and to safeguard forests and wildlife are the Liberal-Intellectual Principles

108. (b)
Explanation
• They aim at establishing social and economic democracy in the country. Directive Principles of
State Policy (DPSP) are positive as they require the State to do certain things.

109. (c)
Explanation
• Both the statements are correct. Parliament will have to enact law for the enforcement of the
Fundamental Duties. Ex: - Forest conservation Act.

110. (a)
Explanation
• There is not any Legal sanction against their Violation. However, the Parliament is free to enforce
them by suitable Legislation. Thus Statement 1 is Correct.
• In 1992, SC Ruled that in determining the Constitutionality of any Law, if a court finds that the
Law in question seeks to give effect to a Fundamental duty, it may consider such law to be
“Reasonable” in relation to Article 14 and article 19. Thus Statement 2 is incorrect.
• As they are enforceable by the Law, Parliament can provide for penalty in case of failure of
fulfilment of these. Thus Statement 3 is correct.

111. (a)
Explanation
• The original constitution did not contain the fundamental duties. It was added into the Indian
constitution by the 42ndconstitutional amendment act 1976, added 10 Fundamental Duties. The
11th Fundamental Duty was added in the year 2002 by the 86th Constitutional Amendment Act.
The Fundamental Duties are confined to citizens only and do not extend to foreigners.

23
For all current updates on polity and governance, please follow Saurabh Sir's Telegram Channel
"Polity by Saurabh Kumar”

112. (a)
Explanation
• In 1976, the Government of India appointed the Sardar Swaran Singh Committee to make
recommendations about Fundamental Duties. It recommended the inclusion of a separate chapter
on Fundamental Duties in the Constitution, which would contain a list of 8 Fundamental Duties.
• The Fundamental Duties in the Indian Constitution are inspired by the Constitution of erstwhile
USSR.
• The duty to pay taxes, despite being recommended by the Committee, was not accepted by the
then government and thus was not incorporated into the Constitution.

113. (c)
Explanation
1. The 86th Constitutional Amendment Act of 2002 introduced an additional Fundamental Duty,
which is to ensure that every citizen provides opportunities for the education of their child or ward
aged between six and fourteen years.

114. (d)
Explanation
• In Case of ordinary bill of the Parliament, Presidential suspensive veto can be over- ridden by the
Re passage of the bill by the Houses. Note: With respect to state bills, state legislature has no
power to override the Suspensive veto of President. Governor can withhold the bill for the
President’s consideration and even if state legislature resends the bill to governor and governor to
President, he still can withhold his assent. Thus statement 1 is Correct.
• He can also keep the bill pending for indefinite Period. Thus statement 2 is correct.
• In India, the President has exercised his absolute veto before. In 1954, it was exercised by Dr.
Rajendra Prasad as a President and later in 1991, it was used by the then President R
Venkataraman. Thus statement 3 is correct.

115. (c)
Explanation
Self-Explanatory

116. (b)
Explanation
• Judges, not the Vice President take the oath to uphold the Constitution of India.

117. (a)
Explanation
Self-Explanatory

118. (b)
Explanation
• Some discretionary powers are as follows:
➢ Can dissolve the legislative assembly if the chief minister advises him to do following a vote of
no confidence.
➢ Can recommend the president about the failure of the constitutional machinery in the state.
➢ Can reserve a bill passed by the state legislature for president’s assent.

24
For all current updates on polity and governance, please follow Saurabh Sir's Telegram Channel
"Polity by Saurabh Kumar”

➢ Can appoint anybody as chief minister If there is no political party with a clear- cut majority
in the assembly.
➢ Determines the amount payable by the Government of Assam, Meghalaya, Tripura and
Mizoram to an autonomous Tribal District Council as royalty accruing from licenses for
mineral exploration.
➢ Can seek information from the chief minister with regard to the administrative and legislative
matters of the state.
➢ Can refuse to sign to an ordinary bill passed by the state legislature.

119. (d)
Explanation
• The Governor shall not hold any other office of profit.
• The Governor shall be entitled without payment of rent to the use of his official residences.
• The Governor shall be also entitled to such emoluments, allowances and privileges as may be
determined by Parliament by law and, until provision in that behalf is so made, such emoluments,
allowances and privileges as are specified in the Second Schedule.
• The emoluments and allowances of the Governor shall not be diminished during his term of office.
The Governor takes Oath of:
➢ Faithfully execute the office of Governor.
➢ Preserve, protect and defend the Constitution.
➢ Devote myself to the service and well-being of the people.

120. (b)
Explanation
• Article 164 (1A) of the Constitution prescribed that the total number of Ministers, including the
Chief Minister, in the Council of Ministers in a State shall not exceed 15% of the total number of
members of the Legislative Assembly of that State.
• This provision was introduced through the 91st Constitution (Amendment) Act, 2003.
• It also Provides that the number of Ministers, including the Chief Minister in a State shall not be
less than twelve.

121. (c)
Explanation
• The electoral college for the election to the president of India:
➢ All the elected members of Lok Sabha and Rajya Sabha.
➢ All the elected members of the legislative assemblies of the states (including L.A. of NCT and
U.T. Puducherry). Since only these two UTs have L.A. Therefore, C is the correct answer.

122. (a)
Explanation
• Only the citizens with the age of 35 years and above can contest for the election to the office of
President. The office has a fixed tenure of five years for any number of terms. The office is an
integral part of Parliament.

123. (c)

25
For all current updates on polity and governance, please follow Saurabh Sir's Telegram Channel
"Polity by Saurabh Kumar”

Explanation
• A Parliamentary Executive means an executive that is dependent on support of the majority in the
parliament.

124. (b)
Explanation
• The President cannot exercise his power of pardon independent of the government.
• In several cases like the Maru Ram Vs UOI, the Supreme Court has ruled that the President has
to act on the advice of the Council of Ministers while deciding mercy pleas.
• Rashtrapati Bhawan forwards the mercy plea to the Home Ministry, seeking the Cabinet’s advice.
• The Ministry in turn forwards this to the concerned state government; based on the reply, it
formulates its advice on behalf of the Council of Ministers.

125. (b)
Explanation
• Governors can issue an ordinance on the matter relating to restrictions on Freedom of Trade and
commerce, but only with the consent of the President in this case.

126. (c)
Explanation
• The Vice President is probably best known as being “a heartbeat away from the presidency”,
meaning that if a sitting President dies or is impeached, the Vice President takes over.
• The Vice President of the United States shall be President of the Senate, but shall have no Vote,
unless they are equally divided.” As head of the upper house of congress, the Vice President votes
on legislation or other motions only when Senators are deadlocked 50-50.
• The Vice President serves in the President’s Cabinet, along with the heads of the 15 executive
departments, the Secretaries of Agriculture, Commerce, Defence etc.

127. (b)
Explanation
• The salary of the Prime Minister of India is funded by the Government of India. The specific amount
is determined by the Parliament of India and is specified in the Prime Minister’s Salary,
Allowances, and Pension Act, 1954. The salary is paid directly to the Prime Minister from the
Consolidated Fund of India, which is a central government account that receives revenue from
various sources, including taxes. Their salary, allowances, and other income are taxable under
the provisions of the Income Tax Act, 1961. The Prime Minister is required to file an annual income
tax return, disclosing their income and paying any applicable taxes.
• The basic Salary of a PM is 2 lakhs/month and that of the MP is 1 lakh/month.

128. (a)
Explanation
• The Cabinet committee on Parliamentary Affairs is headed by the Prime Minister.

129. (d)
Explanation

26
For all current updates on polity and governance, please follow Saurabh Sir's Telegram Channel
"Polity by Saurabh Kumar”

• Direct election for the office of Prime Minister is against the ethos of Parliamentary democracy. In
a Parliamentary democracy, only the elected representatives from the respective constituencies in
Lok Sabha choose their leader to run the government.

130. (c)
Explanation
• Rule No 222 in Chapter 20 of the Lok Sabha Rule Book and correspondingly Rule 187 in Chapter
16 of the Rajya Sabha rulebook governs privilege. It says that a member may, with the consent of
the Speaker or the Chairperson, raise a question involving a breach of privilege either of a member
or of the House or of a committee thereof. The rules however mandate that any notice should be
relating to an incident of recent occurrence and should need the intervention of the House. Notices
have to be given before 10 am to the Speaker or the Chairperson. Thus statement 1 is correct.
• The Speaker/RS chairperson is the first level of scrutiny of a privilege motion. The Speaker/Chair
can decide on the privilege motion himself or herself or refer it to the privileges committee of
Parliament. If the Speaker/Chair gives consent under Rule 222, the member concerned is given
an opportunity to make a short statement. Thus statement 2 is correct.
• Statement 3 is Correct. They can give Punishment.

131. (b)
Explanation
• It is done by Members Present and Voting. Thus statement 1 is Incorrect.
• While there is another test, Composite floor test, which is necessitated when more than one person
stakes the claim to form the government and the majority is not clear. Governor may call a special
session to assess who has the majority. The majority is counted based on those present and voting
and this can be done through voice vote also.

132. (a)
Explanation
• Technically, a session of the Indian Parliament is the period between the first sitting of a House
and its prorogation or dissolution. Thus Statement 1 is Correct.
• The period between the prorogation of a House and its reassembly in a new session is called
‘recess’. Thus Statement 2 is Incorrect.

133. (c)
Explanation
• Parliament exercises control over Executive through debates and discussions on the floor. It has
instruments like short duration discussions during question and zero- hours, calling attention
motion, adjournment motion, no-confidence motion, censure motion, etc.
• It also supervises the activities of the Executive with the help of its committees like committee on
government assurance, committee on subordinate legislation, committee on petitions, etc.
The ministers are collectively responsible to the Parliament in general and to the Lok Sabha in
particular. As a part of collective responsibility, there is individual responsibility, that is, each
minister is individually responsible for the efficient administration of the ministry under his
charge. They continue in office so long as they enjoy the confidence of the majority members in
the Lok Sabha.

134. (a)

27
For all current updates on polity and governance, please follow Saurabh Sir's Telegram Channel
"Polity by Saurabh Kumar”

Explanation
• A Financial bill (I) is treated as an ordinary bill in some cases like:
1. it can be either rejected or amended by the Rajya Sabha.
2. In case of a disagreement between the two Houses over such a bill, the President can summon
a joint sitting of the two Houses to resolve the deadlock.
3. When the bill is presented to the President, he can either give his assent to the bill or withhold
his assent to the bill or return the bill for reconsideration of the Houses.
4. Joint sitting is used in case of Ordinary Bill, not in case of Money Bill, Finance Bill and
Appropriation Bill.

135. (a)
Explanation
• The Appropriation Bill cannot contain expenditure in excess because extra money can’t be drawn
by the Government. Government can withdraw only what is mentioned in the Appropriation Bill.
Thus Statement 2 is Incorrect.
• Article 112 (2) says:
The estimates of expenditure embodied in the annual financial statement shall show separately.
1. the sums required to meet expenditure described by this Constitution as expenditure charged
upon the Consolidated Fund of India
2. the sums required to meet other expenditure proposed to be made from the Consolidated Fund
of India.
• Anything with respect to tax is done through Finance bill which is the last stage of passing the
budget.
136. (d)
Explanation
• Under this procedural device, a Member may, with the prior permission of the Speaker, call the
attention of a Minister to any matter of urgent public importance and the Minister may make a
brief statement thereon. There shall be no debate on such a statement at the time it is made. After
the statement, brief clarifications can be sought from the Minister by the Member who has initiated
the Calling Attention and other Members whose names appear in the List of Business are called
by the Speaker.
• Only those matters which are primarily the concern of the Union Government can be raised
through a Calling Attention notice.
• The Calling Attention procedure is an Indian innovation which combines asking a question with
Supplementaries and making brief comments; the Government also gets adequate opportunity to
state its case.
• The Calling Attention matter is not subject to the vote of the House. Thus, all the Statements are
Correct.

137. (b)
Explanation
• The time immediately following the Question Hour and laying of papers and before any listed
business is taken up in the House has come to be popularly known as the `Zero Hour'.
As it starts around 12 noon, this period is euphemistically termed as `Zero Hour'. For raising
matters during the ‘Zero Hour’ in Lok Sabha, Members give notice between 8.30 a.m. and 9.00
a.m. every day to the Speaker stating clearly the subject which they consider to be important and
wish to raise in the House.

28
For all current updates on polity and governance, please follow Saurabh Sir's Telegram Channel
"Polity by Saurabh Kumar”

• It is, of course, for the Speaker to allow or not to allow for raising such matters in the House. The
term `Zero Hour' is not formally recognised in our parliamentary procedure.

138. (c)
Explanation
• The Constitution provides for an Address by the President to either House or both Houses
assembled together [Article 86(1)]. The Constitution also makes incumbent upon the President to
address both Houses of Parliament assembled together at the commencement of the first Session
after each General Election to the Lok Sabha and at the commencement of the first Session each
year and inform Parliament of the causes of its summons. [Article 87(1)].
• The matters referred to in the Address by the President to the Houses are discussed on a Motion
of Thanks moved by a Member and seconded by another Member.
• No Member can raise questions on the Address by the President. Any action on the part of a
Member which mars the occasion or creates disturbance is punishable by the House to which that
Member belongs. Discussion on matters referred to in the Address takes place on a Motion of
Thanks moved by a Member and seconded by another Member.
• The scope of discussion on the Address is very wide and the functioning of the entire
administration is open for discussion; the limitations inter alia are that Members should not refer
to matters which are not the direct responsibility of the Government of India, and the name of the
President should not be brought in during the debate since the Government, and not the
President, is responsible for the contents of the Address. Thus, all the Statements are Correct.

139. (b)
Explanation
• Discussion on a motion is initiated by the mover of the motion, and later, unless he withdraws it,
the House either adopts in in totality or with such amendments as it may like to make, or it may
reject it altogether.
• The mover of a motion frames it in a form in which he/she wishes it ultimately to be passed by
the House and on which the vote of the House can conveniently be taken.
• The members who wish the motion to be passed in a different form may move amendments after
the original motion has been proposed by the Speaker. These amendments too should be in the
form in which the motion as amended can be passed by the House and must, therefore, be relevant
to the subject matter of the main motion. Thus Statement 2 and 3 is Incorrect.

140. (b)
Explanation
• The Speaker does not leave the House just after dissolution. The Speaker holds office from the
date of her election till immediately before the first meeting of the Lok Sabha after the dissolution
of the one to which she was elected.
• She is eligible for re-election. On the dissolution of the Lok Sabha, although the Speaker ceases
to be a member of the House, she does she does not vacate her office. Thus Statement 1 is Incorrect
and Statement 2 and 3 is Correct.
• Speaker’s decision can be challenges as seen in Aadhaar case. Also Supreme court has clarifies
in Raja Ram pal case: The court noted that proceedings tainted by substantial illegality are open
to judicial scrutiny.

29
For all current updates on polity and governance, please follow Saurabh Sir's Telegram Channel
"Polity by Saurabh Kumar”

141. (a)
Explanation
• The Budget Division of the Department of Economic Affairs in the finance ministry is the nodal
body responsible for producing Budget Limitations of Estimate Committee: -
➢ The power to examine the budget estimates is not an absolute one. The committee can only
examine the budget after it is voted upon and not before that.
➢ Nowhere the power to question the policies of the Parliament has been conferred upon the
committee.
➢ All the recommendations made by the committee are advisory in nature and stand non-binding
for the parliament.

142. (b)
Explanation
• Speaker only presides and President Summons. Thus Statement 1 is Incorrect.

143. (d)
Explanation
• The election of Deputy Chairman shall be held on such date as the Chairman may fix, and the
Secretary shall send notice thereof to every member.
• At any time before noon on the day preceding the day so fixed, any member may give notice in
writing addressed to the Secretary, of a motion that another member be chosen as the Deputy
Chairman of the Council, and the notice shall be seconded by a third member and shall be
accompanied by a statement by the member whose name is proposed in the notice that he is
willing to serve as Deputy Chairman, if elected.

144. (c)
Explanation
• The finance bill is introduced every year to give effect to the financial proposals of the Government
of India for the next following financial year. The procedure in respect of the finance bill is the
same as in the case of other money bills.
• The finance bill is introduced immediately after the presentation of the budget. The introduction
of the bill cannot be opposed. According to the Provisional Collection of Taxes Act, 1931, the
finance bill has to be passed by parliament and assented to by the president before the expiry of
the seventy-fifth day after the day on which it was introduced.
• As the finance bill contains taxation proposals, it is considered and passed by the Lok Sabha only
after the demands for grants have been voted and the total expenditure is known. The Finance
Act completes the process of the enactment of the budget.

145. (b)
Explanation
• Under the Government of India Transaction of Business Rules (TBR), 1961 an executive arm of
the government was assigned the task of conducting the business of it. These Rules emerge out
of Article 77(3) of the Constitution, which states: “The President shall make rules for the more
convenient transaction of the business of the Government of India, and for the allocation among
Ministers of the said business.” Thus, Cabinet Committees are formed which are instrumental in
reducing the workload of the Cabinet.

30
For all current updates on polity and governance, please follow Saurabh Sir's Telegram Channel
"Polity by Saurabh Kumar”

• These committees are extra-constitutional in nature and are nowhere mentioned in the
Constitution.
• There are two types of cabinet committees:
1. Standing Cabinet Committees which are permanent in nature with a specific job. These are
specified in the First Schedule of TBR. The Cabinet Ministers are called its ‘members’ while
the people without the rank of Cabinet Committee are called ‘special invitees’.
2. Ad hoc committees of ministers, including Groups of Ministers (GoMs), may be appointed by
the Cabinet or by the Prime Minister for specific matters. They are temporary in nature and
are formed from time to time.
• The Prime Minister constitutes Standing Committees of the Cabinet and sets out the specific
functions assigned to them. He can add or reduce the number of committees. The composition of
a Cabinet Committee varies from 3 to 8 people.
• Even Ministers who are not part of the Cabinet can be added to a Cabinet Committee. The
members of the Cabinet Committee can be from both the Lok Sabha and the Rajya Sabha.

146. (b)
Explanation
• The Constitution mentions only about Committees, and silent on the Composition. Thus
statement 2 is Incorrect.
• Rest of the Statement is Self-Explanatory.

147. (d)
Explanation
• Once it is clear that there is a hung Parliament, the President/Governor in case of Lok Sabha
and State Legislative assembly takes charge.
• The President invites the leader of the largest single party in the house to form the government.
If this is not possible, then the leader of the largest pre-poll alliance is invited to form the
government. If even this is not an option, the last resort is that the leader of the largest post-poll
alliance is called by the President to form the government, according to the Sarkaria
Commission’s recommendations.
• However, it could happen that the leader of the single largest party may be ruled out if it is clear
that there would be no support for him from other parties. In that case, the President needs to
use his judgement as to who would be able to form and maintain a stable government.
• The onus is on the President to use his discretion to assess the situation, hold consultations and
finally invite a leader who will be capable.

148. (d)
Explanation
• Both Adjournment and Prorogation does not have any effect on the bill.
• Article 196(4) provides that the bill pending in Legislative Council, which has not been passed by
the Legislative assembly, shall not lapse on the dissolution of the Assembly.
• Article 196(5) provides that a Bill which is pending in the Assembly, or which having been passed
by the Assembly is pending in the council, shall lapse on a dissolution of the Assembly.
• Bill pending with the Governor for his assent does not lapse on the dissolution of the Assembly.

31
For all current updates on polity and governance, please follow Saurabh Sir's Telegram Channel
"Polity by Saurabh Kumar”

149. (c)
Explanation
• The MLC can be reappointed any number of times. Thus statement 1 is Incorrect.
• The vacancies are filled by election and nomination by the Governor in every third year. Thus the
2nd Statement is correct.
• Parliament by the law determines their continuation of tenure for 6 years. Thus 3rd statement is
correct.

150. (c)
Explanation
• The law or Constitution does not clearly define what constitutes an office of profit, but the
definition has evolved over the years with interpretations made in various court judgments. An
office of profit has been interpreted to be a position that brings to the office-holder some financial
gain, or advantage, or benefit. The amount of such profit is immaterial.
• The opinion of ECI is binding on the President in such matters. In 1964, the Supreme Court ruled
that the test for determining whether a person holds an office of profit is the test of appointment.
• Several factors are considered in this determination including factors such as: (i) whether the
government is the appointing authority, (ii) whether the government has the power to terminate
the appointment, (iii) whether the government determines the remuneration, (iv) what is the source
of remuneration, and (v) the power that comes with the position.

151. (d)
Explanation
• In Guru Gobinda Basu VS Sankari Pd Ghosal 1964, SC Ruled that CAG, Judges of SC and HC
are not eligible to Contest election because they are serving in Connection with the affairs of the
Union.
• In Sukant VS Vasant Rao 2006, SC ruled that, Holding OOP under Local Authority is not a
disqualification to be elected as a member of SLA and Lok Sabha, but is a disqualification for
being elected as President and Vice President.
• Thus, all the Statements are Correct. Question is asking, which is Incorrect, thus Ans is D

152. (c)
Explanation
• Statement 2 is Incorrect as Readjustment is done only after Census.
• Statement 3 is Incorrect as Power of Enforcement of Orders lies with the Election Commission.

153. (b)
Explanation
• Though the constitution has no direct mention of the "Special Session" but the Article 352 under
Emergency Provisions touch upon a "special sitting of the House.
• The President of India has the authority to convene Parliament on an indefinite number of
occasions, as deemed necessary. Even as the President is responsible for issuing the summons,
it is the Cabinet committee on Parliamentary affairs that initiates the session.
• Special session has been called for more than 10 times. First in 1962 after independence and the
most recent one is in 2023.

32
For all current updates on polity and governance, please follow Saurabh Sir's Telegram Channel
"Polity by Saurabh Kumar”

154. (b)
Explanation
• While the power to summon the house is vested in the President, in practice, the proposal is
initiated by the government. The Department of Parliamentary Affairs intimates the proposed date
of commencement and the duration of a session to the secretaries general of the Rajya Sabha and
the Lok Sabha. After the proposal is agreed by the chairman of Rajya Sabha and the Speaker of
the Lok Sabha, orders of the President to summon the Houses on the date and time specified are
obtained by the secretaries general of the two houses and then they issue summons to the
members individually.
• A Motion of Thanks is passed in the respective houses of the Parliament while the President
addresses the house together in the central hall of the Parliament.

155. (c)
Explanation
• Cases when a bill lapse:
1. A bill originated in the Lok Sabha but pending in the Lok Sabha – lapses.
2. A bill originated and passed by the Rajya Sabha but pending in Lok Sabha – lapses.
3. A bill originated and passed by the Lok Sabha but pending in the Rajya Sabha – lapses.
4. A bill originated in the Rajya Sabha and returned to that House by the Lok Sabha with
amendments and still pending in the Rajya Sabha on the date of the dissolution of Lok Sabha-
lapses.

• Cases when a bill does not lapse:


1. A bill pending in the Rajya Sabha but not passed by the Lok Sabha does not lapse.
2. If the president has notified the holding of a joint sitting before the dissolution of Lok Sabha,
does not lapse.
3. A bill passed by both Houses but pending assent of the president does not lapse.
4. A bill passed by both Houses but returned by the president for reconsideration of Rajya Sabha
does not lapse.
5. Some pending bills and all pending assurances that are to be examined by the Committee on
Government Assurances do not lapse on the dissolution of the Lok Sabha.

156. (b)
Explanation
• The origin of Parliamentary privileges in India can be traced as far back as 1833 when a fourth
member was added to the governor-general’s council following the Charter act 1833.
• The 44th Amendment Act of 1978, allowed the media to publish true reports of parliamentary
proceedings except those related to a secret sitting of the house.

157. (a)
Explanation
• When a question of privilege is referred to the Committee by the House, the report of the Committee
is presented to the House by the Chairman or, in his absence, by any member of the Committee.
• Where a question of privilege is referred to the Committee by the Speaker, the report of the
Committee is presented to the Speaker who may pass final orders thereon or direct that it be laid
on the Table of the House.

33
For all current updates on polity and governance, please follow Saurabh Sir's Telegram Channel
"Polity by Saurabh Kumar”

• The Speaker/Chairman may refer to the Committee any petition regarding the disqualification of
a member on the ground of defection for making a preliminary inquiry and submitting a report to
him.
• The Business Advisory Committee of Lok Sabha consists of 15 members including the Speaker
who is the ex-officio Chairperson. The members are nominated by the Speaker.

158. (a)
Explanation
• Supplementary and Excess Demands for Grants
1. If the amount sanctioned for a particular service is found to be sufficient for the purposes of
that year or when a need arises during the financial year for supplementary or additional
expenditure upon some new service not contemplated in the budget for that year, the President
causes to be laid before both the Houses of Parliament another statement- Supplementary
Demands for Grants showing estimated amount of that expenditure.
2. If any money has been spent on any service during a financial year in excess of the amount
granted for that year, the President causes it to be presented to the Lok Sabha a demand for
such excess. All cases involving such excesses are brought to the notice of the Parliament by
the Comptroller and Auditor General through his report on the Appropriation Accounts. The
excesses are then examined by the Public Accounts Committee which makes recommendations
regarding their regularisation in its report to the House.
3. The Supplementary Demands for Grants are presented to and passed by the House before the
end of the financial year, while the demands for excess grants are made after the expenditure
has actually been incurred and after the financial year to which it relates, has expired.
4. Normally, the vote on account is taken for two months for a sum equivalent to one-sixth of the
estimated expenditure for the entire year under various demands for grants.
5. During an election year, the vote on account may be taken for a longer period, say, three to
four months, if it is anticipated that the main demands and the Appropriation Bill will take
longer than two months to be passed by the House As a convention, vote on account is treated
as a formal matter and passed by the Lok Sabha without discussion. Vote on account is passed
after the general discussion on the Budget is over and before the discussion on demands for
grants is taken up.

159. (b)
Explanation
• A Simple majority is required in the case of election of the Presiding officer of the Lok Sabha of the
Parliament and deputy chairman of the Rajya Sabha, not in the case of Chairman of Rajya Sabha.

160. (a)
Explanation
• The court upheld the sweeping discretion available to the Speaker in deciding cases of
disqualification of MLAs.
• While the Speaker’s decisions can be challenged subsequently, the court cannot stay or prevent
the process. Hence, judicial review cannot be available at a stage prior to the making of a decision
by the Speaker/Chairman and a “quia timet” action would not be permissible. Nor would
interference be permissible at an interlocutory stage of the proceedings. Quia timet is an
injunction to restrain wrongful acts which are threatened or imminent but have not yet
commenced.

34
For all current updates on polity and governance, please follow Saurabh Sir's Telegram Channel
"Polity by Saurabh Kumar”

• Besides, the Court can review only infirmities based on violation of constitutional mandate, mala
fides, non-compliance with rules of natural justice, and perversity.

161. (c)
Explanation
• The law provides for a member to be disqualified if he ‘voluntarily gives up his membership’.
However, the Supreme Court has interpreted that in the absence of a formal resignation by the
member, the giving up of membership can be inferred by his conduct.
• The Supreme Court, in the Ravi Naik vs. Union of India case, has interpreted the phrase
‘voluntarily gives up his membership.' It says: “The words ‘voluntarily gives up his membership'
are not synonymous with ‘resignation' and have a wider connotation.
• A person may voluntarily give up his membership of a political party even though he has not
tendered his resignation from the membership of that party. Even in the absence of a formal
resignation from membership, an inference can be drawn from the conduct of a member that he
has voluntarily given up his membership of the political party to which he belongs.
• In another judgment in the case of Rajendra Singh Rana vs. Swami Prasad Maurya and Others,
the Supreme Court held that the act of giving a letter requesting the Governor to call upon the
leader of the other side to form a Government itself would amount to an act of voluntarily giving
up membership of the party on whose ticket the said members had got elected.

162. (a)
Explanation
• Article 252: - Power of Parliament to legislate for two or more States by consent and adoption of
such legislation by any other State.
• If it appears to the Legislatures of two or more States to be desirable that any of the matters with
respect to which Parliament has no power to make laws for the States except as provided in Articles
249 and 250 should be regulated in such States by Parliament by law, and if resolutions to that
effect are passed by all the House of the Legislatures of those States, it shall be lawful for
Parliament to pass an Act for regulating that matter accordingly, and any Act so passed shall
apply to such States and to any other State by which it is adopted afterwards by resolution passed
in that behalf by the House or, where there are two Houses, by each of the Houses of the
Legislature of that State.
• Any Act so passed by Parliament may be amended or repealed by an Act of Parliament passed or
adopted in like manner but shall not, as respects any State to which it applies, be amended or
repealed by an Act of the Legislature of that State.

163. (c)
Explanation
• With the consent of the State Government, the President may, without any legislative sanction,
entrust any executive function to that State.
• Irrespective of any consent of the State concerned, Parliament may, while legislating with respect
to Union subject, confer powers upon a State or its officers, relating to such subject. Such
delegation has, in short, a statutory basis.

35
For all current updates on polity and governance, please follow Saurabh Sir's Telegram Channel
"Polity by Saurabh Kumar”

• Conversely, with the consent of the Government of India, the Governor of a State may entrust on
the Union Government or its officers, functions relating to a State subject, so far as that State is
concerned.

164. (b)
Explanation
• Statement 1 is Correct - According to Article 268, Centre levied the Stamp duty, but collected
and appropriated by the states. Thus, the proceeds are not part of Consolidated Fund of India.
• Statement 2 is Incorrect - State can also impose cess. Example- Cow cess by UP Government.
• Statement 3 is Correct - According to Article 282, the central government can give grants for any
special purpose.

165. (c)
Explanation
• Statement 1 is Correct - There are 9 Tribunals constituted so far, the earliest one in 1969 and
the recent one in 2018.
• Statement 2 is Correct – River Board shall come into force on such date as the Central
Government may, by notification in the Official Gazette.
• Statement 3 is Correct -In State of Andhra Pradesh vs State of Karnataka 2001, SC says non
implementation of Tribunal award is not a water dispute and comes under SC Jurisdiction and
court can issue Mandamus.

166. (b)
Explanation
• The Executive Power with respect to Concurrent List rests with Centre Only when Constitution
and Parliament both Provides for so. Thus Statement 1 is Incorrect.
• Statement 2 is self-Explanatory and Correct. Firstly, it lays down that the executive powers of the
State are to be exercised in such a manner that it complies with the laws made by the Parliament
or any other existing laws which are applicable in the State. Secondly, it states that the executive
power of the Union includes in its ambit such directions that are given to the State by the Central
Government, which it deems necessary for the purpose. It appears from reading the provision that
if the States duly comply with the first part, then the second part does not seem necessary.
Whereas, if the second part indeed serves its purpose sometimes, then it is evident that the States
are guilty of violating the first part of the provision.
• The Constitution lays down this provision with the assumption that the States will be, at some
juncture, guilty of either wilful defiance or negligence of its duties.
• Although this provision is particularly silent about the consequences in case of non- compliance,
the drastic sanction is laid down in Article 356 of the Constitution. To explain, if a State fails to
comply with the directions issued by the Centre, then it is lawful for the President to hold that a
situation has arisen wherein the State government cannot be carried on according to the
provisions of the Constitution. Consequently, a state emergency can be imposed. The primary
theme of this provision is that there should be a proper execution of the central laws in all the
states.

167. (a)

36
For all current updates on polity and governance, please follow Saurabh Sir's Telegram Channel
"Polity by Saurabh Kumar”

Explanation
• The State Legislature Can Impose tax only on Goods and services inside the State, not outside the
State. Thus Statement 1 is Correct.
• State Legislature Can Impose Tax on Inter State River Valley Project, if bill is reserved for assent
of President. Thus Statement 2 is Incorrect.
• Only Parliament can make Law for Goods and Services in the Course of Inter State Trade and
Commerce. Thus Statement 3 is Incorrect.

168. (b)
Explanation
• Property of Local Authorities within the State are not exempted from Central Taxation. Thus
Statement 1 is Incorrect.
• Exemption of Central Property from State Taxation
➢ The property of Centre is exempted from all taxes imposed by a state or any authority within
a state like municipalities, district boards, panchayats and so on. But, the Parliament is
empowered to remove this ban. The word ‘property’ includes lands, buildings, chattels, shares,
debts, everything that has a money value, and every kind of property movable or immovable
and tangible or intangible. Further, the property may be used for sovereign (like armed forces)
or commercial purposes.
➢ The corporations or the companies created by the Central government are not immune from
state taxation or local taxation. The reason is that a corporation or a company is a separate
legal entity.

• Exemption of State Property or Income from Central Taxation


➢ The property and income of a state is exempted from Central taxation. Such income may be
derived from sovereign functions or commercial functions. But the Centre can tax the
commercial operations of a state if Parliament so provides. However, the Parliament can
declare any particular trade or business as incidental to the ordinary functions of the
government and it would then not be taxable.

169. (b)
Explanation
• Statement 1 and 3 is Correct. The rules for the procedure of meeting at ISC are framed by the
President.
• If any matter is sought up before the Council by the Central Government or by the Government of
any State or Union territory, a formal reference shall be made to the Council, addressed to the
Secretary of the Council.
• On receipt of the reference, the Secretariat of the Council shall examine it in all its aspects. After
reference has been examined, the Secretary of the Council shall submit the case to the Chairman
with his recommendation for obtaining the orders of the Chairman as to whether the issue or
issues raised therein should be included in the Agenda for the meeting of the Council.
• The meetings of the Council shall be held in Delhi, or any other convenient place determined by
the Chairman. Ten members including the Chairman shall form the quorum for a meeting of the
Council.
• The Secretary shall evolve a system of monitoring the action taken on the recommendations of the
Council by the Central Government, the Government of any State or the Union territory concerned.

37
For all current updates on polity and governance, please follow Saurabh Sir's Telegram Channel
"Polity by Saurabh Kumar”

• The meetings of the Council are held in camera. Therefore, the details of the agenda items and
proceedings of the meetings cannot be shared in the public domain.

170. (b)
Explanation
• The Grants in Aid for the Local bodies has been recommended by 14th FC also. Thus statement
1 is Incorrect.
• The 15th FC has again recommended Sector Specific Grants for 8 different sectors like- Nutrition,
Policing, Health, Pre-Primary Education, Housing, Judiciary, Rural Connectivity and Railways,
which was stopped by 14th FC. The statement 2 is Correct. There is a special Grant for Karnataka,
Mizoram and Telangana. Thus statement 3 is Correct.

171. (b)
Explanation
• Statement 1 is Right- Self Explanatory. Under the present form, under Article 352 of the
Constitution, the President can only proclaim an emergency when she/he has a confirmation of
the crisis situation by the Prime Minister and their Cabinet, presented to the President in written
form. Under Article 74, the President can remit back the written dossier for Emergency
proclamation to the Prime Minister and the Cabinet. However, if the Cabinet resends it, the
President must oblige and proclaim an emergency. Unlike in 1975, it is no longer possible for the
Prime Minister to unilaterally take a decision about the Proclamation of an emergency without
any written explanation and transparency.
• Statement 2 is Right - After the Notice, a special session is called within 14 days to decide the
revocation of the emergency.
• Statement 3 is Wrong- President, not the Parliament can modify the transfer of finance from Centre
to State.

172. (c)
Explanation
• Statement 1 is Wrong- President can dismiss the Council of Minister on its own but to dissolve
the State Legislative assembly, Parliament must approve Presidential proclamation.
• Statement 2 is Wrong- Chief Secretary assists the Governor when the President provides power of
administration to the Governor.

173. (a)
Explanation
• If Rajya Sabha passes a resolution supported by 2/3rd members present and voting empowering
parliament to make a law on a matter enumerated in the state list in the best interest of the
country. Such a resolution stays in effect for a year. Such a resolution can be renewed any number
of times but not for more than a year at a time. The laws made under this cease to have an effect
after expiration of six months of the resolution. Thus, the maximum time limit is one year and six
months. However, states can make a law on the same subject, but if there is an inconsistency
between state and union law, the latter prevails.
• When states make a request for Parliament by passing a resolution to that effect then Parliament
becomes empowered to legislate on matters enumerated in the resolution. Once this resolution is
passed, the state forfeits every right with regards to that subject

38
For all current updates on polity and governance, please follow Saurabh Sir's Telegram Channel
"Polity by Saurabh Kumar”

174. (a)
Explanation
• The Centre can delegate executive power to the state with and without conditions while the State
can delegate executive power to the state without conditions only.
• The State can’t delegate executive function to the centre by enacting law for the purpose. It can
be done only through Governor.
• The State government can’t delegate executive function to the centre directly, but through the
Governor only while the centre can delegate executive function to the State directly or through the
President also.

175. (d)
Explanation
Self-explanatory

176. (d)
Explanation
• The State legislature can impose reasonable restrictions on such freedom but only after the
President’s consent.
• The Parliament can appoint an appropriate authority to supervise the provisions of such freedom.

177. (a)
Explanation
• The 44 Amendment Act mandates that the President cannot suspend the right to move the court
for the enforcement of Fundamental Rights guaranteed by Article 20 and 21.
• According to Article 358, when a proclamation of National Emergency is made, the six
fundamental rights under article 19 are automatically suspended for the whole of India, not the
part of India.
• Both the executive action, and law enacted during an emergency can be questioned in the court
of law if these are not related to the emergency.

178. (c)
Explanation
• The Union government is empowered to raise revenue through a gamut of levies, including taxes
(both direct and indirect), surcharges, fees and cess.
• While direct taxes, including income tax, and indirect taxes such as GST are taxes where the
revenue received can be spent by the government for any public purpose in any manner it deems
appropriate for the nation’s good, a cess is an earmarked tax that is collected for a specific purpose
and ought to be spent only for that.
• Every cess is collected after Parliament has authorised its creation through an enabling legislation
that specifies the purpose for which the funds are being raised. Article 270 of the Constitution
allows cess to be excluded from the purview of the divisible pool of taxes that the Union
government must share with the States.

179. (b)
Explanation

39
For all current updates on polity and governance, please follow Saurabh Sir's Telegram Channel
"Polity by Saurabh Kumar”

• Grant-in-aid under Article 275(I) of Constitution of India provided by Govt. of India for
development of infrastructure in the tribal areas. This scheme is implemented through Integrated
Tribal Development Programme.
• Under this scheme Road, Bridges, Dames, School Building, Community Halls, Cultural Complex,
Rural Electrification etc. are undertaken.

180. (d)
Explanation
• In the terms of Financial matters, constitution of India lays down that following bills can be
introduced in the Parliament only on the recommendation of the President:
1. A Bill which imposes or varies any tax or duty in which States are interested.
2. A Bill which imposes any surcharge on tax/duty for the purpose of the centre.
3. A Bill which varies the meaning of the expression” agricultural income”.
4. A Bill which affects the principle on which money is distributed to States.
• The last statement is one of the criteria of the Money Bill, so the President's assent is required.

181. (a)
Explanation
• According to Article 360, it shall be competent for the President during the period any
Proclamation issued under this article is in operation to issue directions for the reduction of
salaries and allowances of all or any class of persons serving in connection with the affairs of the
Union including the Judges of the Supreme Court and the High Courts.

182. (d)
Explanation
• Union Territory of Jammu and Kashmir can’t make law only on Public Order and Police while
Union Territory of Delhi can’t make law on matters related to Public Order, Police and Land. Thus
Statement 1 is incorrect.
• The Lt. governor is empowered to promulgate ordinances during recess of the assembly. An
ordinance has the same force as an act of the assembly. Every such ordinance must be approved
by the assembly within six weeks from its reassembly. He can also withdraw an ordinance at any
time. But he cannot promulgate an ordinance when the assembly is dissolved or suspended.
Further, no such ordinance can be promulgated or withdrawn without the prior permission of the
President.
• The Puducherry Legislative assembly has 33 seats: - 30 elected and 3 nominated by the central
government and Jammu and Kashmir Legislative assembly has 107 seats (24 for POK). Thus
statement 3 is Incorrect.

183. (c)
Explanation
• Administration of Tuensang district shall be carried on by the governor.
• Governor in his discretion shall arrange for equitable distribution of money, b/w Tuensang
district & Rest of Nagaland, provided by the centre.
• There shall be a minister for Tuensang affairs in state COMs.

40
For all current updates on polity and governance, please follow Saurabh Sir's Telegram Channel
"Polity by Saurabh Kumar”

• Final decision on all matters relating to Tuensang district shall be made by the governor at
his discretion.
• Members in Nagaland assembly from the Tuensang district are not elected directly by the
people but by the regional council.

184. (a)
Explanation
• The District Council has Legal, Administrative, judicial Power. By the virtue of this, they can
constitute their Own Court and even the Jurisdiction of HC will apply as specified by the Governor.
Thus Statement 1 is Correct.
• The Governor appoints a commission to examine the matter related to the administration of the
autonomous district and regions. He dissolves DC or RC on the recommendation of these
commissions. Thus statement 2 is Correct.
• There are 3 Autonomous District Councils (ADCs) in Meghalaya namely the Khasi Hills
Autonomous District Council, the Garo Hills Autonomous District Council and the Jantia Hills
Autonomous District Council. Except for a small part of the area under the Shillong Municipality,
the entire state is covered by the District Councils. Term of the District Councils is for five years
from the date of their constitution. Thus statement 3 is incorrect.

185. (a)
Explanation
• Puducherry, Dadra and Nagar Haveli, and Daman and Diu have been created due to cultural
distinctiveness.
• Every union territory is administered by the President acting through an administrator appointed
by him. An administrator of a union territory is an agent of the President and not head of state
like a governor.
• The Parliament can enact laws on any subject from the State List except public order and police
for Jammu and Kashmir.

186. (a)
Explanation
• The Parliament can establish a high court for a union territory or put it under the jurisdiction of
the high court of adjacent state.
• The Constitution does not contain any separate provisions for the administration of acquired
territories. But the constitutional provisions for the administration of union territories also apply
to the acquired territories.
• Under the Government of India (Allocation of Business) Rules 1961, Ministry of Home Affairs is
the nodal ministry for all matters of Union Territories relating to legislation, finance and budget,
services and appointment of Lt. Governors and Administrators.

187. (d)
Explanation
• The President is empowered to declare an area to be a scheduled area. He can also increase or
decrease its area, alter its boundary lines, rescind such designation or make fresh orders for
such redesignation on an area in consultation with the governor of the state concerned.
• Each state having scheduled areas has to establish a tribe’s advisory council to advise on welfare
and advancement of the scheduled tribes. It is to consist of 20 members, three-fourths of whom

41
For all current updates on polity and governance, please follow Saurabh Sir's Telegram Channel
"Polity by Saurabh Kumar”

are to be the representatives of the scheduled tribes in the state legislative assembly. A similar
council can also be established in a state having scheduled tribes but not scheduled areas
therein, if the president so directs.

188. (b)
Explanation
• The governor is empowered to organise and re-organise the autonomous districts. Thus, he can
increase or decrease their areas or change their names or define their boundaries and so on.
• Each autonomous district has a district council consisting of 30 members, of whom four are
nominated by the governor and the remaining 26 are elected on the basis of adult franchise.
• The district and regional councils administer the areas under their jurisdiction. They can make
laws on certain specified matters like land, forests, canal water, shifting cultivation, village
administration, inheritance of property, marriage and divorce, social customs and so on. But all
such laws require the assent of the governor.
• The district and regional councils within their territorial jurisdictions can constitute village
councils or courts for trial of suits and cases between the tribes. They hear appeals from
them. The jurisdiction of the high court over these suits and cases is specified by the governor.

189. (c)
Explanation
• The constitution of India confers the power of judicial review and the power to interpret the
constitution upon the Supreme Court and the High Courts. Therefore, both statements are correct.

190. (c)
Explanation
• The constitution of India confers the power to issue writs on the high courts and the Supreme
Court of India, under art. 32. These writs can be issued for the enforcement of fundamental rights
(by SC and HCs) as well as legal rights (by HCs only). It can be issued against the executive as
well as the judiciary.

191. (d)
Explanation
• Removal of Judges: A judge of a Supreme Court or High Court can be removed only on the ground
of proved misbehaviour or incapacity. A motion containing the charges against the judge must be
approved by a special majority in both Houses of the Parliament. Thus, statement 1 is correct.
• No discussion on the conduct of judges of the Supreme Court can take place inside the Parliament,
except when a motion for the removal of any judge of SC or HC is under consideration. Thus,
statement 2 is correct.

192. (b)
Explanation
• Powers and functions of Supreme Court:
1. Its decisions are binding on all courts within the territory of India.
2. It can transfer any case from one High Court to another.
3. It can transfer (but cannot remove) judges of High Courts.

42
For all current updates on polity and governance, please follow Saurabh Sir's Telegram Channel
"Polity by Saurabh Kumar”

193. (c)
Explanation
• Appellate Jurisdiction: It allows a person to seek advice from the Supreme Court against the
judgments of High Courts.
• Original Jurisdiction: It allows a state to seek advice from the Supreme Court in a matter of conflict
with the centre or any other state.
• Advisory Jurisdiction: It allows the President to seek advice from the Supreme Court on a matter
of public importance or that which involves interpretation of the constitution.
• Under the Constitution, the President may seek the Supreme Court's opinion on two types of
matters.
➢ Any point of law or public fact that has arisen or is likely to arise
➢ Any pre-constitutional contract, agreement, covenant, engagement, or other similar
instrument that gives rise to a dispute.
• In the first instance, the Supreme Court may or may not submit its opinion to the President, but
in the second situation, it is obligated to do so.
• The Supreme Court's decision is advisory rather than binding in both cases. As a result, it does
not bind the President; he or she is free to follow or disregard the opinion.

194. (a)
Explanation
• Statement 1 is Right as it can’t be issued against administrative or executive bodies.
• Statement 2 is wrong because it can’t be issued against Military Tribunal.

195. (c)
Explanation
• Statement 1 is correct: Clause 2 A of Article 124 provides that the age of Judge of the Supreme
court shall be determined by authority decided by Parliament by law provides, but according to
Article 217 (3), Dispute over the age of judge of HC is decided by president.
• Statement 2 is correct: The order of the President can only be passed if it has been addressed by
both the houses in same session (Sarojini Ramaswami vs UOI).

196. (d)
Explanation
Self-explanatory

197. (d)
Explanation
• Original Jurisdiction does not apply to: -
1. The dispute must involve a question of law or fact on which the existence/extent of a legal
right depends. Thus, the questions of political nature are excluded from it.
2. Any suit brought before the Supreme Court by a private citizen against the Centre or a state
cannot be entertained under this article.
3. Also, the provisions mentioned in this Article are subject to other provisions of the
Constitution, i.e., if a remedy to any issue is present under any other Article of the
Constitution, then this Article will not be available. For example, in cases of water disputes

43
For all current updates on polity and governance, please follow Saurabh Sir's Telegram Channel
"Polity by Saurabh Kumar”

between two or more states, the remedy to such conflicts is entertained under article 262, and
not under Article 131.

198. (c)
Explanation
• Both the statements are correct. The rule is all the matter related to substantial question of law
and constitutional law is to be addressed by 5 judges bench.

199. (a)
Explanation
• Statement 1 is correct: Constitution of India debars Judges of SC to appear before any court and
tribunal after retirement.
• Statement 2 is Incorrect: HC judges cannot practice in same court from where they get retirement.

200. (c)
Explanation
• Statement 1 is correct: Though the law-making power rests with Parliament only, but SC says in
Vineet Narain case about continuing Mandamus. It means law made by SC will apply till
Parliament does not enact any law. In this context, SC has law making power as per SC Judges.
• Statement 2 is correct: The ground for transfer is not mentioned in constitution and earlier Judge
from Punjab HC was transferred to Guwahati HC on the charges of corruption.

201. (c)
Explanation
• Statement 1 is Correct: Article 216 says so.
• Statement 2 is also correct: The constitution says about rule making power of SC relating to its
procedure.

202. (b)
Explanation
• SC can refuse Only if appropriate proceedings are not followed. And also, SC can apply doctrine
of latch or delay to reject writ petition. Thus Statement 1 is Incorrect.
• Writ Petitions are part of Fundamental corrects under Article 32. Statements 2 & 3 are correct.

203. (c)
Explanation
• The collegium only sends the name of judges, which is scrutinized by Ministry of Law and Justice
and finally sent to the government to take final decision.

204. (a)
Explanation
• Any Person aggrieved by a ruling can seek Review. Thus Statement 1 is Incorrect.
• As per 1966 Rules framed by the SC, a Review Petition must be filled within 30 Days of the date
of Judgment of the order. Thus Statement 2 is Incorrect.
• HC can also review and correct its own judgment, but such power has not been conferred by the
Constitution. Thus Statement 3 is Correct.

44
For all current updates on polity and governance, please follow Saurabh Sir's Telegram Channel
"Polity by Saurabh Kumar”

205. (c)
Explanation
• Only Initial Posting and promotion is Done by Governor, later one is done by the HC. Thus
Statement 1 is Correct.
• Statement 2 is Correct as it is Self-explanatory.

206. (b)
Explanation
• Human Right Court are established at District level, not at the state level. Thus Statement 1 is
Incorrect.
• Statement 2 and 3 are correct.

207. (d)
Explanation
• Judicial review is the power exerted by the courts of a country to examine the actions of the
legislatures, executive and administrative arms of government and to ensure that such actions
conform to the provisions of the nation’s Constitution.
• Judicial review has two important functions, like, of legitimizing government action and the
protection of constitution against any undue encroachment by the government.

208. (c)
Explanation
• Article 129 and 215 of the Constitution of India empowers the Supreme Court and High Court
respectively to punish people for their respective contempt.
• Section 10 of The Contempt of Courts Act of 1971 defines the power of the High Court to punish
contempt of its subordinate courts.
• The Constitution also includes contempt of court as a reasonable restriction to the freedom of
speech and expression under Article 19, along with elements like public order and defamation.

209. (d)
Explanation
Self-explanatory

210. (b)
Explanation
• As per Article 348 (1) of the Constitution of India, English is the official language for all the high
courts. However as per Clause (2) of Article 348, the Governor of a state, with the previous consent
of President of India, can authorize use of official language of the state in proceedings before its
high court.

211. (a)
Explanation
• According to the tribunal reform act:
1. As per the Act, the minimum age criterion is 50 years for appointment of advocates as
members of tribunals and the tenure is four years.
2. The Chairperson and Members of the Tribunals will be appointed by the central government
on the recommendation of a Search-cum-Selection Committee. The Committee will consist of:

45
For all current updates on polity and governance, please follow Saurabh Sir's Telegram Channel
"Polity by Saurabh Kumar”

➢ The Chief Justice of India, or a Supreme Court Judge nominated by him, as


the Chairperson (with casting vote).
➢ Two Secretaries nominated by the central governments.
➢ The sitting or outgoing Chairperson, or a retired Supreme Court Judge, or a retired
Chief Justice of a High Court, and
➢ The Secretary of the Ministry under which the Tribunal is constituted (with no voting
right)
3. It provides for a four-year term of office (subject to the upper age limit of 70 years for the
Chairperson, and 67 years for members).

212. (b)
Explanation
• Statement 1 and 2 is Correct and 3 is Incorrect
• Government first declares any organisation and Individual as Terrorist; then accordingly the
Tribunal may uphold or term it void.

213. (b)
Explanation
• Statement 1 is incorrect: The judges are appointed by the State Government and there are
Conciliators in place of Lawyers.
• Statement 2 is correct: It is mandatory for the State Government to set up a Family Court for every
area in the State comprising a city or a town whose population exceeds one million.

214. (c)
Explanation
• Permanent Lok Adalat’s have been set up as permanent bodies with a Chairman and two members
for providing compulsory pre-litigative mechanism for conciliation and settlement of cases relating
to Public Utility Services like transport, postal, telegraph etc. Here, even if the parties fail to reach
to a settlement, the Permanent Lok Adalat gets jurisdiction to decide the dispute, provided, the
dispute does not relate to any offence.
• Further, the Award of the Permanent Lok Adalat is final and binding on all the parties.
• The jurisdiction of the Permanent Lok Adalat is Up to Rs. One Crore.
• Here if the parties fail to reach to a settlement, the Permanent Lok Adalat has the jurisdiction to
decide the case.
• The award of the Permanent Lok Adalat is final and binding upon the parties.

215. (b)
Explanation
• The Polluter Pays Principle is a well-recognized and much celebrated environmental law principle.
Since its introduction in 1972, the Polluter Pays Principle has been given a much broader sense.
Therefore, not only does it cover pollution prevention and control measures, but it also covers
liability such as costs for the clean-up of damage to the environment. The Polluter Pays Principle
was for the first time explicitly applied and defined in the case of Indian council for enviro legal
action vs UOI.
• The NGT has also invoked the principle of liability in Vishakhapatnam Gas leakage case. Following
are certain differences between the principles of absolute liability and strict liability:

46
For all current updates on polity and governance, please follow Saurabh Sir's Telegram Channel
"Polity by Saurabh Kumar”

1. Whereas strict liability allows exceptions if the liability has been accrued by an Act of God, act
of third party etc., absolute liability offers no exception to industries involved in hazardous
activities, which are liable for the damage so triggered notwithstanding adherence to the
highest safety norms.
2. Under absolute liability, the extent of damages depends on the magnitude and financial
capability of the organisation. However, under strict liability, compensation is payable as per
the nature and quantum of damages caused.
3. Under the doctrine of absolute liability, the element of escape is not essential. In other words,
rule of absolute liability shall be applicable to those injured within the premise and persons
outside the premise. However, the same is not the case under the doctrine of strict liability.

216. (d)
Explanation
1. Under Section 16 of the Advocates Act 1961 two classes of advocates are classified; Senior
Advocate and Junior or those who are not designated as seniors.
2. Section 16(2) of the Advocates Act, 1961 and Rule 2(a) of Order IV of the Supreme Court Rules,
1966 provide certain guidelines to be followed for the designation of a senior counsel.
3. As per these provisions:
• The Chief Justice and other judges of the concerned court should believe that a particular
advocate is fit to hold the position of a senior advocate.
• The concerned advocate should have exceptional legal expertise and knowledge of the law.
• The consent of such an advocate should be obtained prior.
• The selection should be on the sole ground of his knowledge and expertise in the area of law.
4. The new guidelines prescribe the minimum age as 45 years to apply for the ‘senior advocate’
designation. This age limit may, however, be relaxed by the Committee, the Chief Justice of India,
or a Supreme Court judge if they have recommended an advocate’s name.
5. A “Committee for Designation of Senior Advocates,” or “permanent committee,” is created and
empowered with powers of conferment. The CJI-chaired committee is to consist of two senior-most
SC judges, the Attorney General of India, and a “member of the Bar” nominated by the chair and
other members.
6. The CJI or any other judge could recommend the name of an advocate for designation.
Alternatively, advocates could submit their applications to the “Permanent Secretariat”, which
would evaluate them on criteria like 10–20 years of legal practice, be it as an advocate, district
judge, or judicial member of an Indian tribunal where the qualification for eligibility is not less
than that prescribed for a district judge.

217. (b)
Explanation
• An injunction is a prohibitive writ issued by a court of equity, at the suit of a party complainant,
directed to a party defendant in the action.
• The law of injunction has been provided for by the Specific Relief Act, 1963 and is also regulated
by the Code of Civil Procedure, 1908 in India.
• This writ can be provided by both Supreme or High court.

218. (d)
Explanation

47
For all current updates on polity and governance, please follow Saurabh Sir's Telegram Channel
"Polity by Saurabh Kumar”

• Elections for local bodies are conducted by State Election Commission. The SEC is appointed by
governor of the state. In these elections, the seats reserved for SC and STs are in proportion to
their population in the constituency. The minimum age to contest in these elections is 21 years.
Hence, all the statements above are incorrect.

219. (b)
Explanation
• The merits of local self-government:
1. Reduces the chances of corruption in the government.
2. Strengthens democracy at the grassroot level.
3. Improves the efficiency of policy implementation in the government.

220. (b)
Explanation
• Gram Sabha, not Gram Panchayat, is the body where all the voters in the village are members.
• Gram Panchayat is a council under Panchayati Raj, consisting of several ward members called
sarpanch or President. It is the decision-making body for the entire village.
• It is a legal body.

221. (c)
Explanation
• As per provisions contained in Article 243 D of the Constitution, 1/3rd of the Seats of Panchayati
Raj Institutions and 1/3rd offices of the Chairperson at all level of Panchayati Raj Institutions
covered by Part IX of the Constitution are reserved for women.
• One-third of the total number of seats to be reserved for women. One third of the seats reserved
for SCs and STs are reserved for women. Thus, Both the Statements are Correct.

222. (b)
Explanation
• Bihar, Chhattisgarh, Jharkhand, UP, UK, MP, Orissa: - Mayors are directly elected by the people
and thus hold the executive powers of the municipal corporations.
• The term of Mayor ranges from 1 to 5 years.
• The executive head of Municipal corporation is Municipal commissioner who is appointed by state
government.

223. (d)
Explanation
• In Union of India v/s R.C. Jain, to be deliberated a “local authority”: - The authority must satisfy
the below given rules:
i. Separate legal existence.
ii. Role in a defined area.
iii. Has power to raise funds.
iv. Enjoys independence.

224. (b)
Explanation

48
For all current updates on polity and governance, please follow Saurabh Sir's Telegram Channel
"Polity by Saurabh Kumar”

• Statement 1:- It applies with modification if the President directs, otherwise it applies
automatically.
Every District Planning Committee shall, in preparing the draft development plan, have regard to:-
1. Matters of common interest between the Panchayats and the Municipalities including spatial
planning, sharing of water and other physical and natural resources, the integrated
development of infrastructure and environmental conservation.
2. The extent and type of available resources whether financial or otherwise.
3. Consult such institutions and organisations as the Governor may, by order, specify.
• The Chairperson of every District Planning Committee shall forward the development plan, as
recommended by such Committee, to the Government of the State.

225. (d)
Explanation
• The duties and powers of the Comptroller and Auditor General in relation to accounts are
contained in Section 10, 11 and 12 of the Comptroller and Auditor General’s (Duties, Powers and
Conditions of Service) Act, 1971. Earlier it was contained in the Government of India (Audit and
Accounts) order 1936 in pursuance of Government of India Act, 1935, according to which Auditor
General was responsible for keeping the accounts of the Federation and of each Province. In 1976,
the President relieved C&AG of the responsibility of compiling the accounts of the Union
Government in terms of proviso under section 10 of the Act. The Act confers similar powers to the
Governor of a State in respect of State Government Accounts. However, except the State of Goa,
where accounts compilation is not with the C&AG, in all other States, accounts compilation
remains the responsibility of the C&AG. Under the Act the Comptroller and Auditor General is
now responsible for compiling the accounts of:
(a) All States (except Goa);
(b) The Union Territories of Chandigarh, Dadra and Nagar Haveli; and
(c) Indian Audit and Accounts Department.
• Secret service expenditure is outside the purview of the CAG and he cannot call for particulars of
expenditure incurred by the executive agencies, but has to accept a certificate from the competent
administrative authority that the expenditure has been so incurred.

226. (d)
Explanation
• Office of the Accountant General was established in 1858 (the year the British took over
administrative control of India from the East India Company). In 1860 Sir Edward Drummond
was appointed as the first Auditor General.
• Meanwhile after some restructuring the Auditor General of India came to be called the Auditor
and Accountant General to the Government of India.
• In 1866, the position was renamed Comptroller General of Accounts, and in 1884, it was re-
designated as Comptroller and Auditor General of India.
• Under the Government of India Act 1919, the Auditor General became independent of the
government as statutory backing was given for the position. Thus Statement 1 is Correct.

227. (b)
Explanation
• Accountant General', means the Head of an Office of Accounts subordinate to the Comptroller and
Auditor General of India.

49
For all current updates on polity and governance, please follow Saurabh Sir's Telegram Channel
"Polity by Saurabh Kumar”

• The Muddiman committee has recommended separation of account and audit as a necessary
financial reform for the first time

228. (b)
Explanation
• CAG does audit of Panchayat and Municipalities on the request of the President and Governor.
RBI and LIC is audited by internal auditors.
229. (c)
Explanation
Self-Explanatory

230. (a)
Explanation
• Statement 1 is Incorrect. No such function exists for the AG.
• The system of AG can be traced to UK. Thus Statement 2 is Incorrect.
• Because of this he is not paid salary but a retainer to be determined by the President.
• The Attorney General gets a retainer equivalent to the salary of a judge of a Supreme Court. This
retainer is paid from Consolidated Fund of India. Thus Statement 3 is Correct.

231. (d)
Explanation
• There is a clear distinction between the Advocate General and the Additional Advocate General.
Whereas the Advocate General is a Constitutional Authority, Additional Advocate General is
merely an Advocate retained by the State Government.
• Whereas the Advocate General can be only one, appointed under Article 165 of the Constitution
of India, Additional Advocate General is appointed under the executive powers of State under
Article 162 of the Constitution of India, and there can be more than one Additional Advocate
General, to defend the State Government.

232. (b)
Explanation
• Chief Electoral Officer is the head at the State level for both Election in Lok Sabha and election at
State Legislative Assembly. Thus Statement 1 is Incorrect.
• Candidate has to take oath before Returning or Assistant Returning Officer after Nomination, if
the Candidate is outside India, Then Oath will be administered before High Commissioner or
Ambassador. Thus Statement 2 is Correct.
• Statement 3 is Factual and Correct. An EVM being used by ECI can maximum of 2,000 votes

233. (a)
Explanation
• The EVMs were first used in 1982 in the by-election to North Paravur Assembly Constituency in
Kerala for a limited number of polling stations. Thus Statement 1 is Incorrect.
• EVMs can cater to a maximum of 64 candidates including NOTA. There is provision for 16
candidates in a Balloting Unit. If the total number of candidates exceeds 16, more balloting units
can be attached (one per 16 candidates) up to a maximum of 64 candidates by connecting 4
Balloting Units. However, in case of M3 EVMs (Post 2013), EVMs can cater to a maximum of 384
candidates including NOTA by connecting 24 Balloting Units. Thus Statement 2 is Incorrect.

50
For all current updates on polity and governance, please follow Saurabh Sir's Telegram Channel
"Polity by Saurabh Kumar”

• VVPATs with EVMs were used for the first time in a bye-election from 51-Noksen (ST) Assembly
Constituency of Nagaland. Thus Statement 3 is Correct.

234. (c)
Explanation
• As per the guidelines of Ministry of Law and Justice, issued in October 1979, the entire
expenditure on the actual conduct of elections to Lok Sabha is borne by the Government of India.
In the case of elections to the State Assembly when they are held independently, the expenditure
is borne by the respective State Government.
• If the elections to the Lok Sabha & State Assembly are held simultaneously, then the expenditure
is shared equally between the Government of India & the State Government. The Election
Commission of India (ECI) bears expenditure for honorarium to the observers deployed during
elections.

235. (b)
Explanation
• The State Finance Commission has been recommended for the first time by the Thungon
Committee and introduced in the 73rd Amendment. Thus Statement 1 is Incorrect.
• Statement 2 is Correct as it is self-Explanatory.
• It shall be constituted after every 5 years or earlier also. Thus statement 3 is Correct.

236. (b)
Explanation
• The President by Notification declares any Community as SC. Thus Statement 1 is Correct.
Parliament not the Central Government can modify the Order. Thus Statement 2 is Incorrect.
• The Commission presents its report to the President and the President sends the report to the
Governor. Thus Statement 4 is Incorrect.

237. (d)
Explanation
• NCM Consists of Chairperson, Vice Chairperson and 5 Members. Statement 1 is Correct.
• It Consists of Minorities members only. Statement 2 is Correct.
• National Commission for minority Educational Institutions recommends for granting Minority
status to Institution under section 11 of the NCMEA Act. Thus statement 3 is Incorrect.

238. (c)
Explanation
• Rules under Article 309 Can be framed by the President and Governor only if Parliament and SLA
have not made the Laws. Thus statement 1 is Correct. Reasonable opportunity to hear is sought
in article 311(2). It is not available in following Cases: -
1. In case of Criminal charges. Thus statement 2 is Correct.
2. When Public servant is considered as a threat to Security of the Country.

51
For all current updates on polity and governance, please follow Saurabh Sir's Telegram Channel
"Polity by Saurabh Kumar”

• Where the appropriate disciplinary authority is satisfied, for reasons to be


recorded by that authority in writing that it does not consider it reasonably practicable to give to
the person an opportunity of showing cause, no such opportunity need be given.

239. (d)
Explanation
• Electoral registration officer is responsible for preparation of electoral roll. He is appointed by ECI
in consultation with state/UT Government.
• Chief electoral officer is not assisted by returning officer, he himself determines start campaigner.
Polling officer looks into timely conduct of election.

240. (d)
Explanation
• Both the Statement are Correct and Self Explanatory, Question is asking for Incorrect One. Thus
the answer is D.

241. (b)
Explanation
• The 101st Amendment Act of 2016 paved the way for the introduction of a new tax regime (i.e.
goods and services tax - GST) in the country. The amendment inserted a new Article 279-A in the
Constitution. This Article empowered the President to constitute a GST Council by an order.
• The Secretariat of the Council is located at New Delhi. The Union Revenue Secretary acts as
the ex-officio Secretary to the Council.
• Vision of GST council is to establish the highest standards of co-operative federation in the
functioning of the Council, which is the first constitutional federal body vested with powers to
take all major decisions relating to GST.
• The Council is a joint forum of the centre and the states and consists of the following members:
The Union Finance Minister as the Chairperson, the Union Minister of State in-charge of
Revenue or Finance and he Minister in-charge of Finance or Taxation or any other Minister
nominated by each state government.

242. (d)
Explanation
• The decisions of the Council are taken at its meetings. One-half of the total number of members
of the Council is the quorum for conducting a meeting. Every decision of the Council is to be taken
by a majority of not less than three-fourths of the weighted votes of the members present and
voting at the meeting.

243. (d)
Explanation
• Originally, the Constitution of India did not make any provision with respect to the Special
Officer for Linguistic Minorities. Later, the States Reorganisation Commission (1953–55) made a
recommendation in this regard. Accordingly, the Seventh Constitutional Amendment Act of
1956 inserted a new Article 350- B in Part XVII of the Constitution.
• This article contains the following provisions:

52
For all current updates on polity and governance, please follow Saurabh Sir's Telegram Channel
"Polity by Saurabh Kumar”

1. There should be a Special Officer for Linguistic Minorities. He is to be appointed by the


President of India.
2. It would be the duty of the Special Officer to investigate all matters relating to the safeguards
provided for linguistic minorities under the Constitution.
3. The Commissioner has his headquarters at Allahabad (Uttar Pradesh). He has three
regional offices at Belgaum (Karnataka), Chennai (Tamil Nadu) and Kolkata (West Bengal).
Each is headed by an Assistant Commissioner.
4. It must be noted here that the Constitution does not specify the qualifications, tenure,
salaries and allowances, service conditions and procedure for removal of the Special
Officer for Linguistic Minorities.
5. In pursuance of the provision of Article 350-B of the Constitution, the office of the Special
Officer for Linguistic Minorities was created in 1957. He is designated as the Commissioner
for Linguistic Minorities.
6. At the Central level, the Commissioner falls under the Ministry of Minority Affairs.

244. (b)
Explanation
• The Union Public Service Commission (UPSC) is the central recruiting agency in India. It is an
independent constitutional body in the sense that it has been directly created by the Constitution.
Articles 315 to 323 in Part XIV of the Constitution contain elaborate provisions regarding the
composition, appointment and removal of members along with the independence, powers
and functions of the UPSC.
• The UPSC consists of a chairman and other members appointed by the president of India. The
Constitution, without specifying the strength of the Commission has left the matter to the
discretion of the president, who determines its composition. Usually, the Commission consists
of nine to eleven members including the chairman.
• The entire expenses including the salaries, allowances and pensions of the chairman and
members of the UPSC are charged on the Consolidated Fund of India. Thus, they are not
subject to vote of Parliament.

245. (a)
Explanation
• The State Public Service Commission consists of a chairman and other members appointed by the
governor of the state. The Constitution does not specify the strength of the Commission but has
left the matter to the discretion of the Governor. Further, no qualifications are prescribed for the
commission’s membership except that one-half of the members of the commission should be such
persons who have held office for at least ten years either under the government of India or under
the Government of a state.

• The Constitution also authorises the governor to determine the conditions of service of the
chairman and members of the Commission.
• The chairman and members of the Commission hold office for a term of six years or until they
attain the age of 62 years, whichever is earlier (in the case of UPSC, the age limit is 65 years).
• SPSC is not consulted, while making reservations of appointments or posts in favour of any
backward class of citizens and while taking into consideration the claims of scheduled castes and
scheduled tribes in making appointments to services and posts.

53
For all current updates on polity and governance, please follow Saurabh Sir's Telegram Channel
"Polity by Saurabh Kumar”

246. (b)
Explanation
• Article 280 of the Constitution of India provides for a Finance Commission as a quasi-judicial
body.
• It is constituted by the President of India every fifth year or at such earlier time as he considers
necessary.
• The Finance Commission consists of a chairman and four other members to be appointed by the
president. They hold office for such period as specified by the president in his order. They are
eligible for reappointment.
• The Constitution authorizes the Parliament to determine the qualifications of members of the
commission and the manner in which they should be selected.
• K.C. Neogy was the chairman of the 1st Finance Commission of India

247. (a)
Explanation
• The recent amendment Include Chairperson of the National Commission for Backward Classes,
Chairperson of the National Commission for the Protection of Child Rights and the Chief
Commissioner for Persons with Disabilities as Ex-Officio Members of the Commission. Thus
Statement 1 is Correct.
• The Salaries and service conditions are determined by the Central Government, not by Parliament.
Thus Statement 2 is Incorrect. Statement 3 is factual and correct.

248. (c)
Explanation
• Statement 1 is factual and correct.
• The Lokpal shall provide to every person against whom a complaint has been made, before it,
under this Act, legal assistance to defend his case before the Lokpal, if such assistance is
requested for. Thus Statement 2 is Correct

249. (b)
Explanation
• NIA is headed by a Director General, who is an IPS officer and has the rank of Director
General of Police
• The Central Government for the trial of Scheduled Offences constitutes one or more Special
Courts under Section 11 and 22 of the NIA Act 2008.
• The schedule to the Act specifies a list of offences which are to be investigated and
prosecuted by the NIA.
• The list includes
➢ Explosive Substances Act
➢ Atomic Energy Act
➢ Unlawful Activities (Prevention) Act
➢ Anti-Hijacking Act
➢ Suppression of Unlawful Acts against Safety of Civil Aviation Act
➢ SAARC Convention (Suppression of Terrorism) Act

54
For all current updates on polity and governance, please follow Saurabh Sir's Telegram Channel
"Polity by Saurabh Kumar”

➢ Suppression of Unlawful Acts Against Safety of Maritime Navigation and Fixed Platforms
on Continental Shelf Act
➢ Weapons of Mass Destruction and their Delivery Systems (Prohibition of Unlawful
Activities) Act
➢ Any other relevant offences under the Indian Penal Code, Arms Act and the Information
Technology Act.
➢ Narcotic Drugs and Psychotropic Substances Act

250. (a)
Explanation
• The Representation of the People Act, 1951 allows a person to contest a general election or by-
elections or biennial elections from a maximum of two constituencies but the candidate can retain
only one.
• Before a 1996 amendment in the electoral laws, there was no bar on the number of seats a person
could contest.
• Returning Officer could reject nomination papers on the ground that candidate information was
false.
• Election commission can only file FIR against a person, failing to lodge an account of election
expenses.

251. (a)
Explanation
• Only officers with a minimum of six months of service left before retirement will be considered
for the position of State DGP.
• The guidelines set a cap of three shortlisted officers for the DGP position, allowing exceptions
only under specific circumstances
• Empanelment Committee established by the UPSC for the appointment of State DGP will refrain
from assessing IPS officers on central deputation for the State DGP position if the Union Ministry
of Home Affairs informs the State government that releasing the officers is not feasible.

252. (c)
Explanation
• It will have a Chief Commissioner as head, and only two other commissioners as members — one
of whom will deal with matters relating to goods while the other will look into cases relating to
services.
Impose a penalty up to Rs 10 lakh, with imprisonment up to two years, on the manufacturer or
endorser of false and misleading advertisements. The penalty may go up to Rs 50 lakh, with
imprisonment up to five years, for every subsequent offence committed by the same manufacturer
or endorser.
• Ban the endorser of a false or misleading advertisement from making endorsement of any products
or services in the future, for a period that may extend to one year. The ban may extend up to three
years in every subsequent violation of the Act.

253. (b)
Explanation
• Any person who, does not receive a decision within the time specified or is aggrieved by a decision
of the Central Public Information Officer or State Public Information Officer, as the case may be,
may within thirty days from the expiry of such period or from the receipt of such a decision prefer

55
For all current updates on polity and governance, please follow Saurabh Sir's Telegram Channel
"Polity by Saurabh Kumar”

an appeal to such officer who is senior in rank to the Central Public Information Officer or State
Public Information Officer (First Appeal)
• A second appeal against the decision shall lie within ninety days from the date on which the
decision should have been made or was actually received, with the Central Information
Commission or the State Information Commission.
• NGOs which receive considerable and substantial finances from the government or are essentially
dependent on the government fall under the category of “public authority” defined in Section 2(h)
of the RTI Act of 2005. The court defined “substantial” as a “large portion.” It does not necessarily
have to mean a major portion or more than 50%. No hard and fast rule can be laid down in this
regard. Substantial financing can be both direct or indirect.
• In 2018, a division bench of the Delhi high court was split on the issue of whether
• PMNRF is a public authority under the RTI Act and is liable to disclosure of information to
applicants. The matter was thereafter forwarded to the acting chief justice of the HC for an opinion.
The issue is still pending.

254. (a)
Explanation
• Deciding appeals under the RTI Act is a quasi-judicial function
• While the first appeal lies within the public authority itself, the second appeal lies with the Central
Information Commission.
• The Supreme Court has held that non-governmental organisations [NGO] substantially financed,
whether directly or indirectly, by the appropriate government fall within the ambit of 'public
authority' under Section 2(h) of the Right to Information Act, 2005.
• The Prime Minister's National Relief Fund (PMNRF) is not a 'public authority' within the meaning
of Section 2(h) of the Right to Information Act 2005 even PMCARES is not a 'public authority', sp
both can be rejected by PIO

255. (d)
Explanation
• Section 4 of the RTI Act calls for Suo-Moto Disclosure. Thus Statement 1 is Correct. It can be
asked in electronic means and written request only. Thus Statement 2 is also correct.
• Recently, NGOs which receive considerable finances from the government or are essentially
dependent on the government fall under the category of “public authority” defined in Section 2(h)
of the RTI Act of 2005. This means that they have to disclose vital information, ranging from
finances to hierarchy to decisions to functioning, to citizens who apply under RTI. An NGO may
also include societies which are neither owned nor controlled by the government, but if they are
significantly funded by the government, directly or indirectly, they come under the RTI Act. Thus
Statement 3 is Correct.

256. (c)
Explanation
• Powers of NCPCR to inquire into such complaints: Under the RTE Act, 2009, the NCPCR can:
1. inquire into complaints about violation of the law.
2. summon an individual and demand evidence.
3. seek a magisterial enquiry.
4. file a writ petition in the High Court or Supreme Court.
5. approach the government concerned for prosecution of the offender.

2
For all current updates on polity and governance, please follow Saurabh Sir's Telegram Channel
"Polity by Saurabh Kumar”

6. recommend interim relief to those affected.

257. (b)
Explanation
• Statement 1 is correct: The Competition Commission of India (CCI) was established in March 2009
by Government of India under the Competition Act, 2002 for the administration, implementation,
and enforcement of the Act. The commission is a quasi-judicial body, gives opinions to statutory
authorities.
• Statement 2 is not correct: CCI consists of one Chairperson and six Members as per the
Competition Act who shall be appointed by the Central Government.
• Statement 3 is correct: The objectives are:
1. Eliminate practices having adverse effect on competition
2. Promote and sustain competition
3. Protect the interests of consumers
4. Ensure freedom of trade in the markets of India
5. Establish a robust competitive environment through: Proactive engagement with all
stakeholders, including consumers, industry, government and international jurisdictions.

258. (a)
Explanation
• Delimitation Commission is appointed by the President. The present delimitation of constituencies
has been done on the basis of 2001 census data under the provisions of Delimitation Commission
Act, 2002.The next Delimitation Commission will be set up after 2026.
• The Commission’s orders have the force of law and cannot be called in question before any court.

259. (a)
Explanation
• Powers of the Directorate of Enforcement
1. The authority to seize assets.
2. The authority to summon individuals with the same powers as a civil court, including
discovery, inspection, evidence production, summons issuance, examination, and commission
issuance.
3. The power to investigate and make arrests for violations of the Prevention of Money Laundering
Act (PMLA) 2002 and the Foreign Exchange Management Act (FEMA) 1999 without awaiting
the formal filing of an FIR by the police.
4. The admissibility of statements recorded by officials of the Enforcement Directorate (ED) as
evidence in court, as ruled by the Supreme Court in 2022.
5. The capability to recover fines, penalties, and arrears of penalties under the FEMA act.

260. (a)
Explanation
• The establishment of the CBI was recommended by the Santhanam Committee on Prevention of
Corruption (1962–1964). The CBI is not a statutory body. It derives its powers from the Delhi
Special Police Establishment Act, 1946. The CBI is headed by a Director. He is assisted by a
special director or an additional director.

3
For all current updates on polity and governance, please follow Saurabh Sir's Telegram Channel
"Polity by Saurabh Kumar”

• The CBI investigates crime of corruption, economic offences and serious and organized crime other
than terrorism. It is the nodal police agency in India, which coordinates investigation on behalf of
Interpol Member countries.
• General consent, as stipulated in Section 6 of the Delhi Special Police Establishment Act, 1946,
is required for the Central Bureau of Investigation (CBI) to operate within a state. This consent
can be either general or case specific. General consent is the default permission granted to the
CBI to conduct investigations within a state.
• There are exceptions to the requirement of general consent. The Supreme Court and High Courts
have the authority to direct the CBI to investigate a crime anywhere in the country without the
need for consent from the concerned state government. Additionally, the requirement for consent
does not apply in cases where individuals are apprehended in the act of accepting bribes.

261. (b)
Explanation
• The Lokpal and Lokayukta Act, 2013 provided for the establishment of Lokpal for the Union and
Lokayukta for States. These institutions are statutory bodies. Lokpal cannot suo motu proceed
against any public servant. Lokpal is a multi-member body, that consists of one chairperson and
a maximum of 8 members
• The Prime Minister has been brought under the purview of the Lokpal with subject matter
exclusions and specific process for handling complaints against the Prime Minister. Lokpal’s
jurisdiction will cover all categories of public servants, including Group A, Group B, Group C, and
Group D officers and employees of Government. Lokpal cannot suo motu proceed against any
public servant.

262. (b)
Explanation
• The NIA was established in the backdrop of the 2008 Mumbai terror attacks, popularly known as
the 26/11 incident. The NIA is headed by a Director-General. He is appointed by the central
government.
• In pursuance of its mandate, the NIA collects, collates and analyses counter-terrorism
investigation. It also shares inputs with its sister intelligence agencies and law enforcement units
both at central and state governments level.
• The NIA aims at creating deterrence for existing and potential terrorist groups/ individuals. The
NIA aims to develop as a storehouse of all terrorist- related information.

263. (c)
Explanation
Self-Explanatory

264. (b)
Explanation
• The SC recommended establishing a truth and reconciliation commission, similar to post-
apartheid South Africa, to investigate human rights violations by both state and non-state actors.

265. (b)
Explanation

4
For all current updates on polity and governance, please follow Saurabh Sir's Telegram Channel
"Polity by Saurabh Kumar”

• Giving to Earth Amplify Action (GAEA) was introduced at the World Economic Forum’s Annual
Meeting 2023, at Davos, with the theme being ‘Cooperation in a Fragmented World.’ The primary
goal of this new program is to make available $3 trillion in annual funding for activities aimed at
achieving net zero emissions, reversing nature loss, and restoring biodiversity by 2050.The fund
is to be accumulated through Public-private and philanthropic partnerships (PPPP).
• ⇒ The Global Risks Report is an annual study published by the World Economic Forum ahead of
the Forum's Annual Meeting in Davos, Switzerland. The Global Risks Report 2023 explores some
of the most severe risks we may face over the next decade. Major global risks in the next 2 years
include the cost of living, natural disasters, and extreme weather events, as well as geoeconomics
confrontation.
• Over the next 10 years, the risks encompass failure to mitigate climate change, biodiversity loss,
and ecosystem collapse.
• FireAID Initiative was launched in 2022 by the World Economic Forum (WEF). It uses Artificial
Intelligence to allow better prediction of wildfires and efficient use of resources during firefighting
operations.
• It is a multi-stakeholder initiative, which pools resources from governments, civil society, and the
private sector, was initiated by Koç Holding, Turkey’s largest industrial conglomerate.
• Deloitte, an international professional services network, joined the initiative and contributed its
own AI technology. This technology was employed to develop a 'digital twin' specifically for fire
management.

266. (a)
Explanation
• Only one (second statement)
• First Information Report, is the written document that police prepare upon receiving information
regarding the occurrence of a cognizable offence. It is an important document both from the point
of view of prosecution and defence. It acts as the foundation of the case, as the entire case depends
on it.
• F.I.R can also be registered by the Judicial Magistrate by giving the direction to the concerned
jurisdictional area of the Police Station.
• An FIR can be lodged by the victim, a witness to the incident, or any person with knowledge of the
incident.
• FIR once lodged cannot be withdrawn by the Informant. it can only be quashed by the inherent
power vested upon the High court by virtue of Section 482 crpc or BNSS section 582.
• Chargesheet defined under Section 173 CrPC: It is final formal document of accusation that the
police prepare. It contains the opinion of the investigating officer that he/she has collected
sufficient material for the trial of the accused by the court. It can be filed by Investigation Officer
or police officer and filed in court.
• The time limit to file charge sheet is related to arrest of the accused in the case. The charge sheet
is to be filed within 60 days from the date of arrest of the accused in cases triable by lower courts
and 90 days in cases triable by Court of Sessions.
• If the charge sheet is not filed within the prescribed time mentioned above, the accused has a
right to default bail.

5
For all current updates on polity and governance, please follow Saurabh Sir's Telegram Channel
"Polity by Saurabh Kumar”

• Charge sheet can be quashed by the High Court as per the provisions of Section 483 of CrPC, the
court has an inherent power that it can exercise to prevent the abuse of the power and to meet
the ends of the justice.

267. (a)
Explanation
• Only one (statement 3)
• The United Nations General Assembly(UNGA) adopted a resolution titled ‘Education for
Democracy’ that reaffirms the right of everyone to education. The resolution, which was co-
sponsored by India, recognises that “education for all” contributes to the strengthening of
democracy. The resolution encourages member states to integrate education for democracy into
their education standards.
• This isn’t the first time that an education resolution has been adopted by the UN General
Assembly. Earlier in 2015, a similar resolution was passed at the Assembly encouraging all UN
entities to use education to promote peace, human rights, and democracy.

268. (b)
Explanation
• Only two (statement 1 and 2)
• India set a goal in 2019 to eradicate measles and rubella by 2023 after missing the previous
deadline of 2020.
• Intensified Mission Indradhanush 5.0 (IMI 5.0) campaign aims to enhance immunisation coverage
for all vaccines provided under the Universal Immunization Programme (UIP) as per the National
Immunization Schedule (NIS). Special focus is on improvement of Measles and Rubella vaccination
coverage.
• Rubella is caused by a different virus than measles
• Rubella is a contagious viral infection best known by its distinctive red rash. It's also called
German measles or three-day measles. It is caused by the rubella virus which is an enveloped
single-stranded RNA virus.
• Measles is a highly contagious viral infection caused by a single-stranded, enveloped RNA virus
with 1 serotype.
• Rubella and Measles share some signs and symptoms, such as the red rash.

269. (c)
Explanation
• UN Habitat: - Leaves no one Behind project draws on the collective expertise of UN-Habitat,
UNESCAP and UN-OICT to use the convening power for addressing the complex, cross-sectoral
challenges facing South Asian urban policy makers aiming to mainstream inclusive, safe concepts
into urban policy and planning frameworks by engaging national, sub-national and local
governments.
• Strengthened regional network of governments and local urban institutions to promote and
mainstream LNOB in urban development agenda, specifically SDGs 11 & 6.
• The project aims to include disabled and other marginalised groups in the course of urban
development.

270. (c)
Explanation

6
For all current updates on polity and governance, please follow Saurabh Sir's Telegram Channel
"Polity by Saurabh Kumar”

• World Social Report 2023 published biennial by the United Nations Department of Economic and
Social Affairs (UN DESA), with the theme “Leaving No One Behind in an Aging World. This report
explores the socio-economic opportunities and challenges that arise from the aging population.
• The World Social Report (formerly known as the Report on the World Social Situation) has been
published since 1997.

271. (c)
Explanation
• Period poverty is a lack of access to menstrual products, education, hygiene facilities, waste
management, or a combination of these

272. (c)
Explanation
• Living will be a person’s right to make an advance directive about the course of his or her
treatment, including the removal of life support, if such a situation arises.
• In the Aruna Shanbaug case (2011), the Supreme Court permitted passive euthanasia. In the
Puttaswamy case, the Supreme Court stated that Article 21 encompasses the concept of individual
dignity, thus permitting passive euthanasia.
• As per 2018 guidelines, a living will be required to be signed by an executor (the individual seeking
euthanasia) in the presence of two attesting witnesses, preferably independent, and to be further
countersigned by a Judicial Magistrate of First Class (JMFC).
• Also, the treating physician was required to constitute a board comprising three expert medical
practitioners from specific but varied fields of medicine, with at least 20 years of experience, who
would decide whether to carry out the living will or not. If the medical board granted permission,
the will had to be forwarded to the District Collector for his approval.
• The Collector was to then form another medical board of three expert doctors, including the Chief
District Medical Officer. Only if this second board agreed with the hospital board’s findings would
the decision be forwarded to the JMFC, who would then visit the patient and examine whether to
accord approval.
• This cumbersome process will now become easier.
➢ Instead of the hospital and Collector forming the two medical boards, both boards will now be
formed by the hospital. The requirement of 20 years of experience for the doctors has been
relaxed to five years. The requirement for the Magistrate’s approval has been replaced by an
intimation to the Magistrate. The medical board must communicate its decision within 48
hours; the earlier guidelines specified no time limit.
➢ The 2018 guidelines required two witnesses and a signature by the Magistrate; now a notary
or gazetted officer can sign the living will in the presence of two witnesses instead of the
Magistrate’s countersign. In case the medical boards set up by the hospital refuses permission,
it will now be open to the kin to approach the High Court which will form a fresh medical team.

273. (d)
Explanation
• Five UN agencies including WHO is calling for accelerated progress on the Global Action
Plan on Child Wasting.
• These agencies are Food and Agriculture Organization (FAO), UN Refugee Agency
(UNHCR), United Nations Children’s Fund (UNICEF), World Food Programme (WFP) and
WHO.

7
For all current updates on polity and governance, please follow Saurabh Sir's Telegram Channel
"Polity by Saurabh Kumar”

• The action plan aims to prevent, detect and treat acute malnutrition among children
in the worst-affected countries: Afghanistan, Burkina Faso, Chad, etc.

274. (d)
Explanation
• Special Marriage Act 1954:
1. When a person solemnises marriage under Special Marriage Act, then the marriage is not
governed by personal laws but by the Special Marriage Act.
2. The applicability of the Act extends to the people of all faiths, including Hindus, Muslims,
Sikhs, Christians, Sikhs, Jains, and Buddhists, across India. It applies to Indians and
Foreigners too.
3. The parties to the marriage are required to give a notice, in writing, to a “Marriage Officer” of
the district in which at least one of the parties has resided for at least 30 days immediately
preceding the notice.
4. The parties and three witnesses are required to sign a declaration form before the Marriage
Officer.
5. Once the declaration is accepted, the parties will be given a “Certificate of marriage” which is
essentially proof of the marriage.

275. (c)
Explanation
• The National Commission for Protection of Child Rights (NCPCR) is an Indian statutory body
established by an Act of Parliament, the Commission for Protection of Child Rights (CPCR) Act,
2005. The Commission works under the aegis of Ministry of Women and Child Development.
• The National Commission for Protection of Child Rights (NCPCR) emphasizes the principle of
universality and inviolability of child rights and recognizes the tone of urgency in all the child
related policies of the country.
• For the Commission, protection of all children in the 0 to 18 years’ age group is of equal
importance.
• The Commission is further mandated to monitor the proper and effective implementation of:
Protection of Children from Sexual Offences (POCSO) Act, 2012, Juvenile Justice (Care and
Protection of Children) Act, 2015 and Right to Free and Compulsory Education (RTE) Act, 2009.

276. (a)
Explanation
• Data on migration according to 2011 Census.
• In 2011, India had 45.6 crore migrants, which is 38% of the population.
• Between 2001 and 2011, while the population increased by 18%, the number of migrants went
up by 45%.
• Almost all of the migration (99%) was within the country, and only 1% involved international
immigrants.
• Out of the internal migrants, 21 crores were moving from one rural area to another, making up
54% of the classified internal migration.
• Nearly 88% of all internal migration happened within the same state.
• Uttar Pradesh and Bihar were the main sources of people moving to other states, while
Maharashtra and Delhi received the most migrants.

8
For all current updates on polity and governance, please follow Saurabh Sir's Telegram Channel
"Polity by Saurabh Kumar”

• The majority (70%) of within-state migration occurred due to reasons related to marriage and
family.

277. (c)
Explanation
• The National Green Hydrogen Mission, implemented by the Ministry of New and Renewable Energy
from FY 2023–24 to FY 2029–30, aims to establish India as a global hub for the production, usage,
and export of Green Hydrogen and its derivatives.
• The Strategic Interventions for Green Hydrogen Transition (SIGHT) Programme, a subcomponent
of the National Green Hydrogen Mission, seeks to enhance domestic electrolyzer manufacturing
and green hydrogen production.
• The Strategic Hydrogen Innovation Partnership (SHIP), also a subcomponent of the National Green
Hydrogen Mission, operates as a Public-Private Partnership framework, incorporating dedicated
R&D funds with inputs from both the industry and the government.

278. (b)
Explanation
• EQUIP has been launched by the Department of Higher Education, Ministry of Human Resource
Development which sets out to deliver further on the principles of access, inclusion, quality,
excellence, and enhancing employability in higher education.
• EQUIP aims at ushering transformations in India’s higher education system by implementing
strategic interventions in the sector over five years (2019 – 2024).
• Double the Gross Enrolment Ratio (GER) in higher education and resolve the geographically and
socially skewed access to higher education institutions in India. The gross enrolment ratio in
higher education in India was 25.18 in 2017-18. Through EQUIP, the target is to raise this ratio
up to 52 by 2024.
• Upgrade the quality of education to global standards
• Position at least 50 Indian institutions among the top-1000 global universities and to support 20
prominent higher education institutions in India to get global top 200 rankings.
• Funding plans of EQUIP include - financial autonomy of institutions, increasing state funding of
public institutions, mobilization of CSR funds, an alumni donation system, and affordable
educational loans for students.

279. (a)
Explanation
• Global report on Neglected Tropical Diseases (NTD) 2023 released by the World Health
Organisation (WHO).
• Neglected tropical diseases are a diverse group of conditions caused by a variety of pathogens
(including viruses, bacteria, parasites, fungi and toxins) and associated with devastating health,
social and economic consequences.
• NTDs are mainly prevalent among impoverished communities in tropical areas, although some
have a much larger geographical distribution.
• It is estimated that NTDs affect more than 1 billion people, while the number of people requiring
NTD interventions (both preventive and curative) is 1.6 billion.
• NTDs include: Buruli ulcer; Chagas disease; dengue and chikungunya; foodborne trematodiases;
human African trypanosomiasis; leishmaniasis; leprosy; lymphatic filariasis etc.

9
For all current updates on polity and governance, please follow Saurabh Sir's Telegram Channel
"Polity by Saurabh Kumar”

280. (b)
Explanation
• The NPS is a voluntary and long-term retirement investment plan administered by the Pension
Fund Regulatory and Development Authority (PFRDA), Ministry of Finance, Government of India.
It was launched in January 2004 for government employees.
• The Old Pension Scheme was discontinued in 2004, however, it guaranteed life-long income after
retirement.
• Under the OPS, the government pays the entire pension amount to government employees after
retirement. Thus, no amount is deducted from employees’ salaries when they are in service while
in NPS, contribution of employee depends on him but not more than 10% of basic pay.
• Under the old pension system, the retired government employees receive 50 per cent of their last
drawn salary plus dearness allowance as pension every month, while this aspect is not considered
in the new pension system.

281. (d)
Explanation
• Special Category Status is a classification given by the Centre to assist development of states that
face geographical and socio-economic disadvantages.
• Eleven States including Assam, Nagaland, Himachal Pradesh, Manipur, Meghalaya, Sikkim,
Tripura, Arunachal Pradesh, Mizoram, Uttarakhand and Telangana have been accorded the
special category state status.
• Criteria for SCs based on Gadgil Mukherjee formula:
(i.) Hilly and difficult terrain
(ii.) Low population density and/or sizeable share of tribal population
(iii.) Strategic location along international borders
(iv.) Economic and infrastructural backwardness and
(v.) Nonviable nature of state finances.
• The central government bears 90 percent of the state expenditure on all centrally-sponsored
schemes and external aid while rest 10 percent is given as loan to state at zero percent rate of
interest.
• 30 percent of the Centre's gross budget also goes to special category states.
• These states can avail the benefit of debt-swapping and debt relief schemes.
• Exempted from customs duty, corporate tax, income tax and other taxes to attract investment.

282. (a)
Explanation
• The idea of reformative criminal justice system is inspired from Patanjali Sutra

283. (c)
Explanation
• Member of Parliament Local Area Development (MPLAD) scheme, launched in 1993 to provide a
mechanism for the Members of Parliament to recommend works of developmental nature for
creation of durable community assets and for provision of basic facilities including community
infrastructure, based on locally felt needs.
• It fully funded by Government of India and annual MPLADS fund entitlement per MP constituency
is Rs. 5 crores.

10
For all current updates on polity and governance, please follow Saurabh Sir's Telegram Channel
"Polity by Saurabh Kumar”

• The funds released under the scheme are non-lapsable.


• Every year, Members of Parliament (MPs) are expected to recommend works costing at least 15
percent of the MPLADS entitlement for areas inhabited by Scheduled Caste populations and 7.5
percent for areas inhabited by Scheduled Tribe populations.
• Interest earnings from funds released under MPLADs will no longer be added to the fund and used
for development works. Instead, the interest earned has to be mandatorily remitted to the
Consolidated Fund of India (CFI.)
• The role of an MP is, however, limited to the recommendation of a project. The onus is on the
district authority to sanction, execute and complete the recommended project within a particular
timeframe.

284. (c)
Explanation
• WHO recommends that “National Blood System should be governed by National Blood Policy and
legislative framework to promote uniform implementation of standards and consistency in the
quality and safety of blood and blood products”
• In accordance with the directive of the Supreme Court, National Blood Transfusion Council was
constituted in 1996 with the objectives to promote voluntary blood donation, ensure safe blood
transfusion, provide infrastructure to blood centres, develop human resource and formulate and
implement the Blood Policy.
• National Blood Transfusion Council (NBTC) is the policy formulating apex body in relation to all
matters pertaining to operation of blood centres. The NBTC is the central body that coordinates
the State Blood Transfusion Councils (SBTCs) and also ensures involvement of other Ministries
and other health programmes for various activities relate to Blood Transfusion Services (BTS).
• NBTC is supported by a National Transfusion Services Core Coordination Committee under the
chairmanship of Director General of Health Services.
• Presently it is housed within the National AIDS Control Program.

285. (d)
Explanation
• The UB 2016-17 announced 10 publics and 10 private HEIs are to be recognised to emerge as
world-class institutions.
• The institutions should be among the top 50 in the NIRF in their category, and top 500 in
internationally recognised rankings.
• Public institutes eligible to apply are central universities, govt-owned and controlled deemed to be
universities, institutions of national importance (IITs, NITs), and state universities.
• Only the selected public institutions will get Rs 1,000 crore each from the Ministry of Education.

286. (b)
Explanation
• Global Millet (Sree Anna) Conference has been organised by the Agricultural and Processed
Food Products Export Development Authority (APEDA), Ministry of Commerce and Industry.
• India produces more than 170 lakh tonnes of millet, which is 80 percent of Asia’s and 20 percent
of global production.
• Maximum Production of Millets is in Africa and minimum is in Australia and New Zealand.

11
For all current updates on polity and governance, please follow Saurabh Sir's Telegram Channel
"Polity by Saurabh Kumar”

287. (c)
Explanation
• The implementation of the convention by parties is monitored by the Committee against Torture,
which is a body of experts in human rights.
• It is one of the eight United Nations-linked bodies for human rights treaties.
• All State parties to the UNCAT should regularly submit reports to the Committee on how they are
implementing rights.
• Once a country has ratified the convention, it should submit a report within a year. After that,
they are expected to submit a report once in four years.
• The Committee can also entertain complaints from individuals regarding a violation of their rights
under the convention.
• India has not ratified UNCAT, although it signed it in 1997.

288. (b)
Explanation
• DSCI is a not-for-profit premier industry body on data protection in India, setup by NASSCOM,
committed to making cyberspace safe, secure and trusted by establishing best practices,
standards and initiatives in cyber security and privacy.
• DSCI also endeavors to increase India’s share in the global security product and services market
through global trade development initiatives. These aim to strengthen the security and privacy
culture in the India.

289. (a)
Explanation
• It is wholly owned by the Government of India and was set up in 1957 with the objective of
promoting exports from the country.
• It is owned by the Ministry of Commerce and Industry.
• The insurance covers provided by ECGC enables the banks to extend timely and adequate export
credit facilities to the exporters.

290. (c)
Explanation
• HEFA has been set up in 2017 by the Central Government as a Non Profit, Non-Banking Financing
Company (NBFC) for mobilising extra-budgetary resources for building crucial infrastructure in
the higher educational institutions under Central Govt.
• It is a joint venture of the Ministry of Education, GoI and Canara Bank with agreed equity
participation in the ratio of 90.91% and 09.09% respectively.

291. (b)
Explanation
• The Advertising Standards Council of India (ASCI) is a voluntary self-regulatory organization of
the advertising industry in India. Established in 1985, ASCI is registered as a non-profit company
under section 25 of the Company Act.
• The Advertising Standards Council of India (ASCI), is committed to the cause
of self-regulation in advertising, ensuring the protection of the interests of consumers. ASCI

12
For all current updates on polity and governance, please follow Saurabh Sir's Telegram Channel
"Polity by Saurabh Kumar”

seeks to ensure that advertisements conform to its Code for Self-Regulation, which requires
advertisements to be legal, decent, honest and truthful, and not hazardous or harmful, while
observing fairness in competition.

292. (c)
Explanation
• The Ministry of Health in collaboration with World Health Organization (WHO) formally announced
the re-designation of National Health Systems Resource Centre (NHSRC), as the WHO
Collaborating Centre for Priority Medical Devices and Health Technology Policy.
• It has been set up under the National Health Mission (NHM) of Government of India to serve as
an apex body for technical assistance.
• It has a 23-member Governing Body, chaired by the Secretary, Ministry of Health, Government of
India.

293. (a)
Explanation
• An entity shall be considered as a start-up (meaning of Start-up) if it satisfies all the following
conditions: a. If it is incorporated/registered as any of the following:
1. Private Limited Company (as defined in Companies Act, 2013).
2. Partnership Firm (registered under Partnership Act, 1932).
3. Limited Liability Partnership (registered under Limited Liability Partnership Act, 2008).
4. One Person Company (as defined in Companies Act, 2013).
• Provided that such an entity is not formed by splitting up or reconstruction of a business already
in existence.
• It has not completed ten years since incorporation/registration as above.
• Its turnover for any of the financial years has not exceeded INR 100 Crore.
• It satisfies any of the following conditions:
• It is working towards: Innovation of new products/processes/services or Development of new
products/processes/services or Improvement of existing products/processes/services.
• It is a scalable business model with a high potential of: Employment generation or Wealth creation.

294. (a)
Explanation
• MANAGE was established in 1987, as the National Centre for Management of Agricultural
Extension at Hyderabad, by the Ministry of Agriculture & Farmers Welfare, Government of India
as an autonomous Institute, from which its acronym ‘MANAGE’ is derived.
• In recognition of its importance and expansion of activities all over the country, its status was
elevated to that of a National Institute in 1992 and re-christened to its present name i.e., National
Institute of Agricultural Extension Management.
• Feed the Future is America’s initiative to combat global hunger. Led by USAID, the initiative
brings partners together to address the root causes of hunger and poverty by boosting agriculture-
led growth, resilience and nutrition in countries with great need and opportunity for improvement.
• It is Created in the wake of the devastating 2007 and 2008 food price spikes.
• A new Agriculture Partnership between US and India to achieve the Ever Green Revolution to
address Global Food Security was announced during the State visit of US President Mr. Barak
Obama to India in November 2010. The effort included Triangular Cooperation adapting

13
For all current updates on polity and governance, please follow Saurabh Sir's Telegram Channel
"Polity by Saurabh Kumar”

technological advances and innovative solutions to address Food Security Challenges in Africa.
This Pilot Stage focused on three African Countries i.e., Kenya, Liberia and Malawi with potential
to expand throughout the African Continent in future. Consequently, National Institute of
Agricultural Extension Management (MANAGE), Hyderabad and National Institute of Agricultural
Marketing (NIAM), Jaipur conducted training programs.

295. (d)
Explanation
• One District One Product (ODOP) programme was launched by the Ministry of Food Processing
Industries in 2018, to help districts reach their full potential, foster economic and socio-cultural
growth, and create employment opportunities, especially, in rural areas.

296. (d)
Explanation
• The Ministry of Social Justice and Empowerment in 1993 under the Government of India has
started a policy—Mahila Samriddhi Yojana, to encourage women entrepreneurs and give
momentum to their journey as entrepreneurs.
• The Regional Rural Banks (RRB) and nationalised banks offer loans to women who need capital
to start or progress their businesses.
• The target groups under this scheme are as under:
1. Self-Help Groups (SHGs)—These are the economically dependent groups of individuals who
form SHGs to contribute funds to their members.
2. Women from low-income families or the backward classes (caste or class a tribe category).

• Provision for Finance in Mahila Samriddhi Yojana:


1. The loan amount that is sanctioned per beneficiary is Rs. 1,40,000. The amount of loan
received shall be utilised within 4 months from the date of disbursement.
2. The loan sanctioned after the Mahila Samriddhi Yojana application is 95%, and the remaining
balance of 5% will be provided by the State Channelizing Agency (SCA) or beneficiary
contribution.
3. The loan provided under Mahila Samriddhi Yojana is to be paid back in 3.5 years in quarterly
instalments from the date on which the loan was paid out to the applicant’s bank account. A
moratorium period of 3 months is provided under the scheme.
4. The interest rate for the government to State Channelizing Agencies (SCA) is 1% per annum,
and the rate for SCAs to beneficiaries is 4% per annum.

297. (c)
Explanation
• The scheme has two components one for urban India and other for rural India.
1. The Urban component named as Deen Dayal Antyodaya Yojana will be implemented by the
Ministry of Housing and Poverty Alleviation.
2. The rural component named as Deen Dayal Upadhaya Grameen Kaushal Yojna will be
implemented by the Ministry of Rural Development.
• MAIN HIGHLIGHTS OF THE SCHEME
1. Employment through Skill Training and Placement - An expenditure of Rs.15, 000 per person
is allowed on training of urban poor which is Rs.18, 000 in North-East and J&K. Moreover,

14
For all current updates on polity and governance, please follow Saurabh Sir's Telegram Channel
"Polity by Saurabh Kumar”

Training urban poor to meet the enormous demand from urban citizens by imparting market-
oriented skills through City Livelihood Centers.
2. Social Mobilization and Institution Development - It will be done through formation of Self-
Help Groups (SHG) for training members and hand holding, an initial support of 10, 000 is
given for each group. Assistance of Rs.50, 000 is provided to Registered Area Level Federations.
3. Subsidy to urban poor - An interest subsidy of 5% - 7% for setting up individual micro-
enterprises with a loan of up to 2 lakhs and for group enterprises with a loan limit of up to
Rs.10 lakhs.
4. Shelters for urban homeless - Cost of construction of shelters for urban homeless is fully
funded under the Scheme.
5. Other means - Development of vendor markets and also the promotion of skills for the vendors
through setting up infrastructure and special projects for the rag picker and differently abled
etc.

298. (c)
Explanation
• The program includes a range of schemes related to poverty alleviation, employment generation,
education, health, housing, and environmental protection.
• The Ministry of Statistics and Programme Implementation (MOSPI) monitors the performance of
the TPP based on reports received from state governments and central nodal ministries.

299. (c)
Explanation
• It is a not-for-profit industry body representing the interests of the online and mobile value added
services industry. It is registered under The Societies Registration Act, 1860.
• It was established in 2004 by leading online publishers.

300. (d)
Explanation
• A disturbed area is one which is declared by notification under Section 3 of the AFSPA. An area
can be disturbed due to differences or disputes between members of different religious, racial,
language or regional groups or castes or communities.
• The Central Government, or the Governor of the State or administrator of the Union Territory can
declare the whole or part of the State or Union Territory as a disturbed area. A suitable notification
would have to be made in the Official Gazette. As per Section 3, it can be invoked in places where
“the use of armed forces in aid of the civil power is necessary”. Thus statement 1 is Correct.
• Under Section 4 of the AFSPA, an authorised officer in a disturbed area enjoys certain powers.
The authorised officer has the power to open fire at any individual even if it results in death if the
individual violates laws which prohibit (a) the assembly of five or more persons; or (b) carrying of
weapons. However, the officer has to give a warning before opening fire. Thus statement 2 is
Correct.
• It is effective in the whole of Nagaland, Assam, Manipur (excluding seven assembly constituencies
of Imphal) and parts of Arunachal Pradesh. In Arunachal Pradesh, the impact of AFSPA was
reduced to eight police stations. Thus statement 3 is Correct.

15

You might also like